Vous êtes sur la page 1sur 92

See discussions, stats, and author profiles for this publication at: https://www.researchgate.

net/publication/373097527

Problèmes du Club de Maths

Preprint · July 2023


DOI: 10.13140/RG.2.2.18317.10726

CITATIONS READS

0 229

1 author:

Paul Averous
Université Paris-Saclay
1 PUBLICATION 0 CITATIONS

SEE PROFILE

All content following this page was uploaded by Paul Averous on 13 August 2023.

The user has requested enhancement of the downloaded file.


Problèmes du Club de Maths
Paul Averous
2022-2023

Résumé
Ceci est un récapitualtif des problèmes proposés au club de mathématiques de Hoche du-
rant l’année scolaire 2022-2023. Ils sont catégorisés en 4 niveaux de difficulté (Apéritif, Entrée,
Plat, Dessert), parmi lesquels vous trouverez des exercices des 4 grands domaines des maths
olympiques : l’arithmétique, l’algèbre, la combinatoire et la géométrie.

Ce document présente également des solutions des exercices proposés. Il faut cependant noter
que le cours pour aborder ces exercices n’est pas présent, meme si les principaux résultats
démontrés et utiles sont énoncé à la fin de document. Au sein d’une meme catégorie, les exer-
cices ne sont pas classés par ordre de difficulté. Enfin, un certain nombre d’exercices sont
des problèmes présents sur le site Mathraining - dont j’encourage vivement les élèves voulant
progresser en mathématiques olympiques à prendre connaissance - donc je n’en donne pas de
correction, seulement la référence.

Toute question à propos d’un exercice ou d’une correction peut m’etre adressée par mail à
l’adresse : paul.averous@gmail.com
Veuillez me demander avant de diffuser ce document.

Un grand merci également à Paul Perrier d’avoir proposé un certain nombre d’exercices de
combinatoire et d’équations fonctionnelles et à Amélie Triqueneaux pour certains exercices de
géométrie.

Voici un récapitulatif des séances du club cette année :

Séance Thème
1 Objets mathématiques essentiels et stratégies de base
2 Géométrie : Chasse aux angles
3 Combinatoire : Principe des tiroirs, principe des extrema
4 Arithmétique : Divisibilité, pgcd et nombres premiers
5 Combinatoire : Dénombrement
6 Algèbre : Introduction aux inégalités
7 Pot-Pourri
8 Arithmétique : Divers
9 Géométrie : Chasse aux angles
10 Combinatoire : Pavages
11 Arithmétique : Arithmétique modulaire et classiques
12 Combinatoire : Invariants
13 Algèbre : Introduction aux suites
14 Combinatoire : Divers
15 Pot-Pourri
16 Géométrie : Divers
17 Combinatoire : Géométrie-combinatoire
18 Géométrie : Divers
19 Algèbre : Equations fonctionnelles
20 Arithmétique : Divers
21 Algèbre : Introduction aux polynomes
22 Pot-Pourri
23 Algèbre : Introduction aux complexes
24 Algèbre : Introduction aux complexes
25 Algèbre : Inégalités
26 Pot-Pourri
27 Arithmétique : Autour du theoreme d’Euler-Fermat
28 Arithmétique : Autour du theoreme d’Euler-Fermat
29 Géométrie : Tranformations du plan, homothéties

1
Table des matières

1 Exercices 3
1.1 Apéritif . . . . . . . . . . . . . . . . . . . . . . . . . . . . . . . . . . . . . . . . . . 3
1.1.1 Arithmétique . . . . . . . . . . . . . . . . . . . . . . . . . . . . . . . . . . . 3
1.1.2 Algèbre . . . . . . . . . . . . . . . . . . . . . . . . . . . . . . . . . . . . . . 5
1.1.3 Combinatoire . . . . . . . . . . . . . . . . . . . . . . . . . . . . . . . . . . . 7
1.1.4 Géométrie . . . . . . . . . . . . . . . . . . . . . . . . . . . . . . . . . . . . . 9
1.2 Entrée . . . . . . . . . . . . . . . . . . . . . . . . . . . . . . . . . . . . . . . . . . . 11
1.2.1 Arithmétique . . . . . . . . . . . . . . . . . . . . . . . . . . . . . . . . . . . 11
1.2.2 Algèbre . . . . . . . . . . . . . . . . . . . . . . . . . . . . . . . . . . . . . . 13
1.2.3 Combinatoire . . . . . . . . . . . . . . . . . . . . . . . . . . . . . . . . . . . 15
1.2.4 Géométrie . . . . . . . . . . . . . . . . . . . . . . . . . . . . . . . . . . . . . 17
1.3 Plat . . . . . . . . . . . . . . . . . . . . . . . . . . . . . . . . . . . . . . . . . . . . 19
1.3.1 Arithmétique . . . . . . . . . . . . . . . . . . . . . . . . . . . . . . . . . . . 19
1.3.2 Algèbre . . . . . . . . . . . . . . . . . . . . . . . . . . . . . . . . . . . . . . 21
1.3.3 Combinatoire . . . . . . . . . . . . . . . . . . . . . . . . . . . . . . . . . . . 23
1.3.4 Géométrie . . . . . . . . . . . . . . . . . . . . . . . . . . . . . . . . . . . . . 25
1.4 Dessert . . . . . . . . . . . . . . . . . . . . . . . . . . . . . . . . . . . . . . . . . . 26
1.4.1 Arithmétique . . . . . . . . . . . . . . . . . . . . . . . . . . . . . . . . . . . 26
1.4.2 Algèbre . . . . . . . . . . . . . . . . . . . . . . . . . . . . . . . . . . . . . . 27
1.4.3 Combinatoire . . . . . . . . . . . . . . . . . . . . . . . . . . . . . . . . . . . 30
1.4.4 Géométrie . . . . . . . . . . . . . . . . . . . . . . . . . . . . . . . . . . . . . 32

2 Solutions des Exercices 33


2.1 Apéritif . . . . . . . . . . . . . . . . . . . . . . . . . . . . . . . . . . . . . . . . . . 33
2.1.1 Arithmétique . . . . . . . . . . . . . . . . . . . . . . . . . . . . . . . . . . . 33
2.1.2 Algèbre . . . . . . . . . . . . . . . . . . . . . . . . . . . . . . . . . . . . . . 37
2.1.3 Combinatoire . . . . . . . . . . . . . . . . . . . . . . . . . . . . . . . . . . . 40
2.1.4 Géométrie . . . . . . . . . . . . . . . . . . . . . . . . . . . . . . . . . . . . . 43
2.2 Entrée . . . . . . . . . . . . . . . . . . . . . . . . . . . . . . . . . . . . . . . . . . . 48
2.2.1 Arithmétique . . . . . . . . . . . . . . . . . . . . . . . . . . . . . . . . . . . 48
2.2.2 Algèbre . . . . . . . . . . . . . . . . . . . . . . . . . . . . . . . . . . . . . . 51
2.2.3 Combinatoire . . . . . . . . . . . . . . . . . . . . . . . . . . . . . . . . . . . 55
2.2.4 Géométrie . . . . . . . . . . . . . . . . . . . . . . . . . . . . . . . . . . . . . 57
2.3 Plat . . . . . . . . . . . . . . . . . . . . . . . . . . . . . . . . . . . . . . . . . . . . 61
2.3.1 Arithmétique . . . . . . . . . . . . . . . . . . . . . . . . . . . . . . . . . . . 61
2.3.2 Algèbre . . . . . . . . . . . . . . . . . . . . . . . . . . . . . . . . . . . . . . 64
2.3.3 Combinatoire . . . . . . . . . . . . . . . . . . . . . . . . . . . . . . . . . . . 67
2.3.4 Géométrie . . . . . . . . . . . . . . . . . . . . . . . . . . . . . . . . . . . . . 69
2.4 Dessert . . . . . . . . . . . . . . . . . . . . . . . . . . . . . . . . . . . . . . . . . . 73
2.4.1 Arithmétique . . . . . . . . . . . . . . . . . . . . . . . . . . . . . . . . . . . 73
2.4.2 Algèbre . . . . . . . . . . . . . . . . . . . . . . . . . . . . . . . . . . . . . . 75
2.4.3 Combinatoire . . . . . . . . . . . . . . . . . . . . . . . . . . . . . . . . . . . 79
2.4.4 Géométrie . . . . . . . . . . . . . . . . . . . . . . . . . . . . . . . . . . . . . 82

3 Récapitulatif des résultats 86


3.1 Résultats généraux . . . . . . . . . . . . . . . . . . . . . . . . . . . . . . . . . . . . 86
3.2 Arithmétique . . . . . . . . . . . . . . . . . . . . . . . . . . . . . . . . . . . . . . . 87
3.2.1 Divisibilité et nombres premiers . . . . . . . . . . . . . . . . . . . . . . . . . 87
3.2.2 Congruences . . . . . . . . . . . . . . . . . . . . . . . . . . . . . . . . . . . 87
3.3 Algèbre . . . . . . . . . . . . . . . . . . . . . . . . . . . . . . . . . . . . . . . . . . 88
3.3.1 Suites . . . . . . . . . . . . . . . . . . . . . . . . . . . . . . . . . . . . . . . 88
3.3.2 Polynomes . . . . . . . . . . . . . . . . . . . . . . . . . . . . . . . . . . . . 88
3.3.3 Inégalités . . . . . . . . . . . . . . . . . . . . . . . . . . . . . . . . . . . . . 88
3.4 Combinatoire . . . . . . . . . . . . . . . . . . . . . . . . . . . . . . . . . . . . . . . 90
3.5 Géométrie . . . . . . . . . . . . . . . . . . . . . . . . . . . . . . . . . . . . . . . . . 91

2
1 Exercices

1.1 Apéritif

Ces exercices sont très proches du cours, ils servent de mise en jambe. L’objectif de toute séance
est d’avoir au moins terminé les exercices de l’apéritif. Parmi eux se trouvent certains résultats
classiques qu’il est bon de retenir. Les plus importants sont de toute manière rappelés à la fin de
ce document.

1.1.1 Arithmétique

Exercice 1. Solution
A l’aide d’un raisonnement direct, montrer que (a − b)(a + b) = a2 − b2

Exercice 2. Solution
Soit n ∈ N, A l’aide d’un raisonnement par contraposée, montrer que si n2 est pair, alors n est
pair.

Exercice 3. Solution
1
Montrer que n’est pas un nombre décimal.
3

Exercice 4. Solution
n(n + 1)
Soit n ∈ N∗ . Montrer par récurrence que 1 + 2 + 3 + . . . + n =
2
Exercice 5. Solution
Montrer que si a | b et c | d, alors ac | bd

Exercice 6. Solution
Déterminer tous les entiers naturels n ∈ N vérifiant n | n + 7

Exercice 7. Solution
Déterminer tous les entiers naturels n ∈ N vérifiant n2 + 1 | n

Exercice 8. Solution
Déterminer le pgcd de 364 et 154

Exercice 9. Solution
Montrer que ∀n ∈ N, n(n + 1)(2n + 1) est divisible par 6.
n(n + 1)(2n + 1)
Rédmontrer le résultat en montrant que ∀n ∈ N, 12 + . . . + n2 =
6
Exercice 10. Solution
Quels sont les entiers n tels que n et n + 2 sont premiers entre eux ?
Quels sont les entiers n tels que n2 − 1 et n2 − 2n + 1 sont premiers entre eux ?

Exercice 11. Solution


Montrer que si x et y sont des entiers tels que 2x + 1 divise 8y alors 2x + 1 divise y

Exercice 12. Solution


Trouver tous les entiers p tels que :
• p et p + 1 sont premiers.
• p, p + 2 et p + 4 sont premiers.

Exercice 13. Solution


98473092 est-il divisible par 11 ?

3
Exercice 14. Solution
Montrer que si a2 | b2 alors a | b

Exercice 15. Solution


21n + 4
Montrer que pour tout entier n, est irréductible.
14n + 3
Source : IMO 1959

Exercice 16. Solution


Montrer que pour tout entier naturel, n3 − 7n est divisible par 6
Source : Envoi d’arithmétique 2022

Exercice 17. Solution


Déterminer le chiffre des unités de 32023

Exercice 18. Solution


Trouver tous les couples d’entiers (x, y) tels que x2 = 13 + y 2

Exercice 19. Solution


Trouver tous les triplets d’entiers (x, y, z) tels que x2 + y 2 = 4z + 3

Exercice 20. Solution


Trouver tous les triplets d’entiers (x, y, z) tels que x! + 2y = z!
Source : Envoi Pot-Pourri 2023

Exercice 21. Solution


Existe-t-il des entiers a, b, c tels que :
a) a2 + b2 = 100100 + 7 ?
b) a2 + b2 + c2 = 100100 + 7 ?

Exercice 22. Solution


Montrer sans calcul que 87457561565 ne divise pas 100000000000000000000000007

Exercice 23. Solution


Montrer que pour tout entier n, 9n − 2n est divisible par 7.

Exercice 24. Solution


Trouver tous les entiers n tels que 7 | n2 + n + 1

Exercice 25. Solution


Soient a et b deux entiers premiers entre eux. Montrer que ab et a + b sont premiers entre eux.

Exercice 26. Solution


Montrer que le produit de k entiers consecutifs est divisible par k!

Exercice 27. Solution


Déterminer φ(20) où φ(n) est la fonction indicatrice d’Euler ie le nombres d’entiers premiers avec
n compris entre 1 et n.

Exercice 28. Solution


Montrer que pour tout entier n et pour tout k entre 1 et n,
     
n n−1 n−1
= +
k k−1 k
Source : Formule d’Euler

Exercice 29. Solution


Demontrer la formule du binome de Newton : si n est un entier,
n  
n
X n k n−k
(a + b) = a b
k
k=0

Source : Binome de Newton

4
1.1.2 Algèbre

Exercice 30. Solution


Soit x un nombre réel different de 1. Montrer par recurrence l’égalité suivante :
1 − xn+1
1 + x + x2 + x3 + . . . + xn =
1−x
Exercice 31. Solution
Montrer que ∀x ∈ R, x2 ≥ 0. Trouver le cas d’égalité
En déduire que ∀a, b ∈ R, a2 + b2 ≥ 2ab.
√ Trouver le cas d’égalité.
En déduire que ∀c, d ∈ R+ , c + d ≥ 2 cd. Trouver le cas d’égalité.

Exercice 32. Solution


Soit (un )n∈N une suite de réels définie par :

u0 = a
un+1 = un + q pour n > 0
Montrer que ∀n ∈ N, un = a + nq
Source : Terme général d’une suite arithmétique

Exercice 33. Solution


Soit (un )n∈N une suite de réels définie par :

u0 = a
un+1 = un × q pour n > 0
Montrer que ∀n ∈ N, un = a × q n
Source : terme général d’une suite géométrique

Exercice 34. Solution


Déterminer si les suites suivantes sont croissantes ou décroissantes
1) u0 = 0, un+1 = un + 1
2) un = n2 + 5n + 4
3) un+1 = u2n − un + 1

Exercice 35. Solution


Soit (xn )n∈N une suite de réels définie par :

x0 = 1
xn+1 = xn + xn−1 + . . . + x1 + x0 + 1 pour n > 0
x106
Déterminer
x102

Exercice 36. Solution


Montrer que pour tous a, b > 0, a3 + b3 + a + b ≥ 4ab

Exercice 37. Solution


La fonction f (x) = x vérifie-t-elle l’équation :
f (f (x + y) + x2 + y 2 ) + f (2xy) = f (x + y) + f (x + y)2 pour tout x, y ∈ R ?

Exercice 38. Solution


La fonction f (x) = x2 est-elle solution de :
2 √
f (f (x) + y2 ) + f (y) = x4 + 5y4 + f (x y) pour tout x, y ∈ R ?

Exercice 39. Solution


Démontrer que, si f : R −→ R est une fonction strictement monotone, alors elle est injective.

Exercice 40. Solution


Soient a, b et c des nombres reels avec a ̸= 0. Montrer que l’equation ax2 + bx + c admet des
solutions reelles SSI b2 − 4ac ≥ 0

5
Exercice 41. Solution
Soient P et Q des polynomes a coefficients dans R (On notera a l’avenir P, Q ∈ R[X]). Montrer
que P + Q et P Q sont des polynomes a coefficients dans R

Exercice 42. Solution


Soient P, Q ∈ R[X]. Montrer que deg(P +Q) ≤ max(deg P, deg Q) et que deg(P Q) = deg P +deg Q
NB : Par convention, on supposera que pour tout entier α, −∞ + α = −∞ pour que ces resultats
restent valides pour le polynome nul.

Exercice 43. Solution


Pour chacun des exemples suivants, determiner si ce sont des polynomes, et si c’est le cas, leur
degré.
2
⋆ A(X) = X
√ + 3X + 7
⋆ B(X) = √X
⋆ C(X) = √2 − 2X 19√+ πX 42
⋆ D(X) = ( X − 2)( X + 2)

Exercice 44. Solution


Déterminer les degrés et les coefficients dominants des polynomes suivants :
A = X 7 + 3X + 1
B = (X + 7)(X − π)(X 5 + 3X − 12 )
C = (X 2 + 3)(X 3 + 2X − π)(3X − 12 ) + 3X 5 − 2X

Exercice 45. Solution


Pour chacun des nombres suivants, déterminer la partie réelle, la partie imaginaire, le module et
le conjugué :
• z1 = 3 + 4i√
• z2 = 1 + 15i
• z3 = (1 + i)2

Exercice 46. Solution


Soit z = a + ib déterminer l’inverse de z en fonction de a et b

Exercice 47. Solution


Soit z ∈ C. Déterminer une expression du module, de la partie réelle et de la partie imaginaire de
z en fonction de z et z

Exercice 48. Solution


Soit x un réel, montrer que
2x − x2 ≤ 5

Exercice 49. Solution


Soient a, b, c ∈ R+ . Montrer que
a2 + b2 + c2 > 2(bc + ac − ab)

Exercice 50. Solution


Soient a, c, b ∈ R tels que a ≥ b ≥ c ≥ −1. Montrer que
abc + ab + ac + bc + a + b + c + 1 ≥ 0

Exercice 51. Solution


Soient a, b, c, d > 0. Montrer que
a b c d
1< + + + <2
a+b+d b+c+a b+c+d a+c+d
Le problème original (nettement plus dur) demandait quelles valeurs cette expression peut prendre.
Source : IMO

Exercice 52. Solution


Montrer que pour tous a, b, c, d ∈ R, a(a + b) + c2 ≥ b(3a − b) + d(2c − d)

Exercice 53. Solution


Trouver toutes les solutions de l’equation

z 3 = 4 2(1 + i)

6
1.1.3 Combinatoire

Exercice 54. Solution


Combien d’enfants faut-il au minimum dans une école pour que l’on soit sûr que 3 d’entre eux au
moins aient leur anniversaire le même jour ? (Rappelons que certaines personnes naissent un 29
février.)

Exercice 55. Solution


Soient x1 , x2 , · · · , x2022 des entiers. Montrer qu’il existe i ̸= j tels que xi − xj soit divisible par
2021.

Exercice 56. Solution


Montrer que si 7 nombres distincts sont choisis dans l’ensemble {1, 2, 3, 4, 5, 6, 7, 8, 9, 10, 11, 12}
alors, il en existe 2 dont la somme vaut 13.

Exercice 57. Solution


Soit Ω un ensemble non vide de points du plan tels que chacun de ces points soit le milieu de deux
autres. Montrer que Ω possede une infinité d’éléments.

Exercice 58. Solution


On dispose de deux sortes de bonbons, chacune en grande quantité. De combien de manières
peut-on donner un bonbon à chaque élève d’une classe de 20 étudiants ?

Exercice 59. Solution


De combien de manières peut-on choisir 3 entiers distincts compris entre 1 et 100, tels que l’un
d’eux est la moyenne des 2 autres ?

Exercice 60. Solution


On considère un échiquier 8 × 8, combien y a t’il de façons de placer 8 tours tel que deux tours ne
soient jamais sur la même ligne ou la même colonne ? 6 tours ?

Exercice 61. Solution


  
n n
Montrer que =
k n−k

Exercice 62. Solution


Monsieur Deschamps possède des poules et des vaches ; les poules ayant toutes deux pattes, et
les vaches quatre. En prévision de l’hiver, il doit leur confectionner des pantoufles. Il possède 160
animaux en tout, et il a dû confectionner 400 pantoufles. Combien possède-t-il de vaches ?
Source : Coupe Animath de Printemps 2019

Exercice 63. Solution


Est-il possible de paver une grille de taille 4 × 4 avec des pièces de taille 1 × 3 ? Et avec une grille
de taille 2n × 2n pour n ≥ 1 un entier ?

Exercice 64. Solution


Est-il possible de paver une grille de taille 3 × 2n avec n ≥ 1 un entier avec des pièces de la forme
(les rotations de la pièce sont autorisées) :

Exercice 65. Solution


Prouver que l’on peut paver une grille de taille 4 × 4n avec des pièces de la forme (les rotations de
la pièce sont autorisées) :

7
Exercice 66. Solution
On dispose de piles de jetons. On peut séparer une pile pour former deux piles ou fusionner deux
piles pour en former une. Peut-on, en débutant avec une pile de 2022, obtenir 2023 piles de 1 jeton ?

Exercice 67. Solution


On dispose de 16 ampoules dans un carré de 4 par 4. On dispose d’un interrupteur par ligne
d’un interrupteur par colonne. Chaque interrupteur inverse l’état (allumé ou éteint) de toutes les
ampoules de sa ligne ou colonne. Au départ, seule l’ampoule en haut à gauche est allumée. Peut-on
allumer toutes les ampoules ?

Exercice 68. Solution


On dispose d’une pile de 100 pièces. De là, il est possible d’effectuer 2 types d’opérations :
-Enlever une pièce d’un tas d’au moins 3 pièces et diviser le tas restant en deux tas (non vides),
-Supprimer un tas d’une seule pièce.
En trouvant un invariant sur le nombre de pièce additionné au nombre de tas, prouver qu’il est
impossible d’arriver dans la situation où l’on a aucune pièce.

Exercice 69. Solution


On écrit les nombres de 1 à 2021 au tableau. A chaque étape, on efface deux nombres au tableau
et on écrit leur différence. Peut-on finir avec comme seul chiffre écrit au tableau le 2 ?

Exercice 70. Solution


Alice et Bob jouent à un jeu sur un échiquier 8 × 8 : à tour de role, en commençant par Alice, ils
placent des dominos sur deux cases libres de l’échiquier. Le premier joueur qui ne peut plus placer
de domino sans chevaucher un domino existant a perdu. Lequel des deux joueurs dispose d’une
stratégie gagnante ?
On pourra s’inspirer des stratégies "miroir" vu précédement.

Exercice 71. Solution


On place 5 points dans un triangle équilatéral de coté 1. Montrer que deux sont à distance moins
de 0.5.

Exercice 72. Solution


Le plan est colorié en rouge et bleu, prouver qu’il existe deux points de la même couleur à distance
x, quel que soit x un réel positif.

Exercice 73. Solution


Au club de maths, 6 matheux sont présents, certains se connaissant (le relation se connaître étant
réciproque). Montrer que parmis les 6 matheux présent, il y a au moins 3 matheux se connaissant
ou 3 matheux ne se connaissant pas. Est-ce encore le cas avec 5 matheux ?

Exercice 74. Solution


Soit 21 nombres entre 1 et 40 Montrer qu’il en existe 2 premiers entre eux.

Exercice 75. Solution


Sept amis fêtent leur anniversaire le même jour et ont respectivement les âges a1 , . . . , a7 . Ils dis-
posent en tout de a1 + . . . + a7 − 6 bougies qu’il se répartissent. Chacun aimerait avoir au moins
autant de bougies que son âge. Montrer que quelle que soit la manière dont les bougies sont répar-
ties, un des amis a assez de bougies pour fêter son anniversaire.

Exercice 76. Solution


Combien un n-gone possède-t-il de diagonales (ses côtés n’étant pas considérés comme des diago-
nales).

Exercice 77. Solution


On donne 2n points dans le plan, trois quelconque jamais alignés. Démontrer que l’on peut
construire n segments qui ne s’intersectent pas (même aux sommets) et dont les sommets sont
les points du plan.
Et avec 4n points en formant des quadrilatères ?

Exercice 78. Solution


Soient n droites en position générale. Determiner le nombre de points d’intersection sur le plan.
On dira que des droites sont en position générale lorsque deux droites ne sont jamais parallèles et
que trois droites ne sont jamais concourantes.

8
1.1.4 Géométrie

Exercice 79. Solution


Soit ABC un triangle rectangle en A. Soit M le milieu de BC.
Montrer que M A = M B = M C.

Exercice 80. Solution


Soit ABCD un trapèze régulier tel que AB//CD et BAD \ = ABC.\
Montrer que ABCD est inscriptible (ie ses points sont cocycliques).

Exercice 81. Solution


Soit Ω un cercle et P un point exterieur à Ω. Soient A et B les points de tangence a Ω issus de P .
Montrer que AP = BP .

Exercice 82. Solution


Soient d et d′ deux droites parallèles Soit A un point du plan ni sur d ni sur d′ . Soient B et C
deux points distincts de d. Soient enfin B ′ et C ′ les points d’intersection de (AB) et (AC) avec d′
respectivement.
Montrer que
AB AC BC
= = ′ ′
AB ′ AC ′ BC
Source : Théorème de Thalès

Exercice 83. Solution


Soit Γ un cercle de centre O et A, B, C quatre points sur le cercle. Montrer que BOC
\ = 2BAC.
\
La réciproque est vraie.
Source : Théorème de l’angle au centre

Exercice 84. Solution


Soit Γ un cercle et A, A′ , B, C quatre points sur le cercle. Montrer que BA
\ \ La
′ C = BAC.

réciproque est vraie.


Source : Théorème de l’angle inscrit

Exercice 85. Solution


Soit ABCD un quadrilatère cyclique (ie. inscrit dans un cercle). Montrer que BAC
\ = 180 − BDC
\

Exercice 86. Solution


Soit ABC un triangle et D le milieu du cote [AB]. Supposons que CAB
\ = 20 et CDB
\ = 40.
Déterminer l’angle CBA
\
Source : Coupe Animath de printemps 2023

Exercice 87. Solution


Soit Γ un cercle, P un point a l’exterieur de Γ. Soient (d) et (d′ ) deux droites passant par P
et coupant Γ en A, A′ et B, B ′ respectivement. Montrer que P A × P A′ = P B × P B ′ Source :
Puissance d’un point par rapport à un cercle

Exercice 88. Solution


Soit ABC un triangle rectagnle en C et H le pied de la hauteur issue de C.
Montrer les relations suivantes :
AH × AB = AC 2
BH × BA = BC 2
AH × BH = CH 2
Source : lemme d’Euclide

Exercice 89. Solution


Soit ABC un triangle. Notons ω son cercle circonscrit. Soit P le point d’intersection entre (AC)
et la tangente à ω passant par B. Soient B ′ , C ′ les symétriques de B et C par rapport à P
Montrer que les points A, B, B ′ , C ′ sont cocycliques
Indication : penser a utiliser le théorème de l’angle a la tangente

9
Exercice 90. Solution
On considère une rivière droite de largeur L et deux points A et B de part et d’autre de cette
rivière. On veut construire un pont perpendiculaire à la rivière. Où le construire pour que le trajet
de A à B soit le plus court possible ?

Exercice 91. Solution


On considère une droite d et deux point B et C du meme cote de D. B est la position d’un berger,
C celle de sa bergerie et d une rivière. En quel point de la rivière le berger doit-il s’arreter pour
que son trajet total pour rentrer à la bergerie soit minimal ?

Exercice 92. Solution


Etant donnés deux points A et C et un cercle Γ, construire deux points B et D sur Γ tels que
ABCD soit un parallélogramme

Exercice 93. Solution


On se donne un cercle Γ de rayon r, une droite (d) et une longueur a ≤ 2r. Construire une droite
parallèle à (d) qui coupe Γ en X et Y telle que XY = a

10
1.2 Entrée

Ces exercices sont un peu plus techniques, nécessitant des manipulations algébriques, un peu
d’initiative et parfois de l’intuition ou des résultats intermédiaires. Ils restent cependant très ac-
cessibles.

1.2.1 Arithmétique

Exercice 94. Solution


Soit n ∈ N. Montrer par un raisonnement direct que si n est pair, alors n2 + 2n est un multiple de
4

Exercice 95. Solution √


Montrer par l’absurde que 2 est irrationnel

Exercice 96. Solution


Soit n ∈ N. Montrer par récurrence que :
n(n + 1)(2n + 1)
a) 12 + 22 + 32 + . . . + n2 =
6
b) 13 + 23 + 33 + . . . + n3 = (1 + 2 + 3 + . . . + n)2

Exercice 97. Solution


Soient a, b ∈ R deux réels. On considère la proposition suivante : "si a + b est irrationnel, alors a
ou b est irrationnel."
a) Quelle est la contraposée de cette proposition ?
b) Démontrer la proposition.
c) Est-ce que la réciproque de cette proposition est toujours vraie ?

Exercice 98. Solution


Soit n ∈ N. Montrer par recurrence que pour tout entier naturel n, 7 | 8n − 1

Exercice 99. Solution


a) Montrer que si il existe n ∈ N tel que an2 + bn + c = 0, alors n | c
b) Déterminer les entiers n ∈ N tels que n5 − 2n4 − 7n2 − 7n + 3 = 0.

Exercice 100. Solution


Montrer que si a et b sont des entiers impairs, alors a2 + b2 n’est pas un carré parfait.

Exercice 101. Solution


Montrer que si on a n = pα 1 × . . . × pk la décompsition en produit de facteurs premiers de n, alors
1 αk

n possède (α1 + 1) × . . . × (αk + 1) diviseurs.

Exercice 102. Solution


Trouver toutes les solutions entières de l’équation

n4 + 7n3 − 7n2 − 43n + 42 = 0

Exercice 103. Solution


Montrer que 13 divise 270 + 370

Exercice 104. Solution


Trouver tous les entiers a et b tels que a2 + b2 = 10n + 3

Exercice 105. Solution


Déterminer toutes les solutions de l’équation 9u − 7v = 1

Exercice 106. Solution


Soit n ∈ N tel que 2n + 1 et 3n + 1 soient des carrés. Montrer que 40 | n

11
Exercice 107. Solution
Démontrer le critère de divisibilité par 2, par 3, par 5, par 9, par 10 et par 11

Exercice 108. Solution


9
Trouver le dernier chiffre de l’ecriture decimale de 73

Exercice 109. Solution


Trouver tous les inversibles modulo 8

Exercice 110. Solution


Montrer que si (p − 1)! ≡ −1 (mod p), alors p est premier.
Source : Théorème de Wilson

Exercice 111. Solution


Soit n un entier et a un entier premier avec n. Montrer qu’il existe un entier b tel que ab ≡ 1
(mod n).
Source : Existence d’un inverse modulo n

Exercice 112. Solution


Soit n un entier et a un entier premier avec n. Montrer que l’inverse de a modulo n est unique
modulo n.
Source : Unicité d’un inverse modulo n

Exercice 113. Solution


Soit a un entier naturel. Determiner toutes les solutions modulo 15 des équations suivantes :

2a ≡ 7 (mod 15)

3a ≡ 8 (mod 15)

Exercice 114. Solution


Soit p un entier. Montrer que p est premier SSI

(p − 1)! ≡ −1 (mod p)

Source : Théorème de Wilson

Exercice 115. Solution


Existe-t-il un entier dont l’écriture décimale contienne exactement 300 chiffres « 1 », aucun autre
chiffre différent de « 0 », et qui soit un carré parfait ?
Source : Coupe Animath d’automne 2019

12
1.2.2 Algèbre

Exercice 116. Solution


Montrer que ∀x ∈ R∗+ , x + 1
x ≥ 2 et trouver les cas d’égalité

Exercice 117. Solution


y2
Montrer que ∀x, y ∈ R∗+ , x + x ≥ 2y. Trouver les cas d’égalité.

Exercice 118. Solution


Montrer que 5x2 + y 2 + 1 ≥ 4xy + 2x

Exercice 119. Solution


Soit x ∈ R+ . Montrer que 1 + x2 + x6 + x8 ≥ 4x4

Exercice 120. Solution


Soient a, b, c des reels positifs tels que (a + 1)(b + 1)(c + 1) = 8. Montrer que a + b + c ≥ 3

Exercice 121. Solution


Soit Fn le n-ieme terme de la suite de Fibonacci. Montrer que pour tout n ∈ N
√ √
( 1+2 5 )n − ( 1−2 5 )n
Fn = √
5
Pour rappel, la suite de Fibonacci est l’unique suite définie par :

 F0 = 0
F1 = 1
Fn+1 = Fn + Fn−1

Source : Formule de Binet

Exercice 122. Solution


Soit (un )n∈N une suite périodique de période 2022 et de période 5. Montrer que (un ) est constante.
Une suite (un ) est dite périodique de période t si, pour tout entier naturel n, un+t = un .
Source : Envoi d’algèbre 2023

Exercice 123. Solution


Soit Fn le n-ieme terme de la suite de Fibonacci. Montrer que pour tout n ∈ N, Fn+1 ×Fn−1 −Fn2 =
(−1)n+1

Exercice 124. Solution


Soit (un )n∈N une suite de réels définie par :

u0 = 0
un+1 = a × un + b

Déterminer le terme général de la suite.


b
Indication : On pourra poser vn = un −
1−a
Source : Terme général d’une suite arithmético-géométrique

Exercice 125. Solution


Déterminer toutes les fonctions f : R −→ R à pente constante, c’est-à-dire pour lesquelles il existe
un réel a tel que, pour tout x et tout y distinct de x, on ait :

f (x) − f (y)
=a
x−y

Exercice 126. Solution


Déterminer toutes les fonctions f : R −→ R tels que f (x − f (y)) = 1 − x − y
Source : Envoi d’algèbre 2021

13
Exercice 127. Solution
Déterminer toutes les fonctions strictement croissante puis croissant f : R −→ R tels que f (f (x)) =
x pour tout x ∈ R.

Exercice 128. Solution


Déterminer toutes les fonctions f : R −→ R telles que, pout tout x et tout y, on ait :
f (x + y) = f (x) + y

Exercice 129. Solution


Soit P ∈ R[X]. Soit α ∈ R tel que P (α) = 0. Montrer que il existe Q ∈ R[X] tel que ∀x ∈ R,
P (x) = (x − α)Q(x). En deduire qu’un polynome de degre n ∈ N possede au plus n racines.

Exercice 130. Solution


Soit P un polynome a coefficients dans R de la forme P (x) = a0 + a1 x + . . . + an xn . Montrer que
si ∀x ∈ R, P (x) = 0, alors a0 = a1 = . . . = an = 0.
En déduire que si P (X) = a0 + a1 X + a2 x2 + . . . + an X n et Q(X) = b0 + b1 X + . . . + am X m
sont deux polynomes vérifiant ∀x ∈ R, P (x) = Q(x), alors n = m et ∀k ∈ [[0, n]], ak = bk . Ces
hypothèses peuvent-elles etre ameillorées ?

Exercice 131. Solution


Determiner tous les polynomes verifiant chacune des egalites suivantes :
⋆P 2 = XQ2
⋆P (X 2 ) = X 2 P (X)

Exercice 132. Solution


Determiner tous les polynomes P verifiant ∀x ∈ R, P (x) = P (2x)

Exercice 133. Solution


Determiner, pour chacun des complexes la forme exponentielle s’il est sous forme algébrique ou la
forme algébrique s’il est sous forme exponentielle :
π
• z1 = 2ei 4
• z2 = 4 + 3i
• z3 = i
• z4 = eiπ

Exercice 134. Solution


Déterminer la partie réelle de (1 + i)20

Exercice 135. Solution


Soit z ∈ C. Déterminer une racine carrée de z, i.e. un nombre complexe y tel que y 2 = z. On en
donnera une forme exponentielle plus une forme algebrique.

Exercice 136. Solution


Soient a, b, c ∈ R+ , montrer que
(x + y)(y + z)(x + z) ≥ 8xyz

Exercice 137. Solution


Soit x > 0. Montrer que  
1
(1 + x) 1 + ≥4
x
Déterminer le cas d’égalité.

Exercice 138. Solution


Soient x, y > 0. Montrer que
1 x y
≥ 4 2
+ 4
xy x +y y + x2
Exercice 139. Solution
Soient a, b, c, d > 0. Montrer que
a3 + b3 + c3 + d3 ≥ abc + acd + abd + bcd

Exercice 140. Solution


Determiner tous les points du plan dont l’affixe z verifie z + z = |z|

14
1.2.3 Combinatoire

Exercice 141. Solution


A chaque point à coordonnées entières du plan, on attribue un nombre entier strictement positif,
tel que chaque nombre est égal à la moyenne arithmétique de ses quatre voisins (en haut, en bas,
à gauche et à droite). Montrer que toutes les valeurs sont égales.

Exercice 142. Solution


On choisit 5 points distincts a coordonnées entières dans le plan. Montrer qu’il est toujours possible
d’en choisir 2 tels que le segement les joignants passe a travers un autre point du réseau

Exercice 143. Solution


   
n n−1
Montrer que k × =n×
k k−1

Exercice 144. Solution


Combien y a-t-il de façons d’aller du point (0, 0) au point (6, 8) en faisant des pas +(1, 0) ou +(0, 1)
à chaque tours ?

Exercice 145. Solution


De combien de manières peut-on placer 5 pièces identiques dans 3 sacs différents ?

Exercice 146. Solution


Montrer que :
n  
X n
= 2n
k
k=0

Exercice 147. Solution


De combien de manières peut-on paver une grille de taille 2 × 3n avec des pièces de la forme (les
rotations de la pièce sont autorisées) ?

Exercice 148. Solution


De combien de manières peut-on paver une grille de taille 2 × n avec des dominos de taille 1 × 2
(les rotations de la pièce sont autorisées) ? (Une suite entièrement déterminée est accepté comme
réponse)

Exercice 149. Solution


Les nombres a1 , a2 , . . . , an valent 1 ou −1. On pose an+1 = a1 , an+2 = a2 et an+3 = a3 . Sachant
que S = a1 a2 a3 a4 + a2 a3 a4 a5 + . . . + an an+1 an+2 an+3 vaut 0, montrer que n est divisible par 4.
On pourra étudier ce qu’il se passe pour S lorsque l’on change le signe d’un des nombres.

Exercice 150. Solution


Dans un certain pays, deux villes sont toujours reliées soit par une ligne aérienne, soit par un canal
navigable. Montrer qu’un des deux moyens de transport permet de voyager de n’importe quelle
ville à n’importe quelle autre.
On pourra raisonner par récurrence.

Exercice 151. Solution


Une salle est faite de 7 rangées de 10 places. Une classe de 50 élève a cours dans la salle le matin et
l’après-midi. Montrer qu’il existe 2 élèves qui ont été sur la même rangée le matin et l’après-midi.

Exercice 152. Solution


Sur une grille 2018 × 2018, on peut bouger un jeton d’une case vers une autre si ces deux cases ont
un côté commun. Est-il possible, en partant avec le jeton dans le coin inférieur gauche, d’amener
le jeton dans le coin supérieur droit en passant une et une seule fois par toutes les cases ?

15
Exercice 153. Solution
On considère n points dans le plan, trois quelconques jamais alignés (on dit qu’ils sont en position
générale), tels que quatre quelconques soient toujours les sommets d’un quadrilatère convexe.
Prouver que ces n points sont les sommets d’un n-gone convexe.

Exercice 154. Solution


Soit P un polygone convexe à n côtés. Montrer qu’il existe un ensemble S de n − 2 points tels que
tout triangle dont les sommets sont des sommets de P contient exactement un point de S.

Exercice 155. Solution


Montrer que
n  
X n
k = n2n−1
k
k=0

Exercice 156. Solution


Soient x1 , . . . , xn , y1 , . . . , yn des reels tels que min xi ≥ min yj . On pose P = max (xi − yi ) et
1≤i≤n 1≤j≤n 1≤i≤n
G = max xi − min yj . Montrer que :
1≤i≤n 1≤j≤n

P ≤ G ≤ 2P

Source : Coupe Animath de printemps 2020

16
1.2.4 Géométrie

Exercice 157. Solution


Soit ω un cercle de centre O, soit A, B deux points de ω. Soit T l’intersection des tangentes à ω
passant par A et B.
a) Montrer que T A = T B
b) Montrer que les points O,A,T et B sont cocycliques.

Exercice 158. Solution


Soient Γ1 , Γ2 deux cercles de centre O1 et O2 . Soient A, B leurs deux points d’intersection.
Montrer que (AB) ⊥ (O1 O2 )

Exercice 159. Solution


Soit ABC un triangle, soit H son orthocentre (c’est à dire le point d’intersection de ses hauteurs).
Notons D le pied de la hauteur issue de A, E celui de la hauteur issue de B, F celui de la hauteur
issue de C.
a) Montrer que les points A, B, D, E sont cocycliques
b) Montrer que les points A, E, H, F sont cocycliques

Exercice 160. Solution


Soient Γ1 , Γ2 deux cercles. Soient A, B leurs points d’intersection. Soit d1 une droite passant par
A, soit d2 une droite passant par B.
Notons C (resp. D) le point d’intersection autre que A de d1 avec Γ1 (resp. Γ2 ). Notons également
E (resp. F ) le point d’intersection autre que B de d2 avec Γ1 (resp. Γ2 )
Montrer que (CE)//(DF )

Exercice 161. Solution


Soit ABCD un quadrilatère circonscriptible non-croisé. Montrer que AB + CD = BC + AD.

Exercice 162. Solution


Soit ABC un triangle, on note D l’intersection des tangentes au cercles circonscrit de ABC en B
et C. Si ABC
\ = 42 et BCA
\ = 63, que vaut BDC \?

Exercice 163. Solution


Soient C1 et C2 deux cercles de centres respectifs O1 et O2 s’intersectant en X et Y . Soit A un
point de C1 et B l’intersection de (AY ) et de C2 . Montrer que XO1 O2 et XAB sont semblables.
Source : Premier théorème de Miquel

Exercice 164. Solution


Soit ABC un triangle. Soient P , Q et R des points appartenant respectivement aux segments
[BC], [CA] et [AB]. Montrer que les cercles circonscrits aux triangles AQR, P RB et CQP sont
concourants en un point.
Source : Second théorème de Miquel

Exercice 165. Solution


Montrer que dans un triangle, les bissectrices sont concourantes et en déduire que tout triangle
possède un cercle inscrit. Montrer ensuite que les médiatrices sont concourantes et en déduire que
tout triangle possède un cercle circonscrit.

Exercice 166. Solution


Montrer que dans un triangle, les hauteurs sont concourantes

Exercice 167. Solution


Soit Γ un cercle de centre O, P un point a l’exterieur de Γ. Soit (d) une droite passant par P et
coupant Γ en A et A′ . Soit également (d′ ) une droite passant par P tangente a Γ en T . Montrer
que P A × P A′ = P T 2 = OA2 − OT 2

17
Exercice 168. Solution
a) Montrer que l’image d’une droite par une homothetie est une droite parallele a la premiere.
b) Montrer qu’une homothetie de rapport k multiple toutes les longueurs par |k| : les homotheties
conservent les rapports de longueur.
c) Montrer que les homotheties conservent les angles, les droites, les cercles.

Exercice 169. Solution


Soient h1 et h2 deux homotheties de centres respectifs O1 et O2 et de rapports k1 et k2 . On
admttera que la composee de deux homotheties est une homothetie.
a) Montrer que si k1 k2 est different de 1, alors h2 ◦ h1 est une homothetie de rapport k1 k2 et de
centre sur (O1 O2 ).
b) Montrer que si k1 k2 = 1, alors h2 ◦ h1 est une homothetie de vecteur parallele a (O1 O2 ).

Exercice 170. Solution


Soient A, B, A′ et B ′ quatre points. Montrer qu’il exsite une homothetie ou translation qui envoie
A sur A′ et B sur B ′ SSI (AB) ∥ (A′ B ′ ). De plus, dans ce cas elle est unique.

Exercice 171. Solution


Montrer que les médianes d’un triangle sont concourantes et montrer que ce point d’intersection
se trouve aux deux tiers des médianes.

18
1.3 Plat

Exercices plus difficiles, certains sont issus de compétitions internationales, ils nécessitent gé-
néralement de l’intuition et peuvent prendre beaucoup plus de temps. Il doivent rester accessibles
à tout élève ayant très bien assimilé les notions présentées.

1.3.1 Arithmétique

Exercice 172. Solution


1
Soit a ∈ R tel que a + a1 ∈ Z. Montrer que pour tout n ∈ N, an + n ∈ Z
a
Exercice 173. Solution
Soit n ∈ N tel que n ̸= 0. Montrer que si n est le carré d’un entier, alors 2n n’est pas le carré d’un
entier.

Exercice 174. Solution


Montrer qu’il existe une infinité de nombres premiers.
Source : Euclide

Exercice 175. Solution


a) Montrer que a4 + 4b4 = (a2 + 2b2 + 2ab)(a2 + 2b2 − 2ab)
b) Le nombre 4545 + 5454 est-il premier ?
c) Montrer que si n est un entier strictement superieur a 1, alors n4 + 4n n’est pas premier.
Source : Factorisation de Sophie-Germain

Exercice 176. Solution


Soit n ∈ N∗
a) Montrer que si 2n − 1 est premier, alors n est premier
b) Montrer que si 2n + 1 est premier, alors n est une puissance de 2

Exercice 177. Solution


Montrer qu’il existe une infinité de nombres premiers de la forme 4k + 3, k ∈ Z.

Exercice 178. Solution


Trouver toutes les solutions de 3m − 2n = 1

Exercice 179. Solution


Soit φ(n) = |{k ∈ N | pgcd(k, n) = 1}| le nombre de nombres premiers avec n compris entre 1 et
n. Montrer que pour tout a premier avec n, aφ(n) ≡ 1 (mod n)
Source : Théorème d’Euler-Fermat

Exercice 180. Solution


Montrer que si p et p2 + 8 sont premiers, alors p3 + 8p + 2 l’est aussi.

Exercice 181. Solution


Notons A la somme des chiffres de l’écriture décimale de 44444444 , B la somme des chiffres de
l’écriture décimale de A, et C la somme des chiffres de l’écriture décimale de B. Déterminer C.
Source : IMO 1975

Exercice 182. Solution


Déterminer toutes les solutions de l’équation x2 + y 4 + 1 = 6z avec x, y, z des entiers.

Exercice 183. Solution


Déterminer toutes les solutions entières de 2n − 3m = 7.
Source : Valbonne 2020, entrainement de mi-parcours, groupe B

19
Exercice 184. Solution
Trouver toutes les solutions entieres de

1 + 2! + . . . + n! = mk

avec k > 1

Exercice 185. Solution


Soit p un nombre premier  
p
a) Montrer que ∀k ∈ [[1, p − 1]], p |
k
b) Montrer que pour tous entier a, b, (a + b)p ≡ ap + bp (mod p)
c) Démontrer le petit théorème de Fermat : ap−1 ≡ 1 (mod p)
Source : Petit théorème de Fermat

Exercice 186. Solution


Trouver tous les n, m ∈ N tels que n3 − m3 = 24

Exercice 187. Solution


71
Déterminer le reste de la division euclidienne de 270 par 13

Exercice 188. Solution


Trouver tous les nombres premiers p tels que p | 29p + 1

Exercice 189. Solution


Montrer que 13 divise 270 + 370 .

Exercice 190. Solution


Determiner φ(100)
En déduire les deux derniers chiffres de l’écriture décimale de 20062006

Exercice 191. Solution


Soit n un entier naturel, soit a premier avec n. Montrer que an! ≡ 1 (mod n)

20
1.3.2 Algèbre

Exercice 192. Solution


Montrer que a2 + b2 + c2 ≥ ab + ac + bc
Source : Lemme du tourniquet

Exercice 193. Solution


Soient a1 , . . . , an des reels positifs tels que a1 × . . . × an = 1, (a1 + 1) . . . (an + 1) ≥ 2n

Exercice 194. Solution


Soient a, b, c ∈ R+ Montrer que a2 (b + c) + b2 (c + a) + c2 (a + b) ≥ 6abc

Exercice 195. Solution


Soient a, b, c ∈ R+ tels que abc = 81 . Montrer que a2 + b2 + c2 + a2 b2 + a2 c2 + b2 c2 ≥ 15
16

Exercice 196. Solution


Soient a1 , . . . , an ∈ R∗+ tels que a1 + . . . + an = n, montrer que ( a11 + 1) × . . . × ( a1n + 1) ≥ 2n

Exercice 197. Solution √


Soit a > 0. Considérons la suite (un ) définie par u0 ≥ a et pour tout n ∈ N :
 
1 a
un+1 = un +
2 un

un − a √ n
Soit vn = √ . Montrer que |un − a| ≤ 2u0 v02
un + a

Exercice 198. Solution


La suite (ai )i∈N satisfait am+n + am−n = 12 (a2m + a2n ) pour tous m, n ∈ N avec m ≥ n. Si a1 = 1,
déterminer a2023 .

Exercice 199. Solution


Déterminer toutes les fonctions f : Q −→ Q telles que, pour tout n et tout m, on ait :

f (n + m) = f (n) + f (m)

(on pourra commencer à chercher les fonctions dans N puis dans Z avant de passer à Q)
Source : équation de Cauchy sur Q

Exercice 200. Solution


Soient a0 , . . . , an , b0 , . . . , bn des reels. Montrer qu’il exsite un unique polynome P ∈ R[X] tel que
deg P ≤ n et ∀k ∈ [[0, n]], P (ak ) = bk
Source : Polynome d’Interpolation de Lagrange

Exercice 201. Solution


Soit P un polynome de degre 4 verifiant P (0) = P (1) = 1, P (2) = 4, P (3) = 9 et P (4) = 16.
Determiner P (−2)

Exercice 202. Solution


Determiner tous les polynomes verifiant (X + 4)P (X) = XP (X + 1)

Exercice 203. Solution


Démontrer qu’un polynome de degré n ∈ N possède au plus n racines distintes. En déduire que |x|
n’est pas un polynome.

Exercice 204. Solution


Soit n ∈ N∗ . Déterminer toutes les racines de l’unité i.e. tous les nombres complexes z tels que
zn = 1
On notera l’ensemble de ces complexes Un
Source : Racines de l’unité

21
Exercice 205. Solution
Montrer que :
Um ⊂ Un =⇒ m | n

Exercice 206. Solution


1) Montrer que ∀(u, v) ∈ C2 ,
u v |u − v|
− 2 =
|u|2 |v| |u||v|
2) Soient x, y, z ∈ C∗ , monter que

|x| · |y − z| ≤ |y| · |z − x| + |z||y − x|

3) En déduire l’inegalité de Ptolémée :

∀(x, y, z, w) ∈ C4 , |x − y| · |z − w| ≤ |x − z| · |y − w| + |x − w| · |y − z|

Interpreter géometriquement.
Source : Inégalité de Ptolémée

Exercice 207. Solution


Soient a, b, c > 0 tels que a + b + c ≥ 1
a + 1
b + 1c . Montrer que

3
a+b+c≥
abc
Source : Roumanie 2005

Exercice 208. Solution


Trouver toutes les solutions de l’inégalité
√ √ 1
3−x− x+1>
2
Exercice 209. Solution
Soient a, b, c, d > 0. Montrer que
a+c b+d c+a d+b
+ + + ≥4
a+b b+c c+d d+a

Exercice 210. Solution


Pour tout nombre reel x, on note ⌊x⌋ la partie entiere de x, c’est-a-dire le plus grand entier inferieur
ou egal a x. On note {x} sa partie decimale, c’est-a-dire {x} = x − ⌊x⌋. Trouver tous les nombres
reels x tels que ⌊x⌋ × {x} = 2019x
Source : Coupe animath de printemps 2019

Exercice 211. Solution


Calculer la somme suivante :
n  
X n
k
k=0,k≡0[2]

22
1.3.3 Combinatoire

Exercice 212. Solution


1) Soit n ∈ N Montrer que si on a n tiroirs et n + 1 chaussettes, alors il existe un tiroir contenant
au moins 2 chausettes
2) Soient n, k ∈ N Montrer que si on a n tiroirs et kn + 1 chaussettes, alors il existe un tiroir
contenant au moins k + 1 chausettes
3) Soit n ∈ N Montrer que si on a n tiroirs et un nombre infini de chaussettes, alors il existe un
tiroir contenant une infinité de chausettes.
Source : Principe des tiroirs

Exercice 213. Solution


2009 cartes, ayant chacune un côté bleu et un côté jaune, sont alignées côté bleu sur une table.
Deux personnes situées du même côté de la table jouent alors en alternance. Un coup consiste à
choisir un bloc de 50 cartes dont la carte la plus à gauche est bleue et à retourner toutes les cartes
du bloc. La personne qui ne peut plus jouer perd.
Le jeu se termine-t-il forcément ? Si oui, qui a une stratégie gagnante ?

Exercice 214. Solution


Soit n ≥ 1 : on place dans le plan 2n points, trois quelconques non alignés. On en colorie n en
bleu et n en rouge. Montrer qu’il est possible de tracer n segments qui ne se croisent pas, chaque
segment reliant un point bleu à un point rouge, de telle manière que chaque point soit utilisé une
seule fois.

Exercice 215. Solution


Soit n un entier. On choisit n + 1 nombres parmi {1, 2, · · · , 2n}, montrer que l’on peut en trouver
deux premiers entre eux. Montrer qu’on peut aussi en trouver deux tels que l’un divise l’autre.

Exercice 216. Solution


On choisit 10 entiers deux à deux distincts entre 1 et 100 {x1 , x2 , · · · , x10 }. Montrer qu’on peut
extraire deux sous-ensembles disjoints de même somme.

Exercice 217. Solution


Lors d’une soirée dansante, aucun garçon n’a dansé avec toutes les filles, mais chaque fille a dansé
avec au moins un garçon. Montrer que il existe deux garçons g, g ′ et deux filles f , f ′ tel que g a
dansé avec f mais pas avec f ′ , et g ′ a dansé avec f ′ mais pas avec f .

Exercice 218. Solution


  2  2  2  
n n n 2n
Montrer que + + ... + =
0 1 n n

Exercice 219. Solution


Pour quel n est-il possible de paver une grille carrée de taille 2n × 2n avec le coin en bas à gauche
manquant avec des pièces de la forme (les rotations de la pièce sont autorisées) :

Exercice 220. Solution


Pour quels n ≥ 1 est-il possible de paver une grille de taille n × n avec des pièces de la forme (les
rotations de la pièce sont autorisées) :

23
Exercice 221. Solution
Sur une île vivent des caméléons : 25 rouges, 12 verts et 8 bleus. Quand deux caméléons d’une
même couleur se rencontrent, ils se font peur et prennent chacun une des deux autres couleur mais
pas la même. Quand deux caméléons de couleurs différentes se rencontrent, ils se font peur et
prennent tout deux la troisième couleur. Après un an, tous les caméléons sont de la même couleur.
Cette couleur peut-elle être le bleu ?

Exercice 222. Solution


Les entiers de 1 à n sont écrits dans l’ordre. On échange deux nombres. Un mouvement consiste à
échanger n’importe quels deux entiers. Montrer qu’on ne peut pas retrouver l’ordre initial (crois-
sant) dans un nombre pair de mouvements.

Exercice 223. Solution


Soient 2017 droites dans le plan telles qu’il n’en existe pas trois s’intersectant en un même point.
Turbo l’escargot se trouve sur un point n’appartenant qu’à une seule droite. Il se déplace le long
des droites de la façon suivante. Il se meut sur une droite donnée jusqu’à ce qu’il arrive à une
intersection. À chaque fois qu’il rencontre une intersection, il continue son parcours sur l’autre
droite, tournant à gauche ou à droite, alternant son choix à chaque intersection rencontrée. Aucun
demi-tour n’est permis. Est-il possible qu’il parcoure un même segment de droite dans deux sens
opposés durant son parcours ?
Source : EGMO 2017

24
1.3.4 Géométrie

Exercice 224. Solution


Soit ABC un triangle, soit H son orthocentre. Notons HA le symétrique de H par rapport à BC.
Montrer que HA appartient au cercle circonscrit à ABC.

Exercice 225. Solution


Soient ω et Ω deux deux cercles concentriques (c’est-à-dire qu’ils ont le même centre) de sorte que
le cerlce ω soit à l’intérieur du cercle Ω. Soient X et Y deux points sur le cercle ω. On note P et
Q les points d’intersection respectifs du cercle Ω avec les tangentes en X et Y à ω, de telle sorte
que P et Q soient du même côté par rapport à la droite (XY ). Montrer que les points X, Y , P et
Q sont cocycliques.
Source : Envoi de géométrie 2023

Exercice 226. Solution


Soit ABC un triangle quelconque. Soit S le point d’intersection de la bissectrice de l’angle BAC
\
avec la médiatrice du segment [BC]. Montrer que S appartient au cercle circonscrit au triangle
ABC.
Source : Théorème du Pole Sud

Exercice 227. Solution


Soit ABC un triangle. Montrer que
a b c
= =
sin A sin B sin C
Indication : On pourra montrer que toutes ces quantités sont égales au diamètre du cercle circons-
crit.
Source : Loi des sinus

Exercice 228. Solution


Soit ABC un triangle de cotes a, b, c. Montrer que

c2 = a2 + b2 − 2ab cos(C)

Source : Loi des Cosinus (Théorème d’Al-Kashi)

Exercice 229. Solution


Soient ABC et A′ B ′ C ′ deux triangles tels que (AB) soit parallele a (A′ B ′ ), (BC) a (B ′ C ′ ) et (AC)
a (A′ C ′ ). Montrer que les droite (AA′ ), (BB ′ ) et (CC ′ ) sont concourantes ou paralleles.

Exercice 230. Solution


Soit ABC un triangle avec trois angles aigus.
Construire un carré qui a deux sommets sur [BC], un sur [AB] et un sur [AC]

Exercice 231. Solution


Soit ABC un triangle. On pose a = BC, b = CA et c = AB. Le cercle inscrit à ABC touche
[BC] en X. Le cercle A-exinscrit à ABC touche [BC] en T (rappelons que le cercle A-exinscrit
est le cercle tangent au côté [BC] et aux demi-droites [AB) et [AC) au-delà de B et C). Alors
BX = CT = a+c−b 2 et CX = BT = a+b−c
2

25
1.4 Dessert

Ces exercices sont de deux types : soit ils bien plus compliqués que ceux du plat (ils peuvent etre
issus d’olympiades internationales plus récentes), soit ils présentent des résultats moins importants,
mais qui peuvent demeurer très forts dans de bons contextes. Il faut se souvenir que ces exercices
s’adressent à des élèves de classes de 3e et de 2de, donc certains résultats "classiques" ne le sont
pas (Cauchy, récurrence forte, convexité)

1.4.1 Arithmétique

Exercice 232. Solution


Montrer le principe de récurrence forte : Soit P (n) une proposition dependant de n un entier
naturel. Si on a :
1) P (0) est vraie
2) Si P (0), P (1), P (2), . . . , P (n) sont vraies, alors P (n + 1) est vraie
Alors pour tout n ∈ N, P (n) est vraie.
Source : Principe de récurrence forte

Exercice 233. Solution


Montrer que tout nombre entier superieur ou egal a 2 possede une décomposition en produit de
facteurs premiers.

Exercice 234. Solution


Trouver deux entiers positifs a, b tels que (a + b)7 − a7 − b7 est divisible par 77 mais ab(a + b) n’est
pas divisible par 7.
Source : IMO 1984

Exercice 235. Solution


Montrer qu’il exsite une inifité de nombres premiers de la forme 4k + 1, k ∈ N

Exercice 236. Solution


Soit p un entier. Montrer que l’équation (p − 1)! + 1 = pm n’a pas de solution pour m > 1.

Exercice 237. Solution


Soit p un nombre premier, n un entier strictement positif et q un diviseur strictement positif de
(n + 1)p − np . Montrer que q − 1 est divisible par p.

Exercice 238. Solution


Soit n ≥ 2. Montrer que le produit de trois entiers consécutifs n’est pas une puissance n-ème.

Exercice 239. Solution


Déterminer tous les entiers a, b, c ≥ 0 tels que 2a 3b + 9 = c2

Exercice 240. Solution


Trouver tous les entiers n tels que n | 2n − 1

Exercice 241. Solution


Determiner une formule pour φ(n) en fonction de la decomposition en produit de facteurs premiers
de n.

Exercice 242. Solution


Montrer que pour tout n ∈ N on a : X
φ(d) = n
d|n

26
1.4.2 Algèbre

Exercice 243. Solution


Soient a1 , . . . an , b1 . . . bn ∈ R. Montrer que
n
!2 n
! n
!
X X X
ak bk ≤ a2k b2k
k=1 k=1 k=1
Pn 2
 2 Pn Pn 2

On pourra s’interesser au polynome P (X) = k=1 ak X + (2 k=1 ak bk ) X + k=1 bk , ou
bien developper des membres de l’inegalite.
Source : Inégalité de Cauchy-Schawrz

Exercice 244. Solution


Soient x1 , ..., xn , y1 , ..., yn ∈ R+ . Montrer que

x21 x2 (x1 + . . . + xn )2
+ ... + n ≥
y1 yn y1 + . . . + yn
Source : Inégalité des mauvais élèves

Exercice 245. Solution


Soient a, b, c ∈ R∗+ . Montrer que

a b c 3
+ + ≥
b+c c+a a+b 2
Source : Inégalité de Nesbitt

Exercice 246. Solution


Soient a, b, c les longueurs des côtés d’un triangle. Prouver que

a2 b(a − b) + b2 c(b − c) + c2 a(c − a) ≥ 0

Déterminer lorsque l’égalité a lieu.


Indication : On pourra considerer le cercle inscrit du triangle pour obtenir une relation sur les
longeurs.
Source : IMO 1983

Exercice 247. Solution


Résoudre le système :  2 2 2
x + y + z = 3
y2 z2 x2
x+y+z =3

Exercice 248. Solution


Soit (un ) une suite géométrique. Déterminer a quelle condition (un ) converge et déterminer sa
limite.
Soit (un )n∈N ∈ RN , soit l ∈ R. On dit que (un ) converge vers l si :

∀ϵ > 0, ∃N ∈ N, ∀n ∈ N, n ≥ N =⇒ |un − l| ≤ ϵ

Exercice 249. Solution


La suite d’entiers (xn )n≥1 est définie par x1 = 1, x2 = a, et

xn = (2n + 1)xn−1 − (n2 − 1)xn−2 pour n ≥ 3

où a est un nombre entier positif. Pour quelles valeurs de a a-t-on xi qui divise xj pour tous i ≤ j ?

27
Exercice 250. Solution
Démontrer le résultat suivant :
Soient a, b ∈ C.
Si ∀n ∈ N, un+2 = aun+1 + bun , alors notons P (x) = x2 − ax − b. Soient r1 , r2 les racines (éven-
tuellement complexes) de P . Alors :
1) Si r1 ̸= r2 , alors un = (λ1 r1n + λ2 r2n ), λ1 , λ2 ∈ C
2) Si r1 = r2 = r, alors un = (λ1 + λ2 n)rn , λ1 , λ2 ∈ C
Source : Terme général d’une suite récurrence linéaire d’ordre 2

Exercice 251. Solution


Déterminer toutes les fonctions f : R → R telles que

f (f (x) + 9y) = f (y) + 9x + 24y pour tous x, y ∈ R.

Exercice 252. Solution


Soit f : R
P→ R une fonction convexe. Montrer que ∀(λ1 , λ2 , . . . , λn ) ∈]0, 1[n , ∀(x1 , x2 , . . . xn ) ∈ Rn
n
tels que k=1 λk = 1, !
Xn Xn
f λ k xk ≤ λk f (xk )
k=1 k=1

En déduire l’inégalité arithmético-géométrique.


Rappel : On dit qu’une fonction est convexe si ∀(x, y) ∈ R2 , ∀λ ∈ [0, 1], f ((1 − λ)x + λy) ≤
(1 − λ)f (x) + λf (y)
Source : Inégalité de Jensen

Exercice 253. Solution


Soit f : N −→ N une fonction sujective et g : N −→ N une fonction injective telles que, pour tout
n, on ait :
f (n) ≥ g(n)
Démontrer que f = g.
Source : Roumanie 1986

Exercice 254. Solution


Soit f : N −→ N une fonction. Démontrer qu’il existe un entier n tel que f (f (n)) ̸= n + 2023
Source : IMO 1987

Exercice 255. Solution


Soit α ∈ R et p ∈ R[X]. Montrer que α est une racine double de P SSI P (α) = 0 et P ′ (α) = 0. En
deduire que α est racine de multiplicite k SSI P (α) = P ′ (α) = . . . = P (k−1) (α) = 0
On dira que α est racine de multiplicite k de P si il exsite un polnome Q tel que P = (X − α)k Q
et Q(α) ̸= 0

Exercice 256. Solution


Montrer que l’ensemble des solutions de l’inequation
70
X k 5

x−k 4
k=1

est une reunion d’intervalles disjoints dont la somme des longueurs vaut 1988.

Exercice 257. Solution


Considérons le polynôme du second degré x2 + ax + b avec des coefficients réels. On sait que la
condition nécessaire et suffisante sur ce polynôme pour posséder des racines réelles est que son
discriminant a2 − 4b soit supérieur ou égal à 0. Noter que ce discriminant est aussi un polynôme de
variables a et b. Prouver que la même chose n’est pas vraie pour les polynômes de degré 4 : Montrer
qu’il n’existe pas de polynômes à 4 variables P (a, b, c, d) tel que le polynôme x4 + ax3 + bx2 + cx + d
s’écrive comme un produit de 4 polynômes de degré 1 si et seulement si P (a, b, c, d) ≥ 0 (tous les
coefficients étant réels).

28
Exercice 258. Solution
Montrer que pour tous θ, φ ∈ R,
• cos(θ + φ) = cos(θ) cos(φ) − sin(θ) sin(φ)
• sin(θ + φ) = cos(θ) sin(φ)
 + sin(θ)
 cos(φ)

• cos(θ) + cos(φ) = 2 cos θ+φ cos θ−φ
 2  2
θ+φ
• sin(θ) + sin(φ) = 2 sin 2 cos θ−φ 2
Source : Formules de duplication

Exercice 259. Solution


Calculer :
n  
X n
cos(kx)
k
k=0

Exercice 260. Solution


Dans tout le probleme, p est un nombre premier impair
1) Montrer que si a, b ∈ N sont la somme de deux carres, alors ab est la somme de deux carres.
2) Montrer que si p est la somme de deux carres alors p ≡ 1 (mod 4).
On suppose maintenant que p ≡ 1 (mod 4)
3) Determiner le nombre de carres distincts modulo p.
4) Montrer qu’il existe un entier n tel que n2 ≡ −1 (mod p) SSI p ≡ 1 (mod 4)
5) Montrer qu’il existe des entier a, b des entiers tels que

√ n 1
0<b< p et b −a ≤ √
p p

6) Montrer que p = (bn − ap)2 + b2


7) En deduire le theoreme des deux carres de Fermat : un entier est la somme de deux carres SSI
tous ses diviseurs premiers p congrus a 3 modulo 4 ont une valuation p-adique paire.
Source : Théorème des deux carrés

Exercice 261. Solution


L’objectif de cet exercice est de démontrer l’inégalité arithmético-géométrique, donc on ne la sup-
posera pas connue.
a+b √
1) Soient a, b > 0. Montrer que ≥ ab
2
a+b+c+d √
2) Soient a, b, c, d > 0. Montrer que
4
≥ abcd
4
a+b+c √
3) En déduire que
3
≥ abc
3
4) Conclure pour n variables.
Source : Inégalité arithmético-géométrique

Exercice 262. Solution


Trouver toutes les solutions (x1 , x2 , x3 , x4 , x5 ) du système
 2

 (x1 − x3 x5 )(x22 − x3 x5 ) ≤ 0

 2 2
(x2 − x4 x1 )(x3 − x4 x1 ) ≤ 0



(x23 − x5 x2 )(x24 − x5 x2 ) ≤ 0

(x2 − x1 x3 )(x25 − x1 x3 ) ≤ 0


 4


(x2 − x x )(x2 − x x ) ≤ 0

5 2 4 1 2 4

où x1 , x2 , x3 , x4 , x5 sont des nombres réels strictement positifs.


Source : IMO 1972

29
1.4.3 Combinatoire

Exercice 263. Solution


Soit une suite de taille RS + 1, montrer qu’il existe soit une sous-suite croissante de taille R + 1,
soit une sous-suite décroissante de taille S + 1.
Source : Théorème d’Erdos-Szekeres

Exercice 264. Solution


Montrer que parmi 9 personnes, il y en a 3 qui se connaissent 2 à 2 ou 4 qui ne se connaissent pas
2 à 2. Montrer que le résultat est faux pour tout nombre strictement inférieur à 9.

Exercice 265. Solution


On donne un grillage de m × n carrés et 3 couleurs.
On veut colorier chaque segment du grillage avec une des 3 couleurs de telle sorte que chaque carré
unité a deux côtés d’une couleur et deux côtés d’une deuxième couleur. Combien de tels coloriages
sont possibles ?

Exercice 266. Solution


Plaçons-nous dans un grillage de côté n × n, de combien de façons peut-on aller du point en bas
à gauche au point en haut à droite, si l’on ne peut que monter d’un point ou aller un point sur la
droite à chaque tours, de sorte que l’on ne passe jamais sous la grande diagonale (reliant le sommet
en bas à gauche au sommet en haut à droite).

Exercice 267. Solution


Un tétromino est une tuile qui peut être formée en collant côte à côte quatre carrés d’une unité
de côté. Prouver que le nombre de façons de recouvrir le sol d’une salle de bain de taille 2 × 2n
avec n tétrominos est un carré parfait, quel que soit n ≥ 1. (Une suite entièrement déterminée est
acceptée comme réponse)
Source : Australie 2020

Exercice 268. Solution


Deux personnes jouent sur un échiquier infini composé de carrés 1 × 1. Le joueur 1 choisit un carré
et le marque avec un O. Ensuite, le joueur 2 choisit un autre carré et le marque avec un X. Ils
jouent ainsi, chacun à leur tour, jusqu’au moment où un joueur a marqué une ligne ou une colonne
de cinq carrés consécutifs et ce joueur gagne la partie. Si aucun joueur ne parvient à réaliser cela,
la partie est nulle. Montrer que le joueur 2 peut empêcher le joueur 1 de gagner.

Exercice 269. Solution


Soit n un entier strictement positif. Domitille dispose d’un tableau rectangulaire découpé en carrés
unités. À l’intérieur de chaque carré unité est écrit un entier strictement positif. Elle peut effectuer
les opérations suivantes autant de fois qu’elle le souhaite :
• Choisir une rangée et multiplier chaque nombre de la rangée par n.
• Choisir une colonne et retrancher n à chaque entier de la colonne.
Déterminer toutes les valeurs de n pour lesquelles la propriété suivante est vérifiée :
Quelles que soient les dimensions du rectangle et les entiers écrits dans les cases, Domitille peut
aboutir à un rectangle contenant uniquement des 0 au bout d’un nombre fini d’opérations
Source : Envoi de Combinatoire, 2022

Exercice 270. Solution


Soit n ≥ 3 un entier. On dit qu’un entier naturel non nul est atteignable si c’est 1 ou s’il peut être
obtenu à partir de 1 par une chaîne d’opérations telle que :
• La première opération est une addition ou une multiplication.
• Ensuite, les opérations se suivent en alternant addition et multiplication.
• À chaque étape, on prend le nombre obtenu à l’étape précédente (initialement 1) et on lui applique
l’opération, soit avec 2, soit avec n.
1) Montrer que, si n assez grand, il existe une infinité d’entiers non atteignables.
2) Montrer que, pour n = 3, tous les entiers à part 7 sont atteignables.
Source : Envoi de Combinatoire, 2022

30
Exercice 271. Solution
Sur une ligne infinie, se trouvent des cailloux, avec un ou plusieurs cailloux par case. James a deux
opérations possibles :
a) Enlever un caillou à la case n et n + 1 et en mettre un à la case n + 2.
b) Enlever deux cailloux à la case n, en mette un à la case n + 1 et un à la case n − 1.
Montrer qu’il y a toujours un moment au bout duquel James ne peut plus bouger ses pions.

Exercice 272. Solution


On considère la suite suivante : 1, 0, 1, 0, 1, 0, 2, 3, · · · où les termes à partir du 7-ième sont définis
comme le reste modulo 10 de la somme des 4 termes précédents. Existe-t-il six termes consécutifs
de la suite qui sont 0, 1, 0, 1, 0, 1 ?

Exercice 273. Solution


Montrer que  
m+n X mn
=
k i j
i+j=k

En déduire la valeur de
n  2
X n
k
k=0

Source : Identité de Vandermonde

Exercice 274. Solution


On considère quatre parties convexes du plan telles que l’intersection de trois d’entre elles est
toujours non vide.
a) Montrer que l’intersection des quatre convexes est non vide.
b) Le théorème reste-t-il vrai en remplaçant 4 par n ≥ 4 ?
Source : Théorème de Helly en dimension 2

Exercice 275. Solution


Une configuration de 4027 points du plan est appelée colombienne si elle est constituée de 2013
points de couleur rouge et de 2014 points de couleur bleue, et si trois quelconques de ces points
ne sont pas alignés. En traçant des droites, le plan est divisé en régions. Un tracé de droites est
appelé bon pour une configuration colombienne si les deux conditions suivantes sont vérifiées :
• aucune droite tracée ne passe par un point de la configuration ;
• aucune région ne contient des points de couleurs différentes.
Trouver la plus petite valeur de k telle que, pour chaque configuration colombienne de 4027 points,
il existe un bon tracé de k droites.
Source : IMO 2013

Exercice 276. Solution


On dit qu’un ensemble de points du plan est obtus lorsque trois points quelconques de cet ensemble
sont toujours les sommets d’un triangle obtus. Prouver que tout ensemble de
√ n points du plan, trois
quelconques jamais alignés, contient un sous-ensemble obtus d’au moins n éléments. On pourra
d’abord démontrer la forme faible du théorème d’Erdös-Szekeres : toute suite de taille p2 +1 possède
une sous-suite monotone de taille p

31
1.4.4 Géométrie

Exercice 277. Solution


Soit ABC un triangle. Notons I le centre de son cercle inscrit, IA le centre de son cercle A-exinscrit
(c’est à dire l’intersection de la bissectrice intérieure de BAC
\ et des bissectrice extérieures de ABC
\
et ACB) .
\
a) Montrer que I, B, IA , C sont cocycliques.
b) Montrer que S, le pôle sud de ABC, est le centre de ce cercle.

Exercice 278. Solution


Soit ABC un triangle, soit ω son cercle circonscrit. Soit P un point quelconque, soient PA , PB , PC ,
les projetés orthogonaux de P sur sur BC, AC, et AB.
Montrer que P appartient à ω si et seulement si PA , PB , PC sont alignés.
Source : Droite de Simson

Exercice 279. Solution


Soit ABC un triangle isocèle en A. Soit D un point de [CA) tel que D n’appartienne pas au
segment [AC] et que A ne soit pas le milieu de CD. Notons d1 , d2 les bissectrices intérieure et
extérieure de BAC,
\ et ∆ la médiatrice de [BD]. Notons enfin E (resp. F ) l’intersection de ∆ avec
d1 (resp d2 ).
Montrer que les points A, E, F , D sont cocycliques
Source : France TST 2019

Exercice 280. Solution


Soit ABCD un quadrilatère de côtés a, b, c, d. Montrer que ABCD est circonscriptible SSI a + c =
b + d.

Exercice 281. Solution


Soient C1 , C2 et C3 trois cercles. Montrer que leurs centres d’homothétie positives sont alignés.
On admettera que la composition de deux homothéties dont le produit des rapports est différent
de 1 est une homothétie dont le centre se situe sur la droite des centres.
Source : Théorème de Monge

Exercice 282. Solution


Soit ABCD un quadrilatère convexe tel que les cercles inscrits à ABC et ADC soient tangents.
Montrer que ABCD possede un cercle inscrit.

Exercice 283. Solution


Soient ABC un triangle, O le centre de son cercle circonscrit, G son centre de gravité (ie le point
d’intersection de ses médianes) et H son orthocentre (ie le point d’intersection de ses hauteurs).
On note A0 , B0 et C0 les milieux de [BC], [CA] et [AB] et D, E, F les pieds des hauteurs issues
de A, B et C.
Montrer que O, G et H sont alignés.
Source : Droite d’Euler

Exercice 284. Solution


Soient ABC un triangle, O le centre de son cercle circonscrit, G son centre de gravité (ie le point
d’intersection de ses médianes) et H son orthocentre (ie le point d’intersection de ses hauteurs).
On note A0 , B0 et C0 les milieux de [BC], [CA] et [AB] et D, E, F les pieds des hauteurs issues
de A, B et C.
Montrer que A′ , B ′ , C ′ , D, E, F sont cocyliques.
Source : Cercle d’Euler

32
2 Solutions des Exercices

2.1 Apéritif

2.1.1 Arithmétique

Solution 1. Enoncé
On pose S = (a − b)(a + b). En développant, on a donc S = a2 + ab − ab − b2 = a2 − b2 .
Solution 2. Enoncé
Supposons n impair. Il existe donc un entier k ∈ Z tel que n = 2k + 1. On a alors n2 = (2k + 1)2 =
4k 2 + 4k + 1 = 2(2k 2 + 2k) + 1 = 2k ′ + 1 ou k ′ = 2k 2 + 2k ∈ Z. Ainsi, si n est impair, alors n2 est
impair, donc par contraposée, si n2 est pair, alors n est pair.
Solution 3. Enoncé
1 a
Supposons par l’absurde que est un nombre décimal. Il peut donc s’écrire sous la forme b avec
3 10
a et b des entiers relatifs. On a alors 10b = 3a, donc 10b doit etre divisible par 3, ce qui constitue
1
une contradiction. Ainsi, notre hypothese initiale doit etre fausse, ie ne peut pas etre un nombre
3
décimal.
Solution 4. Enoncé
Montrons par récurrence la proposition
n(n + 1)
P(n) : ”1 + 2 + ... + n = ”
2
1(1 + 1)
Initialisation : P(1) : ”1 = ” est bien vraie.
2
Hérédité : Supposons P(n) vraie pour un certain entier naturel n. On veut montrer qu’alors,
n(n + 1)
P(n + 1) est vraie. Comme on sait que P(n) est vraie, on sait que 1 + 2 + ... + n = , donc
2
n(n + 1) (n + 2) (n + 1)(n + 2)
1 + 2 + ... + n + (n + 1) = + (n + 1) = (n + 1) = , donc P(n + 1)
2 2 2
est vraie.
Conclusion : La proposition est intialisée et héréditaire, donc d’après le principe de récurrence,

n(n + 1)
∀n ∈ N∗ , 1 + 2 + ... + n =
2

Solution 5. Enoncé
a | b et c | d donc il existe k, k ′ ∈ Z tels que b = ka, d = k ′ c, donc bd = (kk ′ ) ac i.e. ac | bd
| {z }
∈Z

Solution 6. Enoncé
Si n | n + 7, alors n | (n + 7 − n) donc n | 7 donc n ∈ {−7, −1, 1, 7}. Réciproquement, tous ces
entiers conviennent bien.
S = {−7, −1, 1, 7}

Solution 7. Enoncé
Pour tout n ∈ N, |n2 + 1| > |n|, donc la seule possibilite est que n = 0. Réciproquement, on a bien
1 | 0.
S = {0}

Solution 8. Enoncé
On utilise l’algorithme d’Euclide :
364 = 2 × 154 + 56
154 = 2 × 56 + 42
56 = 1 × 42 + 14
42 = 3 × 14 + 0

33
Solution 9. Enoncé
Parmi deux nombres consécutifs, au moins l’un des deux est pair, donc n(n + 1) est pair, donc
2 | n(n + 1)(2n + 1). Quand a la divisibilité par 3, si 3 | n, on a fini, sinon le reste de la division
euclidienne de n par 3 est 1 ou 2. Dans le premier cas, on vérifiera que 2n + 1 est divisible par 3
et dans le deuxième, que n + 1 est divisible par 3. Dans tous les cas, n(n + 1)(2n + 1) est divisble
par 3, et donc par 6.

Montrons maintenant par récurrence la proposition


n(n + 1)(2n + 1)
P(n) : ”12 + 22 + ... + n2 = ”
6
Initialisation : P(1) : ”1 = 1(1+1)(2×1+1)
6 ” est bien vraie.
Hérédité : Supposons P(n) vraie. On veut montrer qu’alors, P(n + 1) est vraie. Comme on sait que
n(n + 1)(2n + 1)
P (n) est vraie, on sait que 12 + 22 + ... + n2 = , donc 12 + 22 + ... + n2 + (n + 1)2 =
6
n(n + 1)(2n + 1) n(2n + 1) + 6(n + 1) (n + 1)(n + 2)(2n + 3)
+(n+1)2 = (n+1) = , donc P(n+1)
6 6 6
est vraie.
Conclusion : La proposition est intialisée et héréditaire, donc d’après le principe de récurrence,

n(n + 1)(2n + 1)
∀n ∈ N∗ , 12 + 22 + ... + n2 =
6
Solution 10. Enoncé
On sait que le pgcd de n et n + 2 divise n + 2 − n = 2. Si n est pair, n et n + 2 partagent un facteur
commun 2. Si n est impair, alors n ne possède pas de facteur 2, donc n et n + 2 sont premiers entre
eux.
On a n2 − 1 = (n + 1)(n − 1) et n2 − 2n + 1 = (n − 1)2 , donc les deux nombres possèdent n − 1 en
facteur commun. Pour qu’ils soient premiers entre eux, il faut que n − 1 = ±1. Si n − 1 = 1, alors
n = 2 et on a effectivement 3 qui est premier avec 1. Si n − 1 = −1, alors n = 0 et on a bien −1
qui est premier avec 4.
Solution 11. Enoncé
2x + 1 est impair, donc premier avec 8, donc d’apres le lemme de Gauss, si 2x + 1 | 8y, alors
2x + 1 | y
Solution 12. Enoncé
Remarquons que parmi p et p + 1, il y en a un des deux qui est pair. Or le seul nombre premier
pair est 2, donc soit p, soit p + 1 est égal à 2. Dans le premier cas, on a bien une solution, dans le
deuxième non (1 n’est pas premier).
Remarquons que parmi p et p + 1, il y en a un des trois qui est divisible par 3. Or, le seul nombre
premier divisible par 3 est 3, donc l’un de p, p + 2, p + 4 est égal à trois. Seul le premier cas donne
effectivement une soltion.
Solution 13. Enoncé
On applique le critère de divisibilité par 11, on a 2 − 9 + 0 − 3 + 7 − 4 + 8 − 9 = −8, donc 98473092
n’est pas divisible par 11.
Solution 14. Enoncé
β1
Notons a = pα 1 ×. . .×pk et b = q1 ×. . .×qr les décompositions en produits de facteurs premiers
1 αk βr

de a et b. Alors raisonons par contraposée : Si a ne divise pas b, il exsite l tel que l’exposant de pl
dans la décomposition de b soit inferieur à αl , ce qui ne changera pas après multiplication par 2,
donc a2 ne divise pas b2 .
Solution 15. Enoncé
Problème Mathraining #3120
Solution 16. Enoncé
Le poroblème revient à montrer que n3 ≡ n (mod 6). On vérifie aisément que cela est vrai en
testant n = 0, 1, 2, 3, 4, 5.
Source : Envoi d’arithmétique 2022
Solution 17. Enoncé
On regarde 32023 modulo 10 : on remarque que 34 = 81 ≡ 1 (mod 10) donc il suffit de regarder le
reste de la division euclidienne de 2023 par 4, ie 32023 ≡ 33 ≡ 7 (mod 10)

34
Solution 18. Enoncé
L’équation est équivalente à x2 − y 2 = 13. On factorise : (x − y)(x + y) = 13. Or, x − y et x + y
sont entiers, et 13 est premier, donc :
( ( ( (
x−y =1 x − y = 13 x − y = −1 x − y = −13
ou ou ou
x + y = 13 x+y =1 x + y = −13 x + y = −1

Donc les solutions sont :


S = {(7, 6), (7, −6), (−7, −6), (−7, 6)}

Solution 19. Enoncé


On regarde l’équation modulo 4 : il faudrait que x2 + y 2 ≡ 3 (mod 4). Or, un carré modulo 4 vaut
0 ou 1, donc la somme de deux d’entre eux ne pourra jamais valoir 3. Il n’y a donc pas de solution.

Solution 20. Enoncé


On remarque que si x et z sont tous les deux superieurs ou égaux à 3, on n’a pas de solution
puisque z! − x! = 2y devrait etre divisible par 3. Or, z > x donc x ∈ {0, 1, 2}.
Si x = 0, alors si y > 0, le membre de gauche est impair, donc on a z < 2, impossible. Si y = 0, on
a une solution (x, y, z) = (0, 0, 2)
Si x = 1, on a x! = 0!, donc de meme, on a une solution pour (x, y, z) = (1, 0, 2)
Si x = 2, si y ≥ 2, le membre de gauche est congru à 2 modulo 4, donc z ≤ 3 et comme z > 2, on
a (x, y, z) = (2, 2, 3). Si y = 0, on n’a pas de solution. Si y = 1, on a pas de solution.
Conclusion :
S = {(0, 0, 2), (1, 0, 2), (2, 2, 3)}
Source : Envoi Pot-Pourri 2023
Solution 21. Enoncé
a) On regarde modulo 4 : a2 + b2 ≡ 3 (mod 4), mais c’est impossible car un carré vaut 0 ou 1
modulo 4. Il n’y a donc pas de solution
b) On regarde modulo 8 : a2 + b2 + c2 ≡ 7 (mod 8). Mais un carré est congru à 0, 1, 4 modulo 8,
dans tous les cas, la somme de 3 d’entre eux ne vaut jamais 7. Il n’y a donc pas de solution.
Solution 22. Enoncé
On remarque que 5 | 87457561565 et 5 ne divise pas 100000000000000000000000007, ce qui contre-
dirait la transitivite de la divisibilite si 87457561565 divisait 100000000000000000000000007
Solution 23. Enoncé
On a deux solutions : Soit on remarque que 9 ≡ 2 (mod 7), donc 92 ≡ 2n (mod 7), ce qui conclut,
soit on utilise la factorisation de Bernoulli qui nous donne que 9n − 2n = (9 − 2)(9n−1 + 2 × 9n−2 +
. . . + 2n−1 ).
Solution 24. Enoncé
On regarde les valeurs possibles pour n2 + n + 1 modulo 7. On trouve que les seules solutions sont
les nombres congrus a 2 et 4 modulo 7.
Solution 25. Enoncé
Soit p un diviseur premier commun de ab et de a + b. Alors p | a ou p | b. Supposons sans perte de
generalite que p | a. Alors comme p | a + b, alors p | (a + b) − a = b. Donc p = 1 car a et b sont
premiers entre eux, donc ab et a + b sont premiers entre eux.

Solution 26. Enoncé


On remarque que le produit de k entier consecutifs peut s’ecrire n × (n − 1) × . . . × (n − k + 1).
Alors on a  
n × (n − 1) × . . . × (n − k + 1) n! n
= =
k! (n − k)! × k! k
qui est bien un entier.
Solution 27. Enoncé
On peut lister les nombres entiers compris entre 1 et 20, et on compte ceux qui sont premiers avec
20. On trouve φ(20) = 8

35
Solution 28. Enoncé
On veut connaitre le nombre de facons de constituer une equipe de k personnes si on a un groupe
de n personnes. D’un cote cela vaut evidemment nk .D’un autre, on peut considerer la derniere
personne du groupe de n : si on decide de la prendre dans l’equipe, il reste a choisir k − 1 personnes
parmi les n−1 restantes, sinon il reste a choisir k personnes parmi les n−1 restantes. En combinant
les deux cas, on a donc bien
     
n n−1 n−1
= +
k k−1 k
Source : Formule d’Euler
Solution 29. Enoncé
On peut effectuer un comptage direct en developpant, sinon on procede par recurrence :
Initialisation : (a + b)0 = 1
HéréditéPn: Supposons la propriété
Pn vraie
 pour un certain

Pn nn∈ kN n−k+1. Alors (a+b)n+1 = (a+b) n
Pn ×(a+b) =
n k n−k n k+1 n−k n
 n+1 n+1
(a + b) k=0 k a b = k=0 k a b + k=0 k a b =a +b + k=1 ( k−1 +
n
 k n+1−k Pn+1 n+1 k n+1−k
k )a b = k=0 k a b
Conclusion : La proposition est initialisée et héréditaire, donc pour tout n ∈ N∗ ,

n  
n
X n k n−k
(a + b) = a b
k
k=0

Source : Binome de Newton

36
2.1.2 Algèbre

Solution 30. Enoncé


Soit x ∈ R\{1}. Montrons par récurrence la proposition
1 − xn+1
P(n) : ”1 + x + x2 + . . . + xn =
1−x
Initialisation : 1 = 1−x
1−x , donc P(0) est vraie
Hérédité : Supposons la propriété vraie pour un certain n ∈ N. On a donc :
1 − xn+1
1 + x + . . . + xn + xn+1 = + xn+1
1−x
1 − xn+1 + xn+1 − xn+2
=
1−x
1 − xn+2
=
1−x
Conclusion : La proposition est initialisée et héréditaire donc d’apres le principe de récurrence :

1 − xn+1
∀n ∈ N, 1 + x + . . . + xn =
1−x

Solution 31. Enoncé


On peut traiter le cas ou x est positif puis le cas ou x est négatif, dans les deux cas de résultat est
vrai. On a égalité SSI x = 0.
Pour tous a, b ∈ R, on a (a − b)2 ≥ 0, donc
√ en
√développant, a + b ≥ 2ab. On a égalité SSI a = b
2 2

En appliquant le résultat précédent à a et b, on a le résultat voulu. On a égalité SSI a = b


Solution 32. Enoncé
On procède par récurrence :
Initialisation : u0 = a
Hérédité : Supposons la propriété vraie pour un certain n ∈ N∗ . Alors un+1 = un +q = a+nq +q =
a + (n + 1)q
Conclusion : La proposition est initialisée et héréditaire, donc vraie pour tout entier naturel, ie

∀n ∈ N, un = a + nq

Source : Terme général d’une suite arithmétique


Solution 33. Enoncé
On procède par récurrence :
Initialisation : u0 = a
Hérédité : Supposons la propriété vraie pour un certain n ∈ N∗ . Alors un+1 = un × q = aq n × q =
aq n+1
Conclusion : La proposition est initialisée et héréditaire, donc vraie pour tout entier naturel, ie

∀n ∈ N, un = aq n

Source : Terme général d’une suite géométrique


Solution 34. Enoncé
1) Pour tout entier naturel n, on a un = n (récurrence immédiate) donc la suite est évidemment
croissante.
2) un+1 − un = 2(n + 3) > 0 donc la suite est croissante.
3) un+1 − un = u2n − 2un + 1 = (un − 1)2 ≥ 0 donc la suite est croissante.
Solution 35. Enoncé
x106
On montre par récurrence immédiate que xn = 2n . Alors = 24 = 16
x102
Solution 36. Enoncé √
a3 +b3 +a+b
On applique l’inégalité arithmético-géométrique : a3 b3 ab ie
4
4 ≥

a3 + b3 + a + b ≥ 4ab

37
Solution 37. Enoncé
On essaye f (x) = x : A gauche on a x + y + x2 + y 2 + 2xy. A droite on a x + y + (x + y)2 . En
développant le membre de gauche (ou en factorisant le membre de droite), on constate que c’est
effectivement une solution.
Solution 38. Enoncé 2 2
On essaye f (x) = x2 : A gauche, on a (x2 + y2 )2 +y 2 = x4 +x2 y + 5y4 et à droite, on a x4 + 5y4 +x2 y,
donc on constate que c’est effectivement une solution.
Solution 39. Enoncé
Par contraposée, si f n’est pas injective, il exsite x > y tels que f (x) = f (y), mais alors la fonction
n’est pas strictement monotone, ce qui conclut.
Solution 40. Enoncé
b 2 2
Pour tout nombre réel x, on a P (x) = a x + 2a + 4ac−b la forme canonique de P . On a donc

4a
b 2 b2 −4ac
P (x) = 0 ⇐⇒ x + 2a = 4a2 . Il faut donc b − 4ac ≥ 0. Réciproquement, si b2 − 4ac ≥ 0 on
 2

−b ± b2 − 4ac
a bien des solutions x =
2a
Solution 41.PEnoncé Pmax(n,m)
n Pm
On note P = k=0 ak X k et Q = k=0 bk X k avec an et bm non-nuls. Alors P +Q = k=0 (ak +
bk )X k en complétant la suite la plus courte avec des 0. P + Q est donc bien un polynome à coeffi-
cients réels. Pn Pm
De meme, P Q = k=0 l=0 ak bl X k+l est bien un polynome à coefficients réels (on peut regrouper
les termes de meme degré)
Solution 42. Enoncé
En utilisant les formes vues précédement, on a deg(P + Q) ≤ max(deg P, deg Q) avec une inégalité
stricte SSI P et Q ont le meme degré et des coefficients dominants opposés. De meme, comme an
et bm sont non-nuls, on a an bm qui est non-nul, donc P Q est bien de degré n + m = deg P + deg Q
Solution 43. Enoncé
A est bien un polynome de degré 2, il correspond à la définition.
On remarque que B 2 = X est un polynome de degré 1, or d’apres l’exercice précédent, cela veut
dire que B serait de degré 21 , ce qui est impossible. B n’est donc pas un polynome.
C est un polynome de degré 42
En developpant, D est un polynome de degré 1.
Solution 44. Enoncé
A est de degré 7 et de coefficient dominant 1
B est le produit de deux polynomes de degré 1 et d’un polynome de degré 5, tous de coefficients
dominants 1, donc il est de degré 7 et de coefficient dominant 1
C est la somme d’un polynome de degré 5 et d’un produit de polynomes de degrés 1, 2, 3. C’est
donc un polynome de degré 6 , de coefficient dominant le coefficient devant X 6 ie 3
Solution 45. Enoncé
1) partie réelle : 3 ; partie imaginaire : 4
√; module : 5 ; conjugué : 3 − 4i √
2) partie réelle : 1 ; partie imaginaire : 15 ; module : 4 ; conjugué : 1 − 15i
3) partie réelle : 0 ; partie imaginaire : 2 ; module : 2 ; conjugué : −2i
Solution 46. Enoncé
1 a − ib a b
On a = = 2 2
−i 2
a + ib (a + ib)(a − ib) a +b a + b2
Solution 47. Enoncé
Module : |z|2 = zz
z+z
Partie réelle : ℜ(z) =
2
z−z
Partie imaginaire : ℑ(z) =
2i
Solution 48. Enoncé
L’inégalité est équivalente à 0 ≤ x2 − 2x + 5 = (x − 1)2 + 4, ce qui est évidemment vrai car un
carré est positif.

38
Solution 49. Enoncé
L’inégalité est équivalente à 0 ≤ a2 + b2 + c2 + 2ab − 2bc − 2ac = (a + b − c)2 , ce qui est évidemment
vrai car un carré est positif.
Solution 50. Enoncé
L’expression se factorise : abc + ab + ac + bc + a + b + c + 1 = (a + 1)(b + 1)(c + 1) ≥ 0 car a, b, c ≥ −1
Solution 51. Enoncé
D’un coté, on a :
a b c d a b c d
+ + + > + + + =1
a+b+d b+c+a b+c+d a+c+d a+b+c+d a+b+c+d a+b+c+d a+b+c+d
De l’autre coté, on a :
a b c d a b c d
+ + + < + + + =2
a+b+d b+c+a b+c+d a+c+d a+b b+a c+d c+d
| {z } | {z }
=1 =1

Source : IMO
Solution 52. Enoncé
On développe et on ramène tout du même côté, notre inegalité est alors équivalente à a2 + b2 −
2ab + c2 + d2 − 2cd ≥ 0. On retrouve ici des identités remarquables : notre inégalité est équivalente
à (a − b)2 + (c − d)2 ≥ 0, ce qui est evidemment vrai.
Solution 53. Enoncé
π
On passe le membre de droite sous forme exponentielle, donc notre équation devient z 3 = 8ei 4 .
π
Une solution est donc evidemment z = 2ei 12 . Avec les racines troisièmes de l’unité, on a
π 5π 9π
S = {2ei 12 , 2ei 12 , 2ei 12 }

39
2.1.3 Combinatoire

Solution 54. Enoncé


Comme dans tout problème de ce type, il faut montrer 2 choses :
• 732 n’est pas suffisant. En effet, on peu attribuer chacun des 366 jours de l’année à exactement
2 élèves, donc il n’y en a pas forcément 3 élèves qui ont leur anniversaire le meme jour.
• 733 est suffisant : d’après le principe des tiroirs, si on considère les élèves comme les chaussettes
et les jours de l’année comme les tiroirs, il y a au moins un tiroir qui contient au moins 733
 
366 = 3
chaussettes.
Solution 55. Enoncé
Pour tout i ∈ [[1, 2022]], notons ri le reste de la divison euclidienne de xi par 2021. Tous les ri sont
entre 0 et 2020, donc d’après le principe des tiroirs, il en existe deux qui sont égaux, ie il existe
i, j ∈ [[1, 2022]] tels que i ̸= j et xi − xj est divisible par 2021
Solution 56. Enoncé
On peut regrouper les éléments de l’ensemble par paires, tels que la somme de deux éléments d’une
meme paire fasse toujours 13 : {{1, 12}, {2, 11}, {3, 10}, {4, 9}, {5, 8}, {6, 7}}.
D’après le principe des tiroirs, en choisissant 7 éléments dans l’ensemble, on en prendra deux dans
la meme paire, donc on en aura 2 qui sommeront à 13
Solution 57. Enoncé
Supposons par l’absurde que Ω possède un nombre fini d’éléments. Introduisons un axe des abscisses
de telle sorte que deux points distincts n’aient pas la meme abscisse (ce qui est bien sur possible,
il y a une infinité de directions mais un nombre fini de points). Regardons le point d’abscisse la
plus grande. Il doit etre le milieu de deux autres, mais tous les points ont une abscisse strictement
inferieure, ce qui constitue une absurdité.
Solution 58. Enoncé
Pour chaque étudiant, on a 2 possibilités : soit lui donner un bonbon de la première sorte, soit un
bonbon de la deuxième sorte, donc on a 220 possibilités
Solution 59. Enoncé
Pour que la moyenne arithmétique de deux nombres soit un entier, il faut que ces deux nombres
soient de la meme parité. La moyenne arithmétique de deux nombres est uniquement déterminée
par ces deux nombres. Le nombre de possibilités de choisir trois nombres dont l’un est la moyenne
arithmétique des deux autres est donc le nombre de facons de choisir deux nombres pairs distincts,
plus le nombre de facons de choisir deux nombres impairs distincts, soit :
   
50 50
+ = 2450
2 2
Solution 60. Enoncé
Pour la première ligne, on a 8 choix pour la position de la tour, pour la 2e ligne on en a plus que
7. On procède ainsi et on trouve que le nombre de facons recherché est 8! = 40320
Dans le cas ou on a que 6 tours, il faut d’abord choisir les deux lignes qui ne contieneront pas de
tour, puis appliquer le raisonnement précédent. On trouve un nombre de possibilités total de
 
8 8!
×
2 2!

Solution 61. Enoncé


Le nombre de facons de choisir k personnes dans un groupe de n personnes est le meme que le
nombre de facons de ne pas choisir k personnes dans un groupe de n personnes (lorsqu’on choisit
k personnes, il y en a n − k que l’on choisit pas). On a donc
   
n n
=
k n−k

Solution 62. Enoncé


On note v le nombre de vaches et p le nombre de poules. On a v + p = 160 et 4v + 2p = 400. On
a donc v + p = 160 et 2v + p = 200. En soustrayant la première égalité à la deuxième, on trouve
v = 40 .
Source : Coupe Animath de Printemps 2019

40
Solution 63. Enoncé
On remarque qu’une pièce 1 × 3 recouvre 3 cases. Or, une grille 4 × 4 n’a pas un nombre de cases
divisible par 3 : on ne peut donc pas la recouvrir avec de telles pièces. Le meme raisonnement
s’applique à une grille de taille 2n × 2n pour n ∈ N∗ .

Solution 64. Enoncé


Le réponse est oui. En effet, il suffit de retourner la pièce et de la coller en dessous d’une copie
pour former une grille 3 × 2. On peut alors aligner n notre nouvelle pièce et on a recouvert une
grille 3 × 2n.
Solution 65. Enoncé
La réponse est oui. En effet, on peut remplir une grille 4 × 4 comme sur la figure ci-dessous, donc
en alignant n fois cette figure, on aura bien recouvert notre grille 4 × 4n.

Solution 66. Enoncé


On remarque que le nombre de jetons est un invariant du problème. Il est donc impossible, en
partant de 2022 jetons, de terminer avec 2023 jetons.
Solution 67. Enoncé
On remarque que la parité du nombre d’interrupteurs allumés est un invariant du problème. En
effet, si on a k lampes allumées dans une ligne/colonne et qu’on active l’interrupteur, alors on se
retrouvera avec 4 − k lampes allumées en plus, et k lampes allumées en moins, ce qui fait une
différence de 4 − 2k = 2(2 − k) lampes, qui est bien pair. Etant donné qu’il y a un nombre pair de
lampes en tout et qu’on commence avec un nombre impair de lampes allumées, il est impossible
d’atteindre une position ou toutes les lampes seraient allumées.
Solution 68. Enoncé
On remarque que la parité du (nombre de pièces + nombre de piles) est un invariant du problème
(on laisse au lecteur le soin de s’en convaincre). Alors si on arrive à ne plus avoir de pièce, on a 0
pièce et 0 tas, donc la somme et paire, mais la somme initiale est impaire, donc c’est impossible.
Solution 69. Enoncé
On remarque que la parité de la somme des nombres écrits au tableau est un invariant du problème,
donc la parité du dernier nombre doit etre la meme que celle de 1 + . . . + 2021 = 2021 × 1011 qui
est impair : le dernier nombre ne pourra donc pas etre 2.
Solution 70. Enoncé
Il suffit à Bob de jouer le symétrique du coup d’Alice par rapport au centre du plateau pour
s’assurer de ne jamais perdre. Donc Bob possède une stratégie gagnante
Solution 71. Enoncé
On sépare le triangle en 4 plus petits triangles équilateraux de coté 0.5 (avec les milieux des cotés
du triangle). D’après le principe des tiroirs, il existe deux points dans le meme petit triangle. Or,
deux points dans le meme petit triangle sont bien sur à distance inferieure à 0.5 (la plus grande
longueur étant le coté du triangle), ce qui conclut.
Solution 72. Enoncé
Soit x > 0. On prend un triangle équilateral de coté x dans le plan. D’après le principe des tiroirs,
il exsite deux sommets du triangle de la meme couleur, et ils sont bien à distance x.
Solution 73. Enoncé
On représente le problème par un graphe complet ou les sommets sont les élèves et les aretes sont
coloriées de 2 couleurs : en vert si les deux élèves se connaissent et en rouge sinon. Le problème
revient à montrer qu’il exsite un triangle d’une meme couleur (monochromatique) dans le graphe.
On procède maintenant par l’absurde. Supposons avoir réussi à colorier le graphe sans triangle

41
monochromatique. On considère un sommet du graphe, dont par 5 aretes. Par le principe des ti-
roirs, 3 de ces aretes sont d’une meme couleur, supposons sans perte de généralité vertes. Si l’un
des trois points auquels sont reliés ces aretes sont reliés par une arete verte, on obtient un triangle
vert, absurde, sinon c’est que les trois aretes reliant les trois sommets sont rouges, donc on a un
triangle rouge, absurde.

Pour 5, ce n’est plus le cas, il suffit de considérer un graphe complet à 5 sommets dont le bord est
d’une couleur et l’interieur d’une autre couleur.
Solution 74. Enoncé
On peut faire des paires de nombres consécutifs entre 1 et 40. D’apres le principe des tiroirs, si
on choisit 21 nombres, il y en aura 2 dans la meme paire, ie il y en aura 2 consécutifs, et deux
nombres consécutifs sont toujours premiers entre eux.
Solution 75. Enoncé
Par l’absurde, supposons qu’aucun des amis n’ait assez de bougies pour feter son anniversaire. Alors
il y a au plus (a1 − 1) + . . . + (a7 − 1) = a1 + . . . + a7 − 7 bougies, mais on en a a1 + . . . + a7 − 6,
donc c’est absurde.
Solution 76. Enoncé  
n n(n − 1)
On compte le nombre de segments reliant deux sommets : il y en a = et on retire
2 2
les cotés du n-gone (il y en a n). On a donc
 
n n(n − 3)
−n=
2 2

Solution 77. Enoncé


Pour cet exercice, il suffit d’introduire un axe des abscisses tel que deux points n’aient jamais la
meme abscisse (c’est toujours possible puisqu’il y a une infinité de directions possibles pour l’axe et
un nombre fini de points). Ensuite, on peut regrouper les points par groupes de 2 pour les segments
et par groupes de 4 pour les quadrilatères, selon l’abscisse croissante.
Solution 78. Enoncé
Pour avoir un point d’intersection, comme les droites sont deux à deux non parallèles et trois à
trois non concourantes, il suffit de prendre n’importe quelle paire de droites. Il y a donc autant de
points d’intersection que de paires de droites, ie
 
n n(n − 1)
=
2 2

42
2.1.4 Géométrie

Solution 79. Enoncé


D’après le théorème de l’angle au centre, le centre du cercle circonscrit à ABC se situe sur le
segment BC. Or, il doit etre a égale distance de B et de C, donc M est le centre du cercle
circonscrit, ie
MA = MB = MC

Solution 80. Enoncé


Comme les droites (AB) et (CD) sont parallèles, on a par angles alternes-internes que CDA
\ est
égal au complémentaire de DAB, donc que CDA = 180 − CBA, donc ABCD est bien cyclique.
\ \ \

Solution 81. Enoncé


Soit O le centre de Ω. Les triangles OAP et OBP sont rectangles en A et en B respectivement.
Le théorème de Pythagore permet de conclure, en remarquant que OA = OB.
Solution 82. Enoncé
Il suffit de remarquer grace aux angles alternes-internes ou correspondants (en fonction de la
configuration, croisée ou non), que les triangles ABC et AB ′ C ′ sont semblables. La relation de
Thalès découle alors directement des relations de longueur sur les triangles semblables.
Source : Théorème de Thalès
Solution 83. Enoncé
Soit D le point d’intersection de (CO) avec le cercle. Les triangles COA et COB sont isocèles, donc
on a ACO
[ = CAO [ = φ1 et OCB \ = CBO\ = φ2 . D’après la relation de Chasles, on a AOD \ = 2φ1
et DOB = 2φ2 , donc 2 × ACB = AOB
\ \ \

43
Source : Théorème de l’angle au centre
Solution 84. Enoncé
D’après le théorème de l’angle au centre, 2β = AOB
\ = 2α. donc α = β.

Source : Théorème de l’angle inscrit


Solution 85. Enoncé
On apllique le théorème de l’angle inscrit deux fois, ce qui nous donne que CAB \ et CAD
\ = CDB \=
\ donc comme la somme des angles dans un triangle vaut 180, on a bien DCD
CBD, \ = 180 − BAD
\

Solution 86. Enoncé


\ = 40 donc ADC
CDB \ = 140, donc comme la somme des angles dans un triangle vaut 180,
\ = 20, donc le triangle ADC est isocele en D. Comme D est le milieu de [AB], le triangle
ACD
CDB est isocele en D, donc on trouve CBA
\ = (180 − 40)/2 = 70

44
Solution 87. Enoncé
On remarque que le quadrilatère AA′ B ′ B est cyclique, donc on a l’égalité d’angles P\A′ B ′ =
PB PA
\′ = P
180 − ABB \BA, donc les triangles P BA et P A′ B ′ sont semblables, donc on a = ie
P A′ P B′

P B × P B ′ = P A × P A′

Source : Puissance d’un point par rapport à un cercle


Solution 88. Enoncé
1) On remarque que le cercle circonscrit à HBC est tangent à la droite (AC) en C (on applique
l’exercice 79 au triangle rectangle HCB), donc d’après l’exercice précédent, appliqué au cas parti-
culier ou C et C ′ sont confondus, on a bien AH × AB = AC 2
2) On remarque que le cercle circonscrit à AHC est tangent à la droite (BC) en C (on applique
l’exercice 79 au triangle rectangle AHC), donc d’après l’exercice précédent, appliqué au cas parti-
culier ou C et C ′ sont confondus, on a bien BH × BA = BC 2
3) On peut effectuer le calcul à l’aide du théorème de Pythagore, mais c’est très calculatoire.
Une meilleure manière de traiter cet exercice est de démontrer que AHC, BHC et ABC sont
semblables (laissé en exercice au lecteur)

Source : Lemme d’Euclide

45
Solution 89. Enoncé
D’après le théorème de l’angle à la tangente, ABC
\ = ABP \. Or, la droite (B ′ C ′ ) est le symétrique
de la droite (BC) par rapport à P , donc elles sont parallèles, donc B
\ ′ CP = P \ CB. D’après la
réciproque du théorème de l’angle inscrit, on a fini.

Solution 90. Enoncé


On remarque que le trajet pour traverser la rivière ne change pas peu importe ou on place le
pont. Il faut donc minimiser le trajet en dehors de la rivière. Pour cela, on retire la rivière (en
pliant sa feuille par exemple) et on trace une ligne droite entre A et B. En dépliant la feuille et en
tracant la rivière entre les deux points sur les bords de la rivière, on a le trajet minimal (l’inégalité
triangulaire lorsque la feuille est pliée nous l’assure.)
Solution 91. Enoncé
Soit C ′ le symétrique de C par rapport à d. On trace la droite (BC ′ ) et on note D le point
d’intersection avec d. D est le point recherché : comme DC ′ = DC, l’inégalité triangulaire nous
l’assure.

Solution 92. Enoncé


Soit M le milieu de [AC] : ABCD est un parallélogramme SSI M est le milieu de [BD] SSI
B est le symétrique de D par rapport à M . Pour avoir D sur Γ, il suffit donc d’avoir B sur le
cercle symétrique à Γ par rapport à M . Comme B soit aussi etre sur Γ, B et D sont les points
d’intersection de Γ et de son symétrique.

46
Solution 93. Enoncé
Il suffit de tracer le cercle Γ′ qui est la translation du cercle Γ par un vecteur de meme direction
que d et de longueur a, puis de placer Y sur un des points d’intersection, de tracer la parallèle à d
passant par Y et de placer X à l’intersection de cette parallèle et de Γ.

47
2.2 Entrée

2.2.1 Arithmétique

Solution 94. Enoncé


Supposons que n est pair. Alors, il exsite un entier k tel que n = 2k. On a donc n2 +2n = 4k 2 +4k =
4(k 2 + k) = 4k ′ ou k ′ = k 2 + k est un entier. Ainsi, n2 + 2n est un multiple de 4.
Solution 95. Enoncé √ a
Supposons par l’absurde qu’on puisse ecrire 2 comme une fraction irreductible d’entiers . On
b
√ a a2
a alors 2 = , donc 2 = 2 , donc a = 2b , donc d’après l’exerice 2, a est pair. On peut donc
2 2
b b
l’ecrire comme 2a′ , avec a’ un entier. On a alors 2a′2 = b2 , donc b est pair, mais alors a et b sont
a
tous les deux pairs, ce qui constitue une absurdité puisqu’on a supposé la fraction irreductible
√ b
(on peut simplifier en haut et en bas par 2). Donc 2 est irrationnel.
Solution 96. Enoncé
a) Montrons par récurrence la proposition
n(n + 1)(2n + 1)
P (n) : ”12 + 22 + ... + n2 = ”
6
Initialisation : P (1) : ”1 = 1(1+1)(2×1+1)
6 ” est bien vraie.
Hérédité : Supposons P (n) vraie. On veut montrer qu’alors, P (n + 1) est vraie. Comme on sait que
n(n + 1)(2n + 1)
P (n) est vraie, on sait que 12 + 22 + ... + n2 = , donc 12 + 22 + ... + n2 + (n + 1)2 =
6
n(n + 1)(2n + 1) n(2n + 1) + 6(n + 1) (n + 1)(n + 2)(2n + 3)
+(n+1)2 = (n+1) = , donc P (n+1)
6 6 6
est vraie.
Conclusion : La proposition est intialisée et héréditaire, donc

n(n + 1)(2n + 1)
∀n ∈ N∗ , 12 + 22 + ... + n2 =
6
 2
n(n + 1)
b) Remarquons tout d’abord que pour tout entier naturel n, (1 + 2 + 3 + . . . + n) = 2
.
2
Montrons par récurrence la proposition
 2
3 3 3 n(n + 1)
P (n) : ”1 + 2 + ... + n = ”
2
 2
1(1 + 1)
Initialisation : P (1) : ”1 = ” est bien vraie.
2
Hérédité : Supposons P (n) vraie. On veut montrer qu’alors, P (n + 1) est vraie. Comme on sait que
 2
n(n + 1)
P (n) est vraie, on sait que 1 + 2 + ... + n =
3 3 3
, donc 13 + 23 + ... + n3 + (n + 1)3 =
2
2 2
n2 + 4(n + 1)
 
n(n + 1) (n + 1)(n + 2)
+ (n + 1)3 = (n + 1)2 = , donc P (n + 1) est vraie.
2 4 2
Conclusion : La proposition est intialisée et héréditaire, donc
 2
n(n + 1)
∀n ∈ N∗ , 13 + 23 + ... + n3 =
2

Solution 97. Enoncé


Soient a, b ∈ R deux réels. On considère la proposition suivante : "si a + b est irrationnel, alors a
ou b est irrationnel."
a) Si a et b sont rationnels, alors a + b est rationnel
p r
b) Supposons que a et b s’écrivent respectivement et , ou p, q, r, s sont des entiers et q et s sont
q s
ps + rq
non-nuls. Alors a + b = . Par contraposée, la proposition intiale est donc vraie.
√ √ qs
c) Non : 2 et − 2 sont tous les deux irrationnels mais leur somme (0) est rationnelle.

48
Solution 98. Enoncé
On cherche a montrer
P(n) : ”7 | 8n − 1”
Initialisation : Pour n = 0, on a bien 7 | 0
Hérédité : Supposons le resultat vrai pour un certain n ∈ N. Remarquons que 8n+1 − 1 = 8(8n −
1) + 7. Or, 7 | 8n − 1, donc 7 | 8n+1 − 1.
Conclusion : Le proposition est initialisee et hereditaire, donc d’apres le principe de recurrence,

∀n ∈ N, 7 | 8n − 1

NB : On pourvait egalement remarquer une factorisation qui nous donne le resulat : 8n − 1 =


7 × (8n−1 + . . . + 8 + 1)
Solution 99. Enoncé
a) On a an2 + bn + c = 0 ⇐⇒ c = −n(an + b), donc il faut n | c
b) On applique le résulat precedent : on a n | 3 donc n ∈ {−3, −1, 1, 3}. Réciproquement, seuls −1
et 3 conviennent.
S = {−1, 3}
Solution 100. Enoncé
On regarde les restes possibles pour la division euclidienne par 4 d’un carré parfait.
Soit n un entier. Si n est pair, on peut l’écrire 2k avec k un entier. Alors n2 = 4 × k 2 + 0. Si n est
impair, alors on peut l’écrire 2k + 1 avec k un entier. Alors n2 = 4(k 2 + k) + 1. Les seuls restes
possibles d’un carré par la division euclidienne par 4 sont donc 0 et 1. La somme de deux carrés
impairs a un reste de 2 par la division euclidienne par 4, donc ca ne peut pas etre un carré parfait.
Solution 101. Enoncé
Les diviseurs de n sont les nombres de la forme pβ1 1 × . . . × pβkk , avec ∀i ∈ [[1, k]], βi ∈ [[0, αk ]], ce qui
nous fait, grace à l’unicité de la décomposition en produit de facteurs premiers,

(α1 + 1) × . . . × (αk + 1)

Solution 102. Enoncé


On sait que toute solution de cette équation doit diviser le terme constant, ie il doit diviser 42.
Les diviseurs de 42 sont {−42, −21, −14, −7, −6, −3, −2, −1, 1, 2, 3, 6, 7, 14, 21, 42}. On remarque
immédiatement que 1 est solution, puis que 2 est solution. On peut alors factoriser le polynome,
ou bien juste tester les diviseurs restants. On trouve finalement :

S = {−7, −3, 1, 2}

Solution 103. Enoncé


On remarque que 212 ≡ 312 ≡ 1 (mod 13), il nous suffit alors de calculer 210 + 310 , ce que l’on
peut calculer à la main modulo 13. Ce nombre est donc effectivement un multiple de 13.
Solution 104. Enoncé
On regarde modulo 4. Les carrés modulo 4 peuvent valoir 0 ou 1, et le membre de gauche vaut 3
modulo 4 si n ≥ 2, donc s’il y a une solution, c’est pour n = 0 ou n = 1. n = 0 ne donne clairement
pas de solution. n = 1 donne les solutions (a, b) = (3, 2) et (a, b) = (2, 3)
Solution 105. Enoncé
9 et 7 sont premiers entre eux, donc avec l’algorithme d’Euclide, on trouve que le couple de Bézout
(u0 , v0 ) = (−3, 4) donc l’ensemble des couples solution (u, v) sont les (−3 + 7k, 4 + 9k), k ∈ Z
Solution 106. Enoncé
Notons 2n + 1 = x2 et 3n + 1 = y 2 . Alors n = y 2 − x2 = (y − x)(y + x). Un carré impair est congru
à 1 modulo 4, donc n est pair, donc y − x ou x + y est pair, donc les deux sont pairs, donc au moins
un des deux est divisible par 4, donc n est divisible par 8. Les carrés modulo. Les carrés modulo 5
valent 0, 1 ou 4. On peut donc distinguer des cas :
2n + 1 ≡ 3n + 1 (mod 5) : en faisant la différence, on a n ≡ 0 (mod 5), ce qui conclut.
Si 2n + 1 ou 3n + 1 sont congrus à 1 modulo 5, alors 2n ou 3n est divisible par 5, dans tous les cas
5 | n, ce qui conclut.
Si 2n + 1 ≡ 0 (mod 5) et 3n + 1 ≡ 4 (mod 5), alors n ≡ 2 (mod 5) et n ≡ 1 (mod 5), absurde.
Si 2n + 1 ≡ 4 (mod 4) et 3n + 1 ≡ 0 (mod 5), alors n ≡ 4 (mod 5) et n ≡ 3 (mod 5), absurde.
Donc on a bien 40 | n

49
Solution 107. Enoncé
Pour tous les critères, on écrit n ∈ N avec son écriture décimale (n = a0 + 10a1 + 100a2 + . . .) et
on regarde cette expression modulo l’entier dont on veut le critère de divisibilité.
Solution 108. Enoncé
9
On remarque que 74 ≡ 1 (mod 10), donc 73 ≡ 74k+3 ≡ 3 (mod 10) car 39 ≡ 3 (mod 4), donc le
chiffre des unités est 3
Solution 109. Enoncé
D’après le théorème de Bézout, les inversibles sont exactement les nombres premiers avec 8, donc
1, 3, 5, 7

Solution 110. Enoncé


On raisonne par contraposée. Si p n’était pas premier, on pourrait écrire p = ab, avec 1 < a, b < p,
donc a et b apparaiteraient tous les deux dans (p − 1)!, donc on aurait (p − 1)! ≡ 0 (mod p). Si a
et b ne sont pas distincts, on a juste à remarquer que 2a ou 2b est un facteur dans (p − 1)!.

Solution 111. Enoncé


D’apres le theoreme de Bézout, n et m sont premiers entre eux SSI ∃u, v ∈ Z tels que mu + nv = 1
ie SSI ∃q ∈ Z tel que qm ≡ mq ≡ 1 (mod n).
Source : Existence d’un inverse modulo n

Solution 112. Enoncé


Pour l’unicite, on suppose qu’on a deux inverses de a : q et q ′ Alors q ≡ q(aq ′ ) ≡ (qa)q ′ ≡ q ′
(mod n). Donc l’inverse est bien unique modulo n.
Source : Unicité d’un inverse modulo n
Solution 113. Enoncé
Pour la première, on remarque que 2 × 8 ≡ 1 (mod 15) donc a ≡ 56 ≡ 11 (mod 15)
Pour la deuxième, on remarque qu’il faut qu’il existe k ∈ Z tel que 3a = 15k + 8, mais alors 3
divise 3a − 15k mais pas 8, absurde.
Solution 114. Enoncé
La réciproque est démontrée est plus haut. Le sens direct est le Problème Mathraining #3296
Source : Théorème de Bézout

Solution 115. Enoncé


On remarque que si un nombre est un carré et est divisible par 3, alors il est divisible par 9 (on
peut s’en convaincre rapidement par contraposée). Or en faisant la somme des chiffres, on voit
que le nombre considéré est effectivement divisible par 3 mais pas par 9 (critères de divisibilité
classiques) donc il ne peut pas être un carré.
Source : Coupe Animath d’automne 2019

50
2.2.2 Algèbre

Solution 116. Enoncé


On applique l’inégalité arithmético-géométrique, ce qui conclut immédiatement. On a alors égalité
SSI x = 1
Solution 117. Enoncé
On applique l’inégalité arithmético-géométrique, ce qui conclut immédiatement. On a égalité SSI
x2 = y 2 ie x = y.
Solution 118. Enoncé
On applique l’inégalité arithmético-géométrique à 4x2 + y 2 , puis a x2 + 1, ce qui donne le résultat
voulu en sommant les inégalités trouvées.
Solution 119. Enoncé
On applique l’inégalité arithmético-géométrique, ce qui conclut immédiatement.
Solution 120. Enoncé
(a + 1) + (b + 1) + (c + 1)
On a d’apres l’inégalité arthmético-géométrique,
p
≥ 3 (a + 1)(b + 1)(c + 1) =
3
2, donc on a en réarrangeant, a + b + c ≥ 3
Solution 121. Enoncé
On procède par récurrence double : √ √
1+ 5
− 1− 5
Initialisation : F0 = 0 = 1−1
√ , F1 =
5
2 √
5
2
=1
Hérédité : Soit n ∈ N∗ . Supposons le résultat vrai pour n − 1 et n. Alors :

Fn+1 = Fn + Fn−1
√ √ √ √
( 1+2 5 )n − ( 1−2 5 )n ( 1+2 5 )n−1 − ( 1−2 5 )n−1
= √ + √
5 5
√ !n−1 √ √ !n−1 √
 ! !
1  1+ 5 1+ 5 1− 5 1− 5
=√ +1 − +1 
5 2 2 2 2
√ !n+1 √ !n+1
 
1  1+ 5 1− 5
=√ − 
5 2 2

√ √
Car 1+2 5 et 1−2 5 vérifient x2 = x + 1
Conclusion : La proposition est initialisée et héréditaire donc vraie pour tout entier naturel.
Source : Formule de Binet
Solution 122. Enoncé
Remarquons que si (un ) est periodique de periode T , alors par récurrence immédiate, pour tout
entier naturel k, et tout entier naturel n, un = un+kT .
Alors soit n un entier naturel, on a un+1 = un+1+2×2022 = un+4045 = un + 5 × 809 = un . Donc
un+1 = un ie la suite est constante.
Solution 123. Enoncé
Montrons par récurrence la proposition

P(n) : ”Fn+1 × Fn−1 − Fn2 = (−1)n+1 ”

Initialisation : On a F2 × F0 − F12 = 2 × 1 − 1 = (−1)1+1 , donc P (1) est vraie.


Hérédité : On suppose que P(n) est vraie pour un certain n. On veut montrer que P(n + 1) est
vraie. On a :
2 2
Fn+2 × Fn − Fn+1 = (Fn+1 + Fn ) × Fn − Fn+1
= Fn+1 Fn + Fn2 − Fn+1
2

= Fn+1 (Fn − Fn+1 ) + Fn2


= −(Fn+1 (Fn+1 − Fn ) − Fn2 )
= −(Fn+1 Fn−1 − Fn2 ) = (−1) × (−1)n+1 = (−1)n+2

51
Donc P(n + 1) est vraie (On a bien Fn+1 − Fn = Fn−1 puisque Fn+1 = Fn−1 + Fn )
Conclusion : La proposition est initialisee et hereditaire, donc d’après le princpe de récurrence,

∀n ∈ N∗ , Fn+1 × Fn−1 − Fn2 = (−1)n+1

Solution 124. Enoncé  


Posons vn = un − 1−a
b
. Alors vn+1 = un+1 − 1−ab
= aun + b − b
1−a = a un − b
1−a = avn , donc
 
  −b b
−b
vn = v0 × an = an 1−a , donc un = an +
1−a 1−a
Source : Terme général d’une suite arithmético-géométrique
Solution 125. Enoncé
Analyse : Posons y = 0. Alors pour tout x non nul, f (x) = ax + f (0). Si on note f (0) = b, on a
alors f (x) = ax + b.
Syntèse : Réciproquement, on vérifie bien que toutes les fonctions affines conviennent.

Solution 126. Enoncé


Analyse : On pose x = f (y), on trouve alors f (0) = 1 − f (y) − y, donc f (y) = 1 − f (0) − y. Or,
f (0) est une constante, donc 1 − f (0) =: c aussi. On a alors ∀y ∈ R, f (y) = c − y
Synthèse : Réciproquement, si f (y) = c − y, alors f (x − f (y)) = f (x − (c − y)) = c − (x + y − c) =
2c − x − y = 1 − x − y, donc 2c = 1, donc c = 21 , donc la seule solution est

1
f (x) = −x
2

Source : Envoi d’algèbre 2021


Solution 127. Enoncé
Montrons que la seule solution est f (x) = x. Par l’absurde, supposons qu’il existe x ∈ R tel que
f (x) < x, alors en composant par f , on a x < f (x), absurde. Le cas x < f (x) se traite de meme.
Si maintenant f est croissante, alors soit x un réel. Si x ≤ f (x), alors en composant par x, on a
f (x) ≤ x ≤ f (x), donc x = f (x). Si maintenant x ≥ f (x), on applique le meme raisonnement et
on trouve que pour tout réel, f (x) = x

Solution 128. Enoncé


Analyse : On pose x = 0. On a alors f (y) = f (0) + y
Synthèse : Réciproquement, si on note a := f (0), alors on vérifie que toute fonction de la forme
f (x) = x + a convient.
Solution 129. Enoncé
Remarquons que pour tout x ∈ R,

P (x) = P (x) − P (α)


Xn Xn
= ak xk − ak αk
k=0 k=0
Xn
= ak (xk − αk )
k=1
n k−1
!
X X
l n−1−α
= ak (x − α) xα
k=1 l=0

n k−1
!
X X
l n−1−α
P (x) = (x − a) ak xα
k=1 l=0
| {z }
:=Q(x)

Donc, si un polynome de degré n pouvait avoir n + 1 racines, on pourrait le factoriser par ces n + 1
racines et on aurait un produit de n + 1 polynomes de degré 1, donc un polynome de degré n + 1,
absurde.

52
Solution 130. Enoncé
Si P est non-nul et ∀x ∈ R, P (x) = 0, alors si on note n le degré de P , P possède n + 1 racines
distinctes puisque tous les réels sont des racines de P . D’apres l’exercice précédent, c’est impossible.
Donc P est le polynome nul.
Notons R = P − Q. ∀x ∈ R, R(x) = 0, donc d’apres la question précédente, R = 0 donc P = Q.
Avec ce raisonnement, il suffit en fait que les deux polynomes coincident en max(deg P, deg Q) + 1
points.
Solution 131. Enoncé
Pour la première équation, on regarde les degrés. Alors 2 deg(P ) = 1 + 2 deg(Q), mais alors un
nombre pair serait égal à un nombre impair, donc il n’exsite pas de solution.
Pour la deuxième équation, on remarque en passant au degré que 2 deg P = 2 + deg P donc P est
de degré 2. Notons P (x) = ax2 + bx + c, a ̸= 0. Alors ax4 + bx2 + c = ax4 + bx3 + cx2 , donc d’après
l’exercice précédent, on a b = 0 et b = c donc P (x) = ax2 . Réciproquement, tous les polynomes de
la forme ax2 conviennent (le polynome nul convient également).
Solution 132. Enoncé
Supposons P non nul. Alors si on note a le coefficient dominant de P , et n son degré, on a d’apres
l’exercice 6, a = 2n a, donc n = 0. Alors P doit etre un polynome constant. Réciproquement, tous
les polynomes constants (meme le polynome nul) conviennent.
Solution√ 133.√Enoncé
1) z1 = 2 + i 2
−1
2) z2 = 5ei tan (3/4)
π
3) z3 = ei 2
4) z4 = −1

Solution 134. Enoncé √ π


On passe sous forme exponentielle : (1 + i)20 = ( 2ei 4 )20 = 1024e5iπ = −1024
Solution 135. Enoncé
√ iθ
Si z = reiθ , on constate aisément que re 2 convient.
Si z = a + ib, on note une solution potentielle y = c + id. Alors on a y 2 = c2 − d2 + 2icd
donc on a√a = c2 − d2 (1) et b = 2cd (2). De plus, on peut remarquer que |y|2 = |z| donc
c2 + d2 = a2 + b2 (3).
En combinant (1) et (3), on trouve que
 q√
 c=± a2 +b2 +a
q√ 2
 d=± a2 +b2 −a
2

Il ne nous reste alors qu’à prendre en compte le signe de b : si b est positif, alors c et d sont de
meme signe, et si b est négatif, ils sont de signes opposés. Finalement, on trouve
√ s√
s 
a2 + b2 + a a 2 + b2 − a
y = ± + sgn(b) · i 
2 2

Solution 136. Enoncé



On a d’après l’inégalité arithmético-géométrique appliquée à 2 variables, x + y ≥ 2 xy, y + z ≥
√ √
2 yz et x + z ≥ 2 xz. En les multipliant entre eux, on a bien (x + y)(y + z)(x + z) ≥ 8xyz

Solution 137. Enoncé


On développe : (1 + x) 1 + x1 = 2 + x + 1
≥ 4 car on a bien x + 1
≥ 2 d’après l’IAG.

x x

Solution 138. Enoncé


1 1
D’après l’IAG, on a x4 +y 2 ≥ 2x2 y et y 4 +x2 ≥ 2y 2 x, donc par passage à l’inverse, ≤ 2
x4 + y 2 2x y
1 1
et ≤ , donc en multpliant la première inégalité par x et la deuxième par y, et en
y4
+y 2 2y 2 x
sommant les deux, on a :

x y 1 1 1
+ 4 ≤ + =
x4 +y 2 y +x 2 2xy 2xy xy

53
Solution 139. Enoncé
On a, d’apres l’inégalité arithmético-géométrique :

a3 + b3 + c3
≥ abc
3
a3 + b3 + d3
≥ abd
3
a3 + c3 + d3
≥ acd
3
b3 + c3 + d3
≥ bcd
3
Donc en sommant le tout :

a3 + b3 + c3 + d3 ≥ abc + abd + acd + bcd

Solution 140. Enoncé


On a z + z = 2ℜ(z). Si on note θ l’argument de z, on a |z| = 2|z| cos(θ). Alors on veut cos(θ) = 1
2
donc θ = ± π3 [2π]. Reciproquement, tous les nombres complexes d’argument ± π3 conviennent.

54
2.2.3 Combinatoire

Solution 141. Enoncé


Considérons l’ensemble des valeurs sur ces points. C’est un ensemble d’entiers naturels, donc il
possède un minimum. Considérons un point dont la valeur est ce minimum, alors ses quatre voisins
doivent également prendre cette meme valeur puisqu’ils sont tous les quatre plus grands que le
minimum mais somment a quatre fois le minimum. De proche en proche, tous les points prennent
la meme valeur.

Solution 142. Enoncé


Pour que le segment joignant deux points à coordonnées entières passe par un autre point à
coordonnées entières, il faut les deux coordonnées des deux points soient de meme parité. Par le
principe des tiroirs, en choisissant 5 points, on en a 2 qui ont leurs coordonnées de meme parité,
ce qui conclut

Solution 143. Enoncé


Considérons un groupe de n personnes. On veut déterminer le nombre de facons de choisir une
équipe de k personnes et de choisir un chef parmi elles. On a deux facons de faire cela : Soit on
constitue d’abord le groupe puis on choisit un chef : on a nk de choisir le groupe, puis k facons
de choisir le chef, soit on choisit d’abord de chef, puis les k − 1 personnes manquantes de l’équipe
parmi les n − 1 restantes. On a donc :
   
n−1 n
n =k
k−1 k

Solution 144. Enoncé


Pour aller en (6, 8), il faut effectuer 6 + 8 = 14 pas. Parmi ces 14, il faut caser les 8 qui vont vers
le haut, donc il y a :
 
14
= 3003
8

chemins possibles. On laisse au lecteur le soin de généraliser cette solution pour aller en (n, m)
Solution 145. Enoncé
Imaginons que les 5 pièces sont alignées, et supposons qu’on a 2 barres qui serviront à délimiter
les sacs : toutes les pièces a gauche de la première barre vont dans le premier sac, toutes les pièces
entre la 1re et la 2e barre vont dans le 2e sac, et les pièces a droite de la 2e barre vont dans le 3e
sac. On a donc 7 places qu’il faut remplir avec les deux barres et les 5 pièces. On a donc :
 
7
= 21
2

possibilités.
Solution 146. Enoncé
Considérons une assemblée de n personnes. On veut connaitre le nombre de facons de choisir un
sous-groupe de ces personnes. D’un coté, pour chaque personne, on peut choisir de la prendre ou
de ne pas la prendre, donc il y a 2n possibilités. D’un autre coté, on peut soit prendre une équipe
de 0 personnes, ce qui représente n0 possibilités, ou bien une personne, soit n1 possibilités, soit


finalement :
n  
X n
= 2n
k
k=0

Solution 147. Enoncé


On étudie les manières de remplir le coin en haut à gauche de la grille. Il y en a 3. On remarque
cependant que seules deux d’entre elles permettent également de remplir le coin en bas à gauche.
Parmi ces deux possibilités, on remarque qu’il n’y a qu’une seule facon d’ajouter une nouvelle pièce
sans laisser de trou que l’on ne pourrait pas remplir. Mais alors on a rempli une sous-grille de 2 × 3,
donc il nous reste une grille de 2 × 3(n − 1), et donc en appliquant le meme raisonnement (ou en
faisant une récurrence), on trouve qu’on a un total de 2n possibilités.

55
Solution 148. Enoncé
Notons Fn le nombre de manières de remplir une grille 2 × n avec des dominos 1 × 2. On a F0 = 1
et F1 = 1. On remarque maintenant que si on connait Fn et Fn−1 , alors on connait Fn+1 . En
effet, pour remplir une grille 2 × (n + 1), on a deux possibilités : soit on remplit la grille jusqu’a
n − 1, et on rajoute deux dominos horizontaux, soit on remplit la grille jusqu’a n, et on rejoute
un domino vertical (on notera bien qu’il ne faut pas compter 2 fois le cas n − 1, puisque ajouter
deux dominos verticaux revient à remplir jusqu’à n, et à completer avec un domino vertical). Donc
Fn+1 = Fn + Fn−1 . On retrouve la suite de Fibonacci.
Solution 149. Enoncé
On remarque que si tous les ai valent 1, alors S = n. Montrons maintenant que S est invariant
modulo 4 : si on change le signe d’un des ai , alors 4 des quantités de la somme vont changer de
signe, donc la différence entre la valeur initiale de S et la valeur finale de S va etre divisible par 4,
ce qui signifie que S n’a pas changé modulo 4. Ainsi, on doit avoir n ≡ 2 (mod 4), ie 4 | n

Solution 150. Enoncé


On procède par récurrence. Si on a deux villes c’est évidemment vrai. Supposons que ce soit vrai
pour n villes et considérons n + 1 villes. On choisit une ville et on la met de coté. On applique
l’hypothèse de récurrence sur les n villes restantes : il exsite un moyen de transport (SPDG bateau)
permettant de relier toute paire de villes. Si la nouvelle ville est reliée à une des n premières par
une voie navigable, on a fini, sinon, l’avion convient : il suffit de passer par la n + 1-eme ville pour
avoir accès à toutes les autres villes par avion.
Solution 151. Enoncé
D’après le principe des tiroirs, il y avait le matin une rangée de 8 élèves. Donc d’après le principe
des tiroirs, parmi ces 8 élèves, il y en a deux qui se sont retrouvés sur la meme rangée l’après-midi.

Solution 152. Enoncé


On colorie la grille comme un échiquier. A chaque déplacement, on change de couleur de case. De
plus, les cases d’arrivée et de départ sont de la meme couleur, donc il faudrait effectuer un nombre
pair de déplacements. Or, il y a 20182 −1 cases à traverser, ce qui est impair. C’est donc impossible.
Solution 153. Enoncé
On considère l’enveloppe convexe de ces points et on la triangule. Par l’absurde, si un des points se
situe strictement à l’interieur de l’enveloppe convexe, alors ce point appartient à l’un des triangles.
Le quadrilatère constitué des trois points du triangle et du point à l’interieur n’est pas convexe, ce
qui constitue une absurdité.
Solution 154. Enoncé
On instroduit un axe des abscisses tels que tous les points aient une abscisse distincte (ce qui est
bien possible : on a une infinité de directions mais un nombre fini de points). On note A le point
d’abscisse minimal de P et Z le point d’abscisse la plus grande. Pour tous les sommets de P de
l’arc superieur entre A et Z, on prend un point "juste en dessous" et pour tous les points de l’arc
inferieur, on prend un point "juste au dessus" (au sens orthogonal).
On laisse au lecteur le soin de vérifier que l’ensemble des points considérés convient.

Solution 155. Enoncé


On va remarquer que les deux côtés de cette égalité dénombrent le nombre de façons de choisir,
dans un groupe de n personnes, une équipe (de taille arbitraire) et un capitaine. Dans le membre
de gauche, on choisit le capitaine (n choix), puis pour les n − 1 personnes retantes, on peut décider,
pour chaque personne, soit de la prendre dans l’équipe, soit de ne pas la prendre, on a donc 2n−1
possibilités. Dans le membre de gauche, on dénombre les équipes en fonction de leur nombre de
membres  : si on veut une équipe de k personnes dont un capitaine dans un groupe de n personnes,
on a nk possibilités pour l’équipe, et k possibilités pour le capitaine. Finalement, on somme sur
toutes les valeurs possibles de k pour avoir la formule finale.
Solution 156. Enoncé
Montrons d’abord que P ≤ G. On a pour tous l, m ∈ [[1, n]], xl ≤ max et ym ≥ min , donc
1≤i≤n 1≤j≤n
xl − ym ≤ G. En particulier, en prenant l = m, on a bien P ≤ G
Montrons maintenant que G ≤ 2P . Notons xj = max et yk = min . On a de plus par hypothese
1≤i≤n 1≤i≤n
xk ≥ yj . Alors G = xj − yk ≤ xj − yj + xk − yk ≤ 2P
Source : Coupe Animath de printemps 2020

56
2.2.4 Géométrie

Solution 157. Enoncé


a) Soit O le centre de ω. Les triangles OAT et OBT sont rectangles en A et en B respectivement.
Le théorème de Pythagore permet de conclure, en remarquant que OA = OB.
b) Pour la cocyclicité des points, il suffit de remarquer que OAT
[ = 180 − OBT
[ = 90.

Solution 158. Enoncé


Notons C le point diamétralement opposé à A dans Γ1 . D’après le théorème de l’angle au centre,
on a AOB
\ = 2ACB. \ Par symétrie du problème, on a donc ACB \ = AO \ 1 O2 , donc les droites (CB)
et (O1 O2 ) sont parallèles. Or, [AC] est un diamètre de Γ1 , donc ABC est rectangle en B, donc les
droites (AB) et (O1 O2 ) sont perpendiculaires.

Solution 159. Enoncé


Comme les droites (AD), (BE) et (CF ) sont des hauteurs, on a ABD
\ = BEA\ = CF[ A = 90.
a) La réciproque du théorème de l’angle inscrit appliqué à ADB
\ = BEA \ permet de conclure
b) Comme HEA\ = 180 − HF \ A = 90, le quadrilatère est bien cyclique.

57
Solution 160. Enoncé
Comme ABF D est cyclique, on a DF
\ \ donc DF
B = 180 − DAB, \ \ Or, ABEC est
B = CAB.
cyclique, donc DF
\ \ Donc les droites (CE) et (DF ) sont parallèles.
\ = 180 − CEB.
B = CAB

Solution 161. Enoncé


On notera qu’un quadrilatère circonscriptible est un quadrilatère qui admet un cercle inscrit, ce qui
est différent d’un quadrilatère inscriptible qui lui admet un cercle circonscrit
On considère le cercle inscrit du quadrilatère. On a les 4 égalités indiquées sur la figure. On a alors

AB + CD = x + w + z + y = BC + AD

Solution 162. Enoncé


Comme la somme des angles dans un triangles vaut 180, on a BAC
\ = 180 − 42 − 63 = 75, donc
d’après le théorème de l’angle à la tangente, on a DBC \ = 75, donc
\ = DCB

\ = 180 − 75 − 75 = 30
BDC
Solution 163. Enoncé
D’après le théorème de l’angle au centre, XO
\ 1 Y = 2XAY , donc par symétrie du problème,
\
XO
\ O
1 2 = XAY
\ . De meme, XO
\ O
2 1 = XBY
\ . Donc les triangles XAB et XO1 O2 sont semblables.

58
Source : Premier théorème de Miquel
Solution 164. Enoncé
On note D le point d’intersection des cercles circonscrits à AQR et BP R. Les quadrilatères AQDR
et BP DR sont cycliques donc RDQ\ = 180 − BAC \ et RDP \ Donc QDP
\ = 180 − ABC. \ = 360 −
QDR − RDP = BAC + ABC = 180 − ACB car la somme des angles d’un triangle vaut 180. Donc
\ \ \ \ \
le quadrilatère P DQC est cyclique, ie le cercle circonscrit à CQP passe par D, ie les trois cercles
sont bien concourants.

Source : Second théorème de Miquel


Solution 165. Enoncé
1) Notons I le point d’intersection de deux des bissectrices. Il faut remarquer qu’une bissectrice
est un ensemble de points équidistants de deux droites. Alors I est équidistant des trois cotés du
triangle, donc il appartient à la 3e bissectrice. En particulier, les trois bissectrices d’un triangle
sont concourantes en I, qui est équidistant des cotés du triangle, et qui est donc le centre du cercle
inscrit.
2) Pour les médiatrices, on procède de meme en remarquant que les médiatrices sont l’ensemble
des points équidistants des extrémités d’un segment. Leur point d’intersection est donc équidistant
des 3 sommets du triangle : c’est le centre du cercle circonscrit.
Solution 166. Enoncé
Soit ABC un triangle. On trace pour chaque coté du triangle, la parallèle au coté passant par le
sommet opposé. On note A′ le point d’intersection des parallèles opposé à A, et on définit B ′ et C ′
de la meme manière. L’astuce ici est de remarquer que les triangles ABC, A′ BC, AB ′ C et ABC ′
sont tous isométriques, et donc que les hauteurs du triangle ABC sont les médiatrices du triangle
A′ B ′ C ′ , donc elles sont bien concourantes.

59
Solution 167. Enoncé
Pour la première égalité, il suffit d’appliquer l’exercice 87 dans le cas ou B et B ′ sont confondus.
Pour la deuxième, c’est le théorème de Pythagore, appliqué en remarquand que la tangente est
perpendiculaire au rayon.

Solution 168. Enoncé


a) Il suffit de choisir un point de la droite initiale, dont l’image est alors fixée, et d’appliquer la
réciproque du théorème de Thalès à tout autre point de la droite.
b) On considère un segment de longueur a. Alors d’après le point précédent, son image est un
segment parallèle au premier. Il suffit alors d’appliquer le théorème de Thalès pour avoir le rapport
de longueurs voulu.
c) Les angles sont conservés par le parallèlisme. Les droites sont conservées d’après le point a) et
les cercles sont conservés puisque les angles sont conservés (critères de cocyclicité).
Solution 169. Enoncé
a) Pour le rapport de l’homothétie, il suffit de remarquer que si on a un segment de longueur l,
son image par h1 est de longueur k1 l, et donc l’image par h2 de cette image est de longueur k2 k1 l,
donc le rapport de l’homothétie est bien k1 k2 . Pour le centre de l’homothétie, il faut remarquer si
on a une homothétie h de centre O, alors pour tout point A d’image A′ , O, A et A′ sont alignés,
donc dans notre cas, le centre est sur la droite contenant O1 et l’image de O1 par l’homothétie
h2 ◦ h1 . Or, l’image de O1 est h2 (h1 (O1 )) = h2 (O1 ) car h1 est de centre O1 , donc l’image de O1
est sur la droite (O1 O2 ), donc le centre de l’homothétie aussi.
b) Si k1 k2 = 1, l’image d’un segment lui est toujours parallèle mais est également de la meme
longueur : c’est une translation. C’est bien le meme vecteur pour tout point : il suffit de fixer l’une
des extrémités du segment et de faire varier l’autre n’importe ou dans le plan.
Solution 170. Enoncé
On en montre un sens dans l’exercice 168. Pour l’autre, si AB ̸= A′ B ′ , il suffit de prendre le point
d’intersection de AA′ et BB ′ comme centre de l’homothétie : le théorème de Thalès nous affirme
qu’elle existe bien. Si les deux longueurs sont égales cependant, il ya évidemment une translation
qui envoie l’un sur l’autre.
Solution 171. Enoncé
On note A′ , B ′ et C ′ les milieux respectifs des segments [BC], [AC] et [AB]. On remarque (théo-
rème des milieux) que les droites (B ′ C ′ ) (resp. (A′ C ′ ), (A′ B ′ )) et (BC) (resp. (AC), (AB)) sont
parallèles. Il exsite donc une homothétie de rapport − 21 qui envoie C sur C ′ et B sur B ′ , donc le
point d’intersection des médianes issues de B et C se situe aux deux tiers des segments, comme
voulu. De meme pour les autres couples de médianes, et comme il n’existe qu’un unique point au
deux-tiers de [CC ′ ], les trois médianes se coupent bien en ce point.

60
2.3 Plat

2.3.1 Arithmétique

Solution 172. Enoncé


Montrons par récurrence d’ordre 2
1
P (n) : ”an + ∈ Z”
an
1
Initialisation : P (0) : a0 + = 2 ∈ Z et P (1) est vraie par hypothese de l’énoncé.
a0
Hérédité : Supposons P (n) et P (n − 1) vraies. On veut montrer que P (n + 1) est vraie. On sait
1 1 1 1
que (an + n ) ∈ Z et a + ∈ Z donc leur produit est aussi dans Z, soit (an + n )(a + ) =
a a a a
1 1
an+1 + an−1 + n−1 + n+1 ∈ Z. Or, on suppose egalement P (n − 1) vraie, donc an+1 + an−1 +
a a 
1 1 1 1
+ − a n−1
+ = an+1 + n+1 ∈ Z, donc P (n + 1) est vraie.
an−1 an+1 an−1 a
Conclusion : La proposition est intialisée et héréditaire, donc

1
∀n ∈ N∗ , an + ∈Z
an

NB : On utilise ici une version un peu differente de la demonstration par récurrence : on appelle
récurrence d’ordre n une récurrence ou on utilise les n dernieres propositions pour montrer la
suivante. C’est tout a fait la meme chose que l’image de l’escalier : Si de toute paire de marches
consécutives, je peux monter sur la suivante et que je peux monter sur les deux premieres marches,
alors je peux monter tout l’escalier.
Solution 173. Enoncé
Supposons par l’absurde que n et 2n sont des carrés d’entiers. Alors il existe p, q ∈ N tels que
2n q2 √ q √
n = p2 et 2n = q 2 , mais alors 2 = = 2 , donc 2 = , ce qui constitue une absurdité car 2
n p p
n’est pas rationnel.
Solution 174. Enoncé
Supposons par l’absurde qu’il existe un nombre fini de nombres premiers. Notons les p1 , . . . , pn .
Alors l’entier p1 × . . . × pn + 1 est plus grand que tous ces nombres, mais aucun des nombres
premiers listés ne le divise (sinon, il devrait aussi diviser 1, mais tous les nombres premiers sont
strictement plus grands que 1, donc c’est impossible.), on a donc une contradiction, donc il doit
exister un nombre infini de nombres premiers.
Source : Euclide
Solution 175. Enoncé
a) Un dévelopement du membre de gauche donne le résultat voulu.
b) On peut réécrire 4545 + 5454 = 5454 + 4 × (4136 )4 . La facorisation précédente montre que ce
nombre n’est pas premier.
c) On écarte le cas n = 1. Si n est pair, le résultat est évident. Sinon, on peut écrire n = 2k+1, k ∈ Z.
Alors on a
n4 + 4n = (2k + 1)4 + 4 × (2k )4
= ((2k + 1)2 + 2 × 22k + 2 × (2k + 1) × 2k )((2k + 1)2 + 2 × 22k − 2 × (2k + 1) × 2k )

On vérifiera aisément que les deux facteurs sont bien strictement superieurs à 1
Source : Factorisation de Sophie-Germain
Solution 176. Enoncé
Problème Mathraining #3246
Solution 177. Enoncé
Par l’absurde, supposons qu’il en existe un nombre fini k ∈ N. Notons les p1 , . . . , pk . Considérons
le nombre n = 4p1 × . . . × pk − 1. Alors N n’est divisible ni par 2 ni par un des pi . C’est donc le

61
produit de nombres premiers distincts des pi et de 2. Or, un produit de nombres premiers de la
forme 4m + 1 est un nombre de la forme 4m′ + 1, donc au moins un des divsieurs de N est de la
forme 4l + 3, et n’apparait pas dans la liste des pi , ce qui est absurde. Il y a donc une infinité de
nombres premiers de la forme 4k + 1.

Solution 178. Enoncé


Pour n = 0, il n’y a pas de solution. Pour n = 1, m = 1 convient. Si n ≥ 2, on a 3m − 2n ≡ 3m ≡ 1
(mod 4) donc m doit etre pair. Posons m = 2k, alors 32k −1 = 2n ie (3k −1)(3k +1) = 2n , mais pour
qu’un produit d’entiers soit une puissance de 2, il faut que chacun des facteurs soit une puissance
de 2. Or, les seules puissances de 2 séparées de 2 sont 2 et 4, donc la seule solution possible est
k = 1 et n = 2, donc
S = {(1, 1), (2, 3)}

Solution 179. Enoncé


Considerons l’ensemble des inversibles modulo n, ie l’ensemble des nombres premiers avec n. Il
y en a exactement φ(n) par definition. Notons les {a1 , . . . , aφ(n) }. Soit a premier avec n. Alors
l’ensemble {aa1 , . . . , aaφ(n) } est une permutation de {a1 . . . aφ(n) } modulo n. En effet, si on note
a−1 l’inverse de a modulo n, alors pour tout k ∈ [[1, φ(n)]], aai ≡ ak (mod n) ⇐⇒ ai ≡ a−1 ak
(mod n).
Donc le produit des elements d’un des ensembles est congru au produit des elements de l’autre, ie
a1 × . . . × aφ(n) ≡ aa1 × . . . × aaφ(n) (mod n). Tous les ai étant inversibles, on multiplie des deux
cotes par tous les inverses et on trouve aφ(n) ≡ 1 (mod n)
Source : Théorème d’Euler-Fermat
Solution 180. Enoncé
Problème Mathraining #3151

Solution 181. Enoncé


Modulo 9, on a 44444444 ≡ 74444 . Or, 73 ≡ 1 (mod 9), donc A ≡ B ≡ C ≡ D ≡ 44444444 ≡ 7
(mod 9).
On va maintenant majorer brutalement les quantités : A ≤ 100004444 = 105×4444 ≤ 1025000 donc
B ≤ 9 × 25000 = 225000 donc C ≤ 2 + 9 × 5 = 47 donc D ≤ 4 + 9 = 13. Or, le seul entier entre 0
et 13 congru à 7 modulo 9 est 7 donc D = 7
Source : IMO 1975
Solution 182. Enoncé
Si z ≥ 2, on regarde modulo 4 et on a x2 + y 3 ≡ 3 (mod 4), mais la somme de deux carrés n’est
jamais congrue à 3 modulo 4. Les seules solutions sont donc z = 0, x = y = 0 et z = 1, x = 2,
y = 1. Donc
S = {(0, 0, 0), (2, 1, 1)}

Solution 183. Enoncé


On a 2n = 7 + 3m ≥ 8, donc n ≥ 3. On regarde modulo 8 : 3m ≡ 1 (mod 8), donc m doit etre
pair. Si m = 0, on a la solution (3, 0), on suppose alors m ≥ 1. De meme, en regardant modulo 3,
on trouve n pair, donc en notant n = 2k, m = 2l, on a (2k − 3l )(2k + 3l ) = 7. Or, 7 est premier,
et 2k + 3l est positif, donc on a finalement

S = {(3, 0), (4, 2)}

Réciproquement, ces couples conviennent bien.


Source : Valbonne 2020, entrainement de mi-parcours, groupe B
Solution 184. Enoncé
Notons an = 1 + 2! + . . . + n!. En remarque que pour tout N ∈ N∗ , et pour tout n ≥ N , on a
an ≡ aN −1 (mod N !). Il faut maintenant remarquer (apres de longs calculs) que a8 ≡ 9 (mod 27).
Pour que ce soit une puissance parfaite, il faut donc que ce soit un carré. Or, a4 ≡ 3 (mod 5), mais
3 n’est pas un carré modulo 5, donc il n’y a pas de solution pour n > 8. Il ne reste plus qu’a traiter
les cas n ≤ 8. On peut tous les tester et on a

S = {(1, 1, k), (3, 3, 2)}

62
Solution 185. Enoncé
a) On remarque que k × kp = p × k−1 p−1
. D’apres le lemme de Gauss, on a donc p | p
car p ne
  
k
divise pas k.
b) En développant avec le binome de Newton, on a bien (x + y)p ≡ xp + y p (mod p)
c) On démontre alors par récurrence la proposition :

P(a) : ”p | ap − a”

Initialisation : p | 1p − 1
Hérédité : Supposons la propriété vraie pour un certain a ∈ N∗ . Alors (a + 1)p ≡ ap + 1 ≡ a + 1
(mod p), donc p | (a + 1)p − (a + 1).
Conclusion : La proposition est initialisée et héréditaire, donc pour tout a ∈ N∗ , p | ap − a.
Le résultat se généralise facilement aux nombres négatifs.
Source : Petit théorème de Fermat
Solution 186. Enoncé
On peut factoriser l’expression : (n − m)(n2 + nm + m2 ) = 24. On a évidemment, n ≥ m. De plus,
n2 + nm + m2 | 24, donc en particulier, n2 + nm + m2 ≤ 24.
Si m ≥ 3, on a n2 + nm + m2 ≥ 27, impossible. Donc m ∈ {0, 1, 2}.
Si m = 0, on a n3 = 24, ce qui n’a pas de solution.
Si m = 1, on a n3 = 25, ce qui n’a pas de solution.
Si m = 2, on a n3 = 32, ce qui n’a pas de solution.

Solution 187. Enoncé


D’apres le petit théorème de Fermat, on a 212 ≡ 1 (mod 13). On veut donc déterminer le reste de
la division euclidienne de 7071 par 12. On remarque que 70 ≡ 10 (mod 12), donc 7071 ≡ 1071 ≡ 4
71
(mod 12) car pour tout k ≥ 2, 10k ≡ 4 (mod 12), donc 270 ≡ 24 ≡ 3 (mod 13)
Solution 188. Enoncé
D’après le petit théorème de Fermat, 29p ≡ 29 (mod p), donc 29p + 1 ≡ 30 (mod p). On veut donc
que p | 30, ie p doit etre un diviseur premier de 30. Réciproquement, 2, 3 et 5 conviennent bien.
Donc
S = {2, 3, 5}

Solution 189. Enoncé


D’après le petit théorème de Fermat, on a 270 ≡ 210 (mod 1)3 et 370 ≡ 310 (mod 13). En calculant,
on a alors 270 ≡ 10 (mod 13) et 370 ≡ 3 (mod 13), donc on a bien 13 | 270 + 370
Solution 190. Enoncé
Il y a 40 entiers inferieurs à 100 qui sont premiers avec 100, donc φ(100) = 40. Alors 20062006 ≡
62006 ≡ 22006 × 32006 ≡ 22006 × 36 ≡ 29 × 22006 (mod 100). On remarque également que 220 ≡ 76
(mod 100) et que 762 ≡ 76 (mod 100), donc 20062006 ≡ 76 × 26 × 29 (mod 100), donc

20062006 ≡ 56 (mod 100)

Solution 191. Enoncé


On remarque que φ(n) ≤ n donc φ(n) | n!, donc comme aφ(n) ≡ 1 (mod n), on a bien an! ≡ 1
(mod n) (a est bien premier avec n, donc le theoreme d’Euler-Fermat s’applique bien).

63
2.3.2 Algèbre

Solution 192. Enoncé


On a (a − b)2 + (a − c)2 + (b − c)2 ≥ 0, donc en développant, divisant par 2 et réarrangeant, on a
l’inégalité recherchée.
Source : Lemme du tourniquet
Solution 193. Enoncé
Problème Mathraining #6105
Solution 194. Enoncé
Problème Mathraining #6128
Solution 195. Enoncé
On applique l’inégalité arithmético-géométrique 2 fois : a2 + b2 + c2 ≥ 3 × 41 , et a2 b2 + a2 c2 + b2 c2 ≥
1
3 × 16 . En sommant les deux, on a a2 + b2 + c2 + a2 b2 + a2 c2 + b2 c2 ≥ 16
15

Solution 196. Enoncé


Pour tout i, on a 1 + √1ai ≥ √2 ,
ai donc : ( a11 + 1) × . . . × ( a1n + 1) ≥ 2n × √
a1 ...an .
1

a1 +...+an n
De plus, d’après l’inégalité arithmético-géométrique, on a a1 × . . . × an ≤ = 1, donc

n
en passant à l’inverse (en n’oubliant pas d’inverser le sens de l’inégalité !), on a bien
   
1 1 1
+ 1 × ... × + 1 ≥ 2n × √ ≥ 2n
a1 an a1 . . . an

Solution 197. Enoncé


2n
On vérifiera que v√
n+1 = vn . Alors par récurrence immédiate, on a vn = v0 . Enfin, on remarque
2

que pour tout n, a ≤ un+1 ≤ un (récurrence immédiate en utilisant bien le fait que pour tout
√ √ √ n
entier m, um ≥ a), donc un − a = vn (un + a) ≤ 2u0 v02
Solution 198. Enoncé
Problème Mathraining #4227
Solution 199. Enoncé
Résolvons d’abord l’équation fonctionnelle sur Z
Analyse : En remplacant n et m par 0, on trouve f (0) = 2f (0), donc f (0) = 0. En remplacant m
par 1, on remarque que f (n+1) = f (n)+f (1). Par récurrence immédiate, on a que pour tout entier
naturel n, f (n) = nf (1). Remarquons maintenant en posant n = 1 et m = −1 que f (1) = −f (−1),
donc que f (n − 1) = f (n) − f (1) pour tout entier n, donc par récurrence immédiate en partant de
0, on a que pour tout entier négatif n, f (n) = nf (1). Ainsi, pour tout entier n, f (n) = nf (1)
Synthèse : Réciproquement, peu importe la valeur de f (1) =: a, on vérifie que f (n) = an convient.
(Toute valeur de a convient effectivement)
Pour passer à Q, commencons par remarquer que par récurrence immédiate sur n, pour tout
rationnel x et tout entier naturel n, f (nx) = nf (x). On procède de meme pour n un entier négatif.
Soit maintenant x = pq un rationnel. f (1)p = f (p) = f (q pq ) = pf (x), donc f (x) = x(1).
Synthèse : Réciproquement, toutes les fonction linéaires conviennent effectivement.
Source : Equation de Cauchy sur Q
Solution 200. Enoncé
n
Y (x − ak )
On note Li = . C’est un polynome de degré n qui s’annule en tous les ak , sauf en
(ai − ak )
k=0
k̸=i
n
X
ai ou il vaut 1. On peut donc poser P = bk Lk , qui vérifie toutes les conditions et qui est de
k=0
degré au plus n.
Si il existe deux polynomes P et Q qui vérifient ces conditions, alors P et Q sont de degré au plus
n et coincident en n + 1 points donc ils sont égaux
Source : Polynome d’interpolation de Lagrange
Solution 201. Enoncé
On remarque que le polynome P − X 2 s’annule en 1, 2, 3 et 4. Or, c’est un polynome de degré 4,
donc P − X = a(X − 1)(X − 2)(X − 3)(X − 4). Il ne reste plus qu’a determiner a grace a la valeur
de P en 0. On trouve alors a = 24
1
. On a donc P (−2) = 19

64
Solution 202. Enoncé
On remarque en évaluant en 0 que 0 est une racine de P . En injectant −1, on trouve que c’est une
racine, de meme pour −2 et −3. Donc P (x) = Q(x)x(x + 1)(x + 2)(x + 3).
Réciproquement, on a alors (X + 4)X(X + 1)(X + 2)(X + 3)Q(X) = X(X + 1)(X + 2)(X + 3)(X +
4)Q(x+1). Alors Q(X) = Q(X +1). Cela implique alors que Q est constant : Q(X)−Q(0) s’annule
en tout point de Z
Réciproquement, tout polynome de la forme aX(X +1)(X +2)(X +3) convient (meme le polynome
nul)
Solution 203. Enoncé
Soit P un polynome et α une racine de P . Remarquons que pour tout x ∈ R,
P (x) = P (x) − P (α)
Xn Xn
= ak xk − ak α k
k=0 k=0
Xn
= ak (xk − αk )
k=1
n k−1
!
X X
l n−1−α
= ak (x − α) xα
k=1 l=0

n k−1
!
X X
l n−1−α
P (x) = (x − a) ak xα
k=1 l=0
| {z }
:=Q(x)

Donc, si un polynome de degré n pouvait avoir n + 1 racines, on pourrait le factoriser par ces n + 1
racines et on aurait un produit de n + 1 polynomes de degré 1, donc un polynome de degré n + 1,
absurde.
Si |x| était un polynome, on pourrait noter n son degré. On aurait alors |x| − x qui aurait une
infinité de racines (tous les nombres positifs) mais qui ne serait pas le polynome nul, ce qui est
absurde. Ce n’est donc pas un polynome.
Solution 204. Enoncé
Résolvons cette équation avec la forme exponentielle. Soit z = reiθ , r ∈ R+ , θ ∈ R tel que z n = 1.
Alors rn = 1 et einθ = 1.
D’après la première équation, on a r = 1 et d’après la deuxième équation, il existe k ∈ Z tel que
nθ = 2kπ ie θ = 2kπ
n .
2ikπ 2ik′ π
Enfin, remarquons que si k ≡ k ′ (mod n), alors e n =e n , donc :
2ikπ
Un = {e n , k ∈ [[0, n − 1]]}

Solution 205. Enoncé


2iπ
Supposons que Um ⊂ Un . Posons z = e m . Alors comme z ∈ Um , il existe k ∈ [[0, n − 1]] tel que
2ikπ
z = e n , donc 2iπ
m = n ie n = km ie m | n
2ikπ

Solution 206. Enoncé


1) On a
u v 1 1 v−u |u − v|
− 2 = − = =
|u|2 |v| u v uv |u||v|
2) On a
|y − z| y z y x x z |y − x| |x − z|
= − 2 ≤ − 2 + − 2 = +
|y||z| |y|2 |z| |y|2 |x| |x|2 |z| |y||x| |x||z|
D’après l’inégalité triangulaire. En multipliant par |x||y||z| on obtient le résultat voulu.
3) On applique l’inégalité précédente à x − y, x − z et x − w. On vient de montrer que dans un
quadrilatère, la somme des produits des longueurs des cotés opposés est superieure au produit des
longueurs des diagonales.
NB : On peut montrer que l’égalité est atteinte SSI le quadrilatère est cyclique, ce résultat s’appèle
l’égalité de Ptolémée.
Source : Inégalité de Ptolémée

65
Solution 207. Enoncé
D’apres le lemme du tourniquet, on a x2 + y 2 + z 2 ≥ xy + xz + yz, donc (x + y + z)2 = x2 + y 2 +
z 2 + 2(xy + xz + xy) ≥ 3(xy + xz + yz). On a alors :
 2
2 1 1 1
(a + b + c) ≥ + +
a b c
 
1 1 1
≥3 + +
ab ac bc
a+b+c
=3
abc

3
En divisant par (a + b + c), on retrouve bien finalement : x + y + z ≥
abc
Source : Roumanie 2005
Solution 208. Enoncé
Problème Mathraining #6207
Solution 209. Enoncé
4(a+c) 4(b+d)
1
LHS = (a + c)( a+b 1
+ c+d 1
) + (b + d)( b+c 1
+ d+a ) ≥ a+b+c+d + a+b+c+d = 4 d’après l’inégalité
arithmético-harmonique ( 1 + 1 ≤ 2 ⇐⇒ x+y ≤ x + y ).
2 x+y 4 1 1
x y

Solution 210. Enoncé


Soit x une solution. Notons n ∈ N sa partie entiere et 0 ≤ y < 1 sa partie fractionnaire. On a
alors ny = 2019(n + y), donc y(n − 2019) = 2019n. Or, n ̸= 2019 (on aurait 0 = 20192 ) donc on a
2019n
y= (on ne voulait pas diviser par 0).
n − 2019
On peut maintenant distinguer des cas en fonction des valeurs de n. Si n > 2019, on a 2019n >
n > n − 2019 donc y > 1, impossible. Si maintenant 2019 > y > 0, on a 2019n > 0 et n − 2019 < 0
donc y < 0, impossible. Donc n ≤ 0. Comme y ≥ 0, on a 2019n ≤ n − 2019 donc 2019 ≤ −2018n
donc 0 ≥ n ≥ − 2019
2018 . Donc n = 0 ou n = −1. Enfin, si n = −1, y = − 2020 . Donc
1

 
1
S= 0, −
2020

Source : Coupe animath de printemps 2019


Solution 211. Enoncé
n   n  
X n X n
Notons A = et B = .
k k
k=0,k≡0[2] k=0,k≡1[2]
n  
X n
On a A + B = 2 (formule classique) et A − B =
n
(−1) k
= (1 − 1)n = 0, donc
k
k=0

A = B = 2n−1

66
2.3.3 Combinatoire

Solution 212. Enoncé


1) Supposons par l’absurde que chaque tiroir a au plus une chaussette. Alors il y a en tout au plus
n × 1 = n chausettes, ce qui est absurde puisqu’il y en a n + 1.
2) Supposons par l’absurde que chaque tiroir a au plus k chaussettes. Alors il y a en tout au plus
n × k = kn chausettes, ce qui est absurde puisqu’il y en a kn + 1.
3) Supposons par l’absurde que chaque tiroir a un nombre fini de chaussettes. Notons m le nombre
maximal de chaussettes dans un tiroir. Alors il y a en tout au plus n × m = nm chausettes, ce qui
est absurde puisqu’il y en a une infinité.
Source : Principe des tiroirs
Solution 213. Enoncé
Source : IMO SL 2009
Solution 214. Enoncé
Il y a un nombre fini de telles configurations possibles, donc la somme des longueurs des segments
possède un minimum. Mettons nous dans cette situation minimale. Par l’absurde, si deux segments
se croisent, alors on peut les "décroiser" et l’inégalité triangulaire nous assure alors que la somme
des longueurs sera plus petite, ce qui constitue une absurdité.
Solution 215. Enoncé
On regroupe les n entiers en paires : {{1, 2}, {3, 4}, . . . , {2n − 1, 2n}}. Si on choisit n entiers, alors
d’après le principe des tirois, on en aura deux qui appartienneront à la meme paire. Or, deux
éléments d’une meme paire sont bien premiers entre eux.
D’après l’unicité de la décomposition en produit de facteurs premiers, chaque nombre ici peut
s’écrire de manière unique sous la forme 2k (2l + 1). Or ici, l varie entre 0 et n − 1, donc on a n
choix possibles. D’après le principe des tiroirs, il existe deux entiers parmi les n + 1 qui auront le
meme l, donc l’un divise l’autre.
Solution 216. Enoncé
Remarquons que pour chaque élément de l’ensemble, on peut décider de le prendre ou de ne pas
le prendre. Il y a donc 210 = 1024 possibilités. Remarquons également que la somme d’au plus
10 éléments entre 1 et 100 est inferieure à 1000. Par le principe des tiroirs, il existe donc deux
sous-ensembles distincts qui ont la meme somme. S’ils ne sont pas disjoints, il nous suffit de retirer
les éléments en commun (ils conserveront bien la meme somme puisqu’on retire la meme quantité
aux deux sous-ensembles).
Solution 217. Enoncé
Considérons g le garcon ayant dansé avec le plus de filles et f ′ une fille avec qui il n’a pas dansé. Il
existe un garcon g ′ avec lequel f ′ a dansé. Or, comme g a dansé avec le plus de filles et il n’a pas
dansé avec f ′ , alors il existe une fille f qui n’a pas dansé avec g ′ mais qui a dansé avec g.
Solution 218. Enoncé
 2  2  2         
n n n n n n n n n
Remarquons que + + ··· + = + + ··· + .
0 1 n 0 n 1 n−1 n 0
Considérons deux urnes de n boules chacune. On a donc 2n boules. On veut en choisir n. Pour cela,
on peut en choisir 0 de la première et n de la deuxième, 1 de la première et n − 1 de la deuxième,
etc..., ce qui correspond exactement a la quantité écrite ci-dessus. Cela correspond également au
nombre de facons de choisir n boules parmi 2n, ce qui nous donne l’égalité voulue.

Solution 219. Enoncé


On montre par récurrence que tous les entiers naturels conviennent. En effet, n = 1 convient
trivialement. Supposons maintenant que l’on sait paver toute grille 2n × 2n donc il manque un
coin. Alors On peut séparer notre grille 2n+1 × 2n+1 en quatre grilles de taille 2n × 2n . Il manque
la case en bas a gauche de la grille en bas à guauche, donc par hypothèse de récurrence, on peut la
paver. Pour les autres, il suffit d’ajouter la pièce qui chevauche les trois grilles, et on se retrouve à
devoir paver 3 grilles auquelles il manque un coin, ce que l’on peut faire par hypothèse de récurrence.
Solution 220. Enoncé
Comme chaque pièce recouvre 4 cases, il faut que n soit pair. On colorie maintenant la grille comme
un echiquier. Une pièce recouvre soit 3 cases blanches et une case noire, soit l’inverse. Comme il y
a autant de cases blanches que de cases noires, il nous faut autant de chaque type de pièce. Il nous

67
faut donc un nombre pair de pièces, et donc que le nombre total de cases soit divisible par 8, donc
que n soit divisible par 4. Réciproquement, on peut paver toute grille de 4k × 4k grace au motif
suivant :

Solution 221. Enoncé


Notons R et V le nombre de caméléons rouges et verts respectivement. Modulo 3, R − V est un
invariant du problème (il est facile de s’en convaincre). Or, initialement, R − V ≡ 1 (mod 3), donc
R − V ne pourra jamais etre nul : il restera toujours un caméléon vert ou un caméléon rouge.
Solution 222. Enoncé
La liste des entiers de 1 à n, peu importe l’ordre est appelée une permutation de [[1, n]]. Si on a i
et j tels que i < j mais j apparait avant i dans la liste, on dit que c’est une inversion. On itrosuit
alors la notion de signature : soit σ une permutation, ε(σ) vaut 1 si σ a un nombre pair d’inversions
et −1 sinon. On va montrer que tout mouvement (ie toute transposition) change le signe de ε.
Supposons donc que l’on échange i < j. Le fait de les échanger change de 1 le nombre d’inversions
(si i éait avant j, on ajoute une inversion, sinon on en retire une). Soit maintenant k entre i et j
dans la liste. Si k < i, l’inversion (k, i) est remplacée par (k, j), de meme si k > j. Si i < k < j, on
augmente ou on retire 2 au nombre d’inversions. Dans tous les cas, on change donc le signe de ε.
Mais alors en un nombre pair d’étapes, le signe de ε ne change pas : il reste constant égal à −1, ce
qui ne correspond pas au cours ou la liste est triée.
Solution 223. Enoncé
Montrons que c’est impossible.
Les doites séparent l’espace en régions distincts. Montrons par récurrence que l’on peut colorier
ces régions en 2 couleurs de telle sorte que deux régions partageant une arete ne soient pas de la
meme couleur. Le cas avec aucune droite est trivial. Supposons le résultat vrai pour n droites. En
ajoutant une droite, on inverse les couleurs de toutes les régions d’un meme coté de la droite et on
a fini.
Sans perte de généralité, supposons donc que Turbo l’escargot démare avec une région de couleur
1 à sa gauche et de couleur 2 à sa droite. A chaque tournant, il converve les couleurs à a droite et
à sa gauche, donc il ne pourra jamais revenir sur un meme segment dans les deux sens : il faudrait
pour cela inverser les couleurs à sa droite et à sa gauche.
Source : EGMO 2017

68
2.3.4 Géométrie

Solution 224. Enoncé


On note HA , HB et HC les projetés orthogonaux de A, B et C sur le triangle. On sait que
AHB HHC est cyclique, donc CAB \ = 180 − H\ C HHB . Par angles opposés, CAB = 180 − BHC.
\ \
Comme H est le symétrique de H, on a donc CAB = 180 − BH C, donc le quadrilatère ABH ′ C
′ \ \ ′

est cyclique, ie H ′ appartient au cercle circonscrit à ABC.

Solution 225. Enoncé


Soit d la médiatrice de [XY ]. Puisque O est le centre du cercle ω, O appartient à d. Ainsi, les
cercles centrés en O, et en particulier ω et Ω, sont envoyés sur eux-memes par la symétrie d’axe d.
On note s cette symétrie.
On a s(X) = Y , s(ω) = ω et s(Ω) = Ω, donc s envoie la tangente à ω en X sur la tangente à
s(ω) = ω en s(X) = Y . Le point d’intersection P de Ω avec la tangente à ω en X est envoyé sur le
point d’intersection de s(Ω) avec la tangente à s(ω) en s(X). Il s’agit du point d’intersection de Ω
avec la tangente à ω en Y , a savoir le point Q. Autrement dit, P et Q sont symétriques par rapport
à la médiatrice de [XY ]. Les segments [XY ] et [P Q] sont perpendiculaires à d donc parallèles entre
eux et les segments [P X] et [QY ] sont symétriques par rapport à d. Ceci suffit à conclure que le
quadrilatère P QY X est un trapeze isocèle.
Source : Envoi de géométrie 2023, solution officielle
Solution 226. Enoncé
Notons S ′ le point d’intersection de la bissectrice de l’angle BAC
\ avec le cercle circonscrit à ABC.
Le quadrilatère ABS C est cyclique, donc d’après le théorème de l’angle inscrit, CBS
′ \′ = CBS \′ ,
donc le triangle ABS est isocèle en S, donc S appartient à la médiatrice du segment [BC], donc

S = S ′ est le point d’intersection de la bissectrice, de la médiatrice et du cercle circonscrit.

Source : Théorème du pole Sud

69
Solution 227. Enoncé
On s’interesse tout d’abord au point A. Notons A′ le point diamétralement opposé à B sur le
cercle circonscrit à ABC. Le théorème de l’angle inscrit nous affirme que BAC
\ = BA \ ′ C, donc
b = BC′ ie a = BA′ donc la quantité est bien égale au diamètre du cercle circonscrit. En
sin(A) BA sin(A)
b
appliquant le meme raisonnement à B et C, et en notant R le rayon du cercle circonscrit, on a

a b c
= = = 2R
sin A sin B sin C

Source : Loi des Sinus


Solution 228. Enoncé
On note a (resp. b, c) la longueur du coté opposé à A (resp. B, C). Soit D le pied de la hauteur
issue de A. Notons d = DC et h = AD. Alors DB = a − d. D’après le théorème de Pythagore, on
a c2 = h2 + (a − d)2 et h2 + d2 = b2 , donc c2 = a2 + b2 − 2ad = a2 + b2 − 2ab cos(C)
b

Source : Loi des Cosinus

Solution 229. Enoncé


Soit O le centre le l’homothétie qui envoie B sur B ′ et C sur C ′ (une telle homothétie existe bien
puisque (BC) ∥ (B ′ C ′ )). Montrons que cette meme homothétie envoie A sur A′ : la droite (AC)
est envoyée sur une parallèle à (AC) passant par C ′ et la droite (AB) est envoyée sur une parallèle
à (AB) passant par B ′ . Donc A = (AC) ∩ (AB) est envoyé sur l’intersection des images de (AC) et
(AB), donc sur A′ . Donc O ∈ (AA′ ), ce qui démontre que les trois droites sont bien concourantes.

70
Solution 230. Enoncé
Remarquons que si un tel carré DEF G est construit, alors comme (F G) ∥ (DE), il exsite une
homothétie de centre (BG) ∩ (CF ) = A qui envoie B sur G et C sur F . Cette meme homothétie
envoie donc le carré de coté [BC] ne contenant pas ABC sur DEF G. Il suffit donc de construire ce
carré BCY X, puis de regarder les points d’intersection de (XA) et (Y A) avec (BC). Cela forme
bien le carré recherché.

Solution 231. Enoncé


a+c−b
On utilise la transformation de Ravi : a = y + z, b = x + z, c = x + y. Alors =
2
a + b − c
z+y+x+y−x−z
2 = y et de la meme manière, = z. Le cercle ex-inscrit se traite de la meme
2
manière.

71
72
2.4 Dessert

2.4.1 Arithmétique

Solution 232. Enoncé


Notons Q(n) : ”∀k ∈ [[0, n]], P (k) est vraie". Le theoreme de récurrence nous apprend que si Q(0)
est vraie et Q(n) =⇒ Q(n + 1), alors Q(n) est vraie pour tout entier naturel n. Les hypothese de
l’enoncé nous apprennent qu’on a bien P (0) vraie donc Q(0) vraie et Q(n) =⇒ P (n + 1), donc
Q(n) =⇒ Q(n + 1), donc Q(n) est vraie pour tout entier naturel n, donc P (n) est vraie pout tout
entier naturel n.
Solution 233. Enoncé
On montre par récurrence forte l’existence d’une telle décomposition :
Initialisation : P (2) est vraie puisque 2 est un nombre premier.
Hérédité : Supposons P (2), P (3), . . . , P (n) vraies, montrons P (n + 1) : si n + 1 est premier, on
a fini, puisqu’il est egal a sa propre decompositon, sinon il exsite deux nombres superieurs ou
egaux a 2 p et q tels que n + 1 = pq, par hypothese de récurrence, p et q possedent leur propre
decomposition, donc n + 1 possede une decomposition qui est le produit des decompositions de p
et q, donc P (n + 1) est vraie.
Conclusion : La proposition est initialisée et hérérditaire, donc tout nombre entier superieur ou
egal a 2 possede une unique decomposition en produit de facteurs premiers.
Source : Théorème fondamental de l’arithmétique
Solution 234. Enoncé
Problème Mathraining #3324
Solution 235. Enoncé
Commencons par démontrer que −1 est un résidu quadratique modulo p un nombre premier SSI
p ≡ 1 (mod 4). En effet, soit p tel que −1 soit un résidu quadratique modulo p, alors d’après
p−1
le petit théorème de Fermat, (−1) 2 ≡ 1 (mod p) ⇐⇒ p−1 2 ≡ 0 (mod 2) ⇐⇒ p − 1 ≡ 0
(mod 4) ⇐⇒ p ≡ 1 (mod 4).
Supposons par l’absurde qu’il exsite un nombre fini de nombres premiers de la forme voulue et
notons les p1 . . . pk . Alors soit N = (2p1 . . . pk )2 + 1. Si N est premier, on obtient une contradiction.
Sinon, soit q un diviseur premier de N . On a alors N ≡ 0 (mod q), donc (2p1 . . . pk )2 ≡ −1 (mod q),
donc q ≡ 1 (mod 4). Or, q est premier et n’est égal à aucun des pk : c’est donc un nouveau nombre
premier de la forme recherchée, ce qui conclut.
Solution 236. Enoncé
On veut p | (p − 1)! + 1 donc d’après de théorème de Wilson, p est premier. On a maintenant
(p − 1)! = pm − 1 donc (p − 2)! = 1 + p + . . . + pm−1 . Comme p est premier, p − 1 n’est pas premier
(on vérifie aisement que le cas p = 3 ne convient pas), donc p − 1 | (p − 2)!, donc 1 + . . . + pm−1 ≡ 0
(mod p − 1), donc m ≡ 0 (mod p − 1), donc m vaut au moins p, absurde car (p − 1)! < (p − 1)p <
pp ≤ pm . Il n’y a donc bien pas de solution.
Solution 237. Enoncé
Problème Mathraining #3422
Solution 238. Enoncé
Par l’absurde, supposons avoir un produit de 3 nombres consécutifs non-nuls qui est une puissance
parfaite. Alors (n − 1)n(n + 1) = mk , mais n − 1 et n sont premiers entre eux, tout comme n et
n + 1, donc n et (n − 1)(n + 1) doivent etre des puissances parfaites, ie n = al , n2 − 1 = bc . Alors
a2l = bc + 1 : on a deux puissances qui sont des nombres consécutifs, absurde.
Solution 239. Enoncé
L’équation se réécrit 2a 3b = c2 − 9 = (c − 3)(c + 3).
Si b = 0, on a deux puissances de 2 séparées de 6, donc on a une unique solution (4, 0, 5)
Si a = 0, on a deux puissances de 3 séparées de 6, donc on a une unique solution (0, 3, 6)
Si maintenant a, b ≥ 1, alors (c − 3)(c + 3) est divisible par 6, donc c + 3 et c − 3 étant congrus
l’un à l’autre modulo 6, les deux sont divisibles par 6. En notant c − 3 = 6k et c + 3 = 6(k + 1),
on a 2a 3b = 36k(k + 1), donc a, b ≥ 2. Alors 2a−2 3b−2 = k(k + 1). Si k = 1, on a la solution
(3, 2, 9). Si maintenant k > 1, alors comme k + 1 et k sont premiers entre eux, un des deux est
une puissance de 2, l’autre une puissance de 3. Il faut donc résoudre 2n − 3m = ±1 (ceci est laissé

73
en exercice au lecteur, un des cas ayant déjà été traité plus haut). On trouve alors les solutions
(4, 3, 21), (3, 2, 9), (5, 4, 51). Donc

S = {(0, 3, 6), (3, 2, 9), (4, 0, 5), (4, 3, 21), (5, 4, 51)}

Solution 240. Enoncé


On cherche n tel que 2n ≡ 1 (mod n). n = 1 convient bien, supposons donc n ≥ 2. Soit p le
plus petit diviseur premier de n. Alors d’apres le petit théorème de Fermat affirme que 2p−1 ≡ 1
(mod p) et on a déjà 2n ≡ 1 (mod p). Or, on sait que l’ordre de 2 modulo p doit diviser à la fois
p − 1 et n, mais p étant le plus petit diviseur premier de n, alors le seul diviseur commun à n et
p − 1 est 1, donc 2 ≡ 1 (mod p), donc p = 1, impossible. Donc

S = {1}

Solution 241. Enoncé


Notons n = pα 1 × . . . × pk la décomposition en produit de facteurs premiers de n. Remarquons
1 αk

deux proprietes de la fonction φ :


⋆ Si p est un nombre premier, alors φ(pm ) = pm−1 (p − 1). En effet, entre 1 et pm les seuls entiers
qui ne sont pas premiers avec pm sont les multiples de p : il y en a donc pm−1 (p, 2p, ..., pm−1 p),
donc il y a pm − pm−1 nombres premiers avec pm , donc φ(pm ) = pm−1 (p − 1)
⋆ La fonction φ est multiplicative : si a et b sont premiers entre eux, alors φ(ab) = φ(a)φ(b). C’est
ici une consequence du theoreme des reste chinois, qui sera etudie ulterieurement.
De ces deux points, on peut par recurrence immediate sur le nombre de facteurs premiers distincts
de n, trouver que
   
1 1
φ(n) = pα
1
1 −1
× ... × pkαk −1 =n 1− ... 1
p1 pk

Solution 242. Enoncé


n
X
Commencons par noter que n = 1. On peut partitionner [[1, n]] en fonction de l’ordre des
k=1
elements modulo n. Pour chaque diviseur d de n, il y a exactement φ(d) elements qui sont d’ordre
d. En effet, pour que k soit d’ordre d, il faut qu’il soit premier avec d et pas avec un autre diviseur
de n multiple de d. On en deduit donc le formule voulue :
X
n= φ(d)
d|n

74
2.4.2 Algèbre

Solution 243. Enoncé ! !


n
X n
X n
X n
X
Considérons le polynome P = a2k X2 + 2 a k bk X+ b2k = (ak X + bk )2 ≥ 0. Ce
k=1 k=1 k=1 k=1
polynome est de signe constant donc son discriminant est négatif, ie
n
!2 n
! n
!
X X X
2 ak bk −4 a2k b2k ≤0
k=1 k=1 k=1

n
!2 n
! n
!
X X X
⇐⇒ a k bk ≤ a2k b2k
k=1 k=1 k=1

Source : Inégalité de Cauchy-Schawrz


Solution 244. Enoncé
√ √
On applique l’inégalité de Cauchy-Schwarz aux suites ( √xy11 , . . . , √xynn ) et ( y1 , . . . , yn ), puis on
divise par y1 + . . . + yn .
Source : Inégalité des mauvais élèves
Solution 245. Enoncé
Cette inegalite est symetrique, donc on peut poser a ≤ b ≤ c sans perte de generalite. On a donc
a2 + b2 + c2 ≥ ab + ac + bc d’apres le lemme du tourniquet. (1)
De plus, par mauvais eleves, on a :

a b c a2 b2 c2 (a + b + c)2 a2 + b2 + c2 3
+ + = + + ≥ = +1 ≥
b+c c+a a+b ab + ac bc + ab ac + bc 2(ab + ac + bc) 2(ab + ac + bc) 2

d’apres (1), ce qui conclut.


Source : Inégalité de Nesbitt
Solution 246. Enoncé
Problème Mathraining #6423
Solution 247. Enoncé
Problème Mathraining #6277
Solution 248. Enoncé
Si u0 = 0, la suite converge évidemment vers 0 puisqu’elle est constante égale à 0. Supposons alors
u0 ̸= 0. Notons q la raison de la suite. Pour tout entier naturel n, un = u0 × q n , donc pour que la
suite un converge, il faut que q n converge. On distingue alors quelques cas :
Si q > 1, alors q n devient arbitrairement grand, donc la suite ne converge pas.
Si q = 1, la suite est stationnaire donc elle converge vers u0 puisqu’elle est constante égale à u0 .
Si −1 < q < 1, alors q n peut devenir arbitrairement petit, donc un converge et tend vers 0.
Si q = −1, alors un alterne entre u0 et −u0 . Il suffit alors de prendre ϵ = | u20 | et la suite ne converge
pas.
Si q < −1, alors en valeur absolue, q n va devenir arbitrairement grand, donc la suite ne converge pas.

Solution 249. Enoncé


Problème Mathraining #4312

Solution 250. Enoncé


1) Il suffit de vérifier que les suites (un )n∈N et (λ1 r1n +λ2 r2n )n∈N ont la meme formule de récurrence,
λ1 et λ2 étant définis par les termes initiaux.
2) Il suffit de vérifier que les suites (un )n∈N et ((λ1 +λ2 n)rn )n∈N ont la meme formule de récurrence,
λ1 et λ2 étant définis par les termes initiaux.

Solution 251. Enoncé


Problème Mathraining #5232

75
Solution 252. Enoncé
On montre le résultat par récurrence.
Initialisation : Pour n = 2 le résultat est vrai par définition d’une fonction convexe.
Hérédité : Supposons le résultat
Pn vrai pour un certain n. Soient λ1 , . . . , λn+1 ∈]0, 1[ et x1 , . . . , xn+1
des réels. Alors notons y = k=1 1−λ λk
n+1
x k et λ = 1 − λn+1 Alors λ + λn+1 = 1, donc f (λy +
λn+1 xn+1 ) ≤Pλf (y) + λn+1 xn+1 . On peut alors appliquer l’hypothese de récurence sur y et cela
n
conclut car k=1 1−λ λk
n+1
=1
Conclusion : La proposition est initialisée et héréditaire donc d’après le théorème de récurrence, la
propriété est vraie pour tout entier naturel n.

On peut appliquer l’inégalité à la fonction ln et prendre λi = 1


n pour obtenir l’inégalité arithmético-
géométrique.
Source : Inégalité de Jensen
Solution 253. Enoncé
Supposons par l’absurde qu’il exsite un entier a tel que f (a) ̸= g(a). Alors l’ensemble {g(n)|f (n) ̸=
g(n)} est une sous-partie non-vide de N, donc elle possède un minimum, que l’on note g(b). Alors
g(b) < f (b). Mais alors comme f est surjective, il existe un entier c tel que f (c) = g(b) < f (b).
Comme g est injective, et que c ̸= b, alors g(c) ̸= g(b) = f (c), mais comme g(c) ≤ f (c) = g(b) et
g(c) ̸= g(b), alors g(c) < f (c) et g(c) < g(b), ce qui est absurde car g(b) était supposé minimal.
Source : Roumanie 1986
Solution 254. Enoncé
Soit f une éventuelle solution et g = f ◦ f . On remarque que g est injective, donc f aussi, et que
pour tout entier naturel n, f (n) ̸= n, sinon f (f (n)) = f (n) = n, ce qui est impossible. Notons
Ok = {g n (k), n ∈ N} = {2023n + k, n ∈ N} l’orbite de k par g. On remarque que N est la réunion
disjointe des O1 , . . . , O2023 .
Soit k ∈ [[1, 2023]]. Il exsite m ∈ N et n ∈ [[1, 2023]] tels que f (k) = g m (n) ie f (k) = f 2m (n), donc
par injectivité de f , soit m = 0 et f (k) = n, soit m = 1 et k = f (n) (et dans ce deuxième cas,
f (k) = f (f (n)) = n + 2023). Dans tous les cas, on peut faire des paires {n, f (n)} d’éléments de
[[1, 2023]], mais comme f (n) ne vaut jamais n, on devrait en avoir un nombre pair, absurde car
2023 est impair.
Source : IMO 1987
Solution 255. Enoncé
Si P = (X − α)k Q, alors P ′ = (X − α)k−1 (Q + (X − α)Q′ ). Par récurrence immédiate, on a le
résultat voulu.
Réciproquement, P (α) = P ′ (α) = 0, alors P s’écrit sous la forme (X − α)Q (α est une racine de
P ) et P ′ = (Z − α)Q′ + Q, donc α est une racine de Q en réarrangeant. Par récurrence immédiate,
on a le résultat voulu.
Solution 256. Enoncé
Problème Mathraining #4542
Solution 257. Enoncé
Problème Mathraining #4567
Solution 258. Enoncé
cos(θ + φ) = ℜ(ei(θ+φ) ) = ℜ(eiθ eiφ ) = ℜ(eiθ )ℜ(eiφ ) − ℑ(eiθ )ℑ(eiφ ) = cos(θ) cos(φ) − sin(θ) sin(φ)
sin(θ + φ) = ℑ(ei(θ+φ) ) = ℑ(eiθ eiφ ) = ℑ(eiθ )ℜ(eiφ ) + ℑ(eiθ )ℜ(eiφ ) = sin(θ) cos(φ) + cos(θ) sin(φ)
On remarque avec les points précédents que 2 cos(a) cos(b) = cos(a + b) + cos(a − b). En prenant
2 et φ = 2 on a le résultat voulu.
θ = a+b a−b

On remarque avec les points précédents que 2 sin(a) cos(b) = sin(a + b) + sin(a − b). En prenant
2 et φ = 2 on a le résultat voulu.
θ = a+b a−b

Source : Formules de duplication

Solution 259. Enoncé

76
n   n  
!
X n X n i(kx)
cos(kx) = ℜ e
k k
k=0 k=0
n   !
X n ix k

=ℜ e
k
k=0
= ℜ (e + 1)n par le binome de Newton
ix

 ix ix −ix

= ℜ (e 2 )n (e 2 + e 2 )n
 x n  nix 
= 2n cos ℜ e 2
2
 x n  nx 
= 2n cos cos
2 2
Solution 260. Enoncé
1) On a (a2 + b2 )(c2 + d2 ) = (ad − bc)2 + (ac + bd)2 (il suffit de développer pour s’en convaincre)
2) Modulo 4, on carré vaut 0 ou 1 donc la somme de deux carrés vaut 0, 1 ou 2, mais 0 et 2 sont
impossibles car cela donnerait un nombre pair, donc on a bien p ≡ 1 (mod 4)
3) a2 ≡ b2 (mod p) ⇐⇒ p | a2 − b2 ⇐⇒ p | a − b ou p | a + b car p est premier, donc c’est
p−1
équivalent à a ≡ b (mod p) ou a ≡ −b (mod p), donc on a carrés distincts modulo p.
2
4) On veut montrer que −1 est un carré modulo p. Les carrés sont exactement les nombres qui
p−1
élevés à la puissance p−1 2 donnent 1 (racines du polynome X
2 − 1). Or, comme p ≡ 1 (mod 4),
p−1

2 est pair, donc √ (−1) 2 = 1, donc c’est effectivement un carré.


p−1

5) On pose N = ⌊ p⌋ + 1 et ξ = np et on considère les N réels xk = kξ − ⌊kξ⌋ et les intervalles


     
1 N −1 N −1
0, , ..., , 1 . On distingue alors deux cas : Si un des xk est dans l’intervalle ,1 ,
N N N
alors b = k et a = ⌊kξ⌋ + 1 conviennent ; sinon, par le principe des tiroirs, il exite k et l tels que
xk et xl soient dans le meme intervalle. SPDG, posons k < l. Alors b = l − k et a = ⌊lξ⌋ − ⌊kξ⌋
conviennent.
6) D’après la question précédente, on a a et b des entiers tels que 0 < b2 < p et (bn − ap)2 ≤ p, donc
0 < (bn−ap)2 +b2 < 2p. Or, n2 +1 ≡ 0 (mod p) donc en développant, on trouve (bn−ap)2 +b2 ≡ 0
(mod p) et avec l’inégalité précédente, on a (bn − ap)2 + b2 = p .
7) D’apres les questions précédentes, un nombre premier est la somme de deux carrés SSI il est
congru à 1 modulo 4. Ainsi, comme un nombre carré est la somme de deux carrés (a2 = a2 + 02 ), si
un entier est de la forme décrite dans l’énoncé, il est bien la somme de deux carrés. Réciproquement,
si n = a2 + b2 , alors notons d = gcd(a, b) et n′ = a′2 + b′2 avec a′ = ad , b′ = db . Soit p un nombre
premier congru à 3 modulo 4. Alors il ne divise pas n′ .
En effet, si p ≡ 3 (mod 4) et p | a′2 + b′2 , alors p | a′ et p | b′ puisque si p ne divise pas b′ , alors
a′2 ≡ −b′2 (mod p) et comme b est premier avec p, il est inversible ie (ab−1 )2 ≡ −1 (mod p),
impossible d’après 4).
Donc p ne divise pas n′ (a′ et b′ étant premier entre eux), donc la valuation p-adique de n est la
meme que celle de d2 ie elle est paire.
Source : Théorème des deux carrés
Solution 261. Enoncé
√ √
1) On développe ( a − b)2 et on divise par 2 pour obtenir le résultat.
2) On applique l’inégalité précédente à a+b
2 et 2 , soit
c+d

√ √
a+b c+d √ √ √
q
a+b+c+d 2 + 2 ab + cd 4
= ≥ ≥ ab cd = abcd
2 2 2

3) On applique l’inégalité précédente avec d = a+b+c


3 . On a alors
s
a+b+c  
a+b+c a+b+c+ 3 4 a+b+c
= ≥ abc
3 4 3

77
q
On peut alors diviser par 4 a+b+c
3 puis élever à la puissance 43 , et on obtient

a+b+c √
3
≥ abc
3

4) On peut maintenant montrer le résultat par récurrence : avec le point 2 il est vrai pour toute
puissance de 2 et avec le point 3, s’il est vrai pour n il est vrai pour n − 1, donc pour tout n ∈ N,
x1 + . . . + xn √
on a bien ≥ n ax1 x2 . . . xn
n
Solution 262. Enoncé
Problème Mathraining #6462

78
2.4.3 Combinatoire

Solution 263. Enoncé


Soit (un )1≤n≤RS+1 une suite. On note an et bn la taille de la plus grande sous-suite croissante et
de la plus grande sous-suite décroissante de (um )1≤m≤n dont le dernier terme est un (donc des
entiers naturels). Par l’absurde, si pour tout n, on a an ≤ R et bn ≤ S, alors les couples (an , bn ) ne
peuvent prendre que RS valeurs, donc d’après le principe des tiroirs, il exsite k < l tels que al = ak
et bl = bk , mais alors si SPDG, uk ≤ ul , alors une sous-suite croissante de taille ak terminant par
uk peut se prolonger en une sous-suite croissante de taille ak + 1 > al se terminant par al , absurde.
Source : Théorème d’Erdos-Szekeres
Solution 264. Enoncé
Problème Mathraining #1337
Solution 265. Enoncé
Problème Mathraining #1315
Solution 266. Enoncé
On laisse au lecteur le soin de montrer que les nombres de Catalan sont la solution au problème.
Solution 267. Enoncé
Il faut montrer que la suite de Fibonacci au carré est une solution du problème, en faisant une
récurrence et en faisant une disjonction de cas sur le type de tétromino utilisé dans l’hérédité.
Solution 268. Enoncé
Problème Mathraining #1479
Solution 269. Enoncé
Pour n = 1, on prend un tableau 1 × 2 (vertical) et on remarque que la différence des deux nombres
est un invariant du problème : on ne pourra pas toujours terminer.
Supposons maintenant n > 2. On regarde modulo n − 1, la première opération n’a pas d’influence
et en se placant dans un tableau 1 × 2 à nouveau, on remarque que la différence des termes est un
invariant du problème, donc on ne pourra pas toujours terminer.
Montrons alors que n = 2 est la seule solution. On remarque qu’il suffit de savoir traiter une
colonne pour terminer, on pourra alors les traiter les unes après les autres. (la multiplication de
0 par n donne bien 0...). Traitons alors le cas d’une colonne 1 × 2. Si un des deux nombres ne
n’est pas pair, on le multiplie par 2, de manière à avoir 2 nombres pairs. Ensuite, on retranche 2
à la colonne jusqu’à ce que le plus petit des deux nombres soit égal à 2, puis on le double et on
retranche 2 à la colonne jusquà ce que les deux cases contiennent 2. On peut alors retirer 2 et on
a fini. Pour le cas général de la colonne, il suffit de considérer toutes les cases qui contiennent un
meme nombre comme "liées" : si on multiplie l’une d’entre elles par n, on multiplie aussi toutes les
autres. On remarquera que toutes nos opérations ne peuvent que faire grandir les autres colonnes,
donc on n’aura pas de problème au moment de les faire : aucune de leurs cases sera nulle.
Source : Envoi de Combinatoire 2022
Solution 270. Enoncé
Voir la solution officielle (exercice 16)
Source : Envoi de Combinatoire 2022
Solution 271. Enoncé
Montrons par récurrence forte sur le nombre de cailloux que si on a k cailloux, il existe une constante
Ck tel que chaque caillou peut bouger au plus Ck fois.
Pour k = 1, on a C1 = 1 qui convient.
Supposons le résultat vrai pour tout entier naturel l ≤ k, montrons que c’est vrai pour k. Tant que
la première opération n’est pas utilisée, à chaque étape la somme des distances entre les cailloux
augmente de 2, donc si on effectue D = 2n n2 × max(C1 , . . . , Ck−1 ) opérations, il y aura au moins


2 cailloux consécutifs à distance au moins max(C1 . . . Ck−1 ) + 1, et l’hypothèse de récurrence nous


assure que ces deux groupes ne pourront jamais se rejoindre, et donc par hypothèse de récurrence
à nouveau, on ne pourra effectuer que 2 × max(C1 , . . . , Ck−1 ) opérations de plus, ce qui reste un
nombre fini d’opérations. Si on utilise la première opération, le nombre de cailloux diminue de 1,
donc l’hypothèse de récurrence nous assure que l’on peut effectuer au plus Ck−1 étapes. Dans tous
les cas, on a majoré le nombre d’opérations.

79
Solution 272. Enoncé
L’astuce ici est de remarquer que si on prend 7 termes consécutifs a1 , . . . , a7 , alors a1 +2a2 +. . .+6a6
est un invariant du problème modulo 5. En effet,

a2 + 2a3 + . . . + 6a7 ≡ a2 + 2a3 + 3a4 + 4a5 + 5a6 + (a1 + a2 + a3 + a4 + a5 + a6 ) (mod 5)

Cepedant, initialement, on a 1 + 2 × 0 + 3 × 1 + 4 × 0 + 5 × 1 + 6 × 0 ≡ 4 (mod 5), mais ce qu’on


cherche à atteindre est 1 × 0 + 2 × 1 + 3 × 0 + 4 × 1 + 5 × 0 + 6 × 1 ≡ 2 (mod 5), donc on ne pourra
jamais l’atteindre.
Solution 273. Enoncé
Imaginons avoir 2 urnes : une de m boules et une de n boules. On souhaite connaitre le nombre de
facons de choisir k boules parmi le tout. D’un cote on a évidemment m+n k possiblités. D’un autre
cote, on peut distinguer des cas en fonction du nombre de boules choisies dans la première  urne,
que l’on note i. Il faut alors choisir j = k − i boules dans la deuxieme urne. On a donc mi nj . En
sommant sur les possibilités pour i, on a
  X mn
n+m
=
k i j
k=i+j

En remarquant que n n
, et en appliquant la formule précédente, on a
 
k = n−k

  n  2
2n X n
=
n k
k=0

Solution 274. Enoncé


a) Notons C1 , . . . , C4 les quatre convexes de l’énoncé. Comme l’intersection de trois est non vide,
on peut trouver les points A1 , A2 , A3 et A4 tel que pour tout i, Ai soit dans l’intersection de tous
les Cj pour j ̸= i. Alors on distingue deux cas :
Si le quadrilatère A1 A2 A3 A4 est convexe, alors on peut prendre le point d’intersection de ses dia-
gonales. Comme les parties C1 . . . , C4 donc convexes, il est bien évidemment dans l’intersection
des 4 parties, ce qui conclut.
Sinon, un des trois points se situe dans le triangle formé par les trois autres. On peut alors vérifier
que ce point appartient bien aux 4 convexes, puisqu’il appartient déjà a l’intersection de 3 d’entre
eux, et que le quatrième est donné par son appartenance au triangle formé par 3 points du qua-
trième.

b) La réponse est oui : il suffit de procéder par récurrence pour s’en convaincre.
Solution 275. Enoncé
La réponse est 2013 . Pour le montrer, et comme dans tout problème de ce type, il faut montrer
2 choses :

1) k ≥ 2013. En effet, si on considère un 2016-gone régulier dont les sommets alternent entre
rouge et bleu, avec un 2014-ième point bleu quelque part d’autre dans le plan, alors il nous faut au
moins 2013 droites pour séparer correctement les points : il faut que chaque arc du cercle délimité
par deux de nos points soit coupé, et une droite coupe au plus deux arcs.

2) Montrons que 2013 est suffisant : remarquons que pour toute paire de points de la meme
couleur A et B, on peut tracer deux droites qui les séparent du reste des points, puisqu’il suffit
de prendre deux parallèles de part et d’autre de (AB) suffisemment proches de A et de B pour
qu’aucun autre point ne soit dans la région. Considérons maintenant l’enveloppe convexe de nos
points, on distingue deux cas : Si il y a un point rouge sur l’enveloppe convexe, on peut le sépa-
rer du reste des points par une droite, puis appairer le reste des points rouges pour les séparer
comme vu précédement. Sinon, il n’y a que des sommets bleus sur l’enveloppe convexe, mais alors
en en prenant deux consécutifs C et D, on peut les séparer du reste des points par une unique
droite parallèle à (CD) et proche de C et D : c’est l’avantage de l’enveloppe convexe. Ensuite, on
conclut comme précédement avec les 2012 points restants : on appaire les points, ce qui nous donne
2012/2 = 1006 paires, et il nous faut 2 droites par paire, donc on a un total de 2 × 1006 + 1 = 2013
droites.
Source : IMO 2013

80
Solution 276. Enoncé
Théorème d’Erdos-Szekeres : Soient a, b deux entiers strictement positifs. Parmi toute suite de
ab+1 réels, on peut extraire une sous-suite croissante de a+1 termes ou une sous-suite décroissante
de b + 1 termes.
Démonstration : Soit (un )1≤n≤ab+1 une suite. On note an et bn la taille de la plus grande sous-
suite croissante et de la plus grande sous-suite décroissante de (um )1≤m≤n dont le dernier terme
est un (donc des entiers naturels). Par l’absurde, si pour tout n, on a an ≤ a et bn ≤ b, alors les
couples (an , bn ) ne peuvent prendre que ab valeurs, donc d’après le principe des tiroirs, il exsite
k < l tels que al = ak et bl = bk , mais alors si SPDG, uk ≤ ul , alors une sous-suite croissante de
taille ak terminant par uk peut se prolonger en une sous-suite croissante de taille ak + 1 > al se
terminant par al , absurde.
Il ne reste alors plus qu’a considerer la suite des ordonnées des points rangés
√ dans l’ordre croissant
de leurs abscisses et d’appliquer le théorème précédent à a = b = ⌊ n⌋ (on peut par exemple
utiliser le produit scalaire pour vérifier que les triangles sont bien obtus)

81
2.4.4 Géométrie

Solution 277. Enoncé


1) IA est sur la bissectrice exterieure de ABC,
\ donc (IB) et (IA B) sont prependiculaires. De
meme, (IC) et (IA C) le sont aussi, donc IBIA = 180 − ICI
\ \ A = 90, donc le quadrilatère ICIA B
est cyclique.
2) Il nous suffit de montrer que S est le centre du cercle circonscrit à IBC, puisque IA est sur
ce cercle. On sait déjà que SB = SC car S est sur la médiatrice de [BC]. Montrons donc que
SI = SB. Pour cela, montrons que BIS d = IBS. d En effet, SIBd = IAB
[ + IBA[ (relation de Chasles
dans un triangle). Or, I est sur la bissectrice issue de B, donc ABI
[ = IBC[ et par cocyclicité des
points A, B, C, S, on a également CBS = CAI = BAI, ce qui conclut.
[ [ [

Solution 278. Enoncé


On note A′ , B ′ et C ′ les projetés orhogonaux de P sur (BC), (AC) et (AB) respectivement.
Ces trois points dont alignés SSI A\ ′ B ′ C ′ = 180. Or, comme P \ B′A = P \C ′A = P
\ A′ C = 90, les
quadrilatères P C AB et P CA B sont cycliques, donc P\
′ ′ ′ ′
B ′ A′ = 180 − P
\ CB et P\B′C ′ = P
\ AC ′ =
180 − P AB. Donc les troius points dont alignés SSI P B A + P B C = 180 SSI P CB + P AB = 180
\ \ ′ ′ \ ′ ′ \ \
SSI ABCP est cyclique.

Source : Droite de Simson

82
Solution 279. Enoncé
Montrons que A, B, D, E, F sont cocycliques. On note G1 et G2 les points d’intersection des droites
d1 et d2 respectivement avec le cercle circonscrit à ABD. D’après le théorème du pole Sud dans
le triangle ADB, DG2 = BG2 et d’après le théorème du pole Nord, DG1 = BG1 . On en déduit
donc que (G1 G2 ) est la médiatrice de [BD], et donc que G1 = E et G2 = F .

Solution 280. Enoncé


Rappel : Soit Ω un cercle et P un point exterieur à Ω. Soient A et B les points de tangence a Ω
issus de P . Alors AP = BP .
D’un coté, on considère le cercle inscrit du quadrilatère. On a alors les 4 égalités indiquées sur la
figure. Donc
AB + CD = x + w + z + y = BC + AD

D’un autre coté, si AB+CD = BC +AD, alors on peut distinguer deux cas : si deux cotés adjacents
sont de meme longueur, alors les deux autre cotes le sont aussi et le quadrilatère est bel est bien
circonscriptible. Sinon, supposons que AB > BC. Alors AB − BC = AD − CD > 0. On note P
et Q les points de [AB] et [DA] respectivement tels que BP = BC et DQ = DC. Alors AP Q est
isocèle en A. Les triangles AP Q, QCD et P CB sont tous les trois isocèles, donc les bissectrices
issues des sommets A, D et B sont confondues avec les médiatrices respectives des segments [P Q],
[QC] et [P C], qui s’intersectent donc en un point. donc il existe un point équidistant de tous les
cotés du quadrilatère ie il admet un cercle iscrit.

83
Solution 281. Enoncé
On remarque que chaque point d’intersection est un centre d’homothétie positive : on a une homo-
thétie qui envoie C1 sur C2 , une qui envoie C2 sur C3 et une qui envoie C1 sur C3 . La composée des
deux premières homothéties est en fait la 3e : C1 est envoyé sur C2 puis sur C3 . Alors le centre de
cette nouvelle homothétie est aligné avec les deux autres, ie les points d’intersection des tangentes
sont alignés.

Source : Théorème de Monge

Solution 282. Enoncé


C’est une application directe de la tranformation de Ravi, puis de la réciproque de l’exercice 280.
(On peut s’en convaincre juste en regardant la figure)

Solution 283. Enoncé


On considère l’homothétie h de centre G et de rapport − 21 . L’image par h du triangle ABC est
A′ B ′ C ′ (voir exercice 171). L’orhtocentre de ABC est envoyé sur l’orthocentre de A′ B ′ C ′ , qui est
également le centre du cercle circonscrit de ABC (voir exercice 166). Donc l’image de H est O.
Donc H, G, O sont alignés. (on a de plus GH = 2GO)

84
Source : Droite d’Euler

Solution 284. Enoncé


On reprend les notations de l’exercice précédent.
Soit O′ le centre du cercle circonscrit à A′ B ′ C ′ . h envoie O sur O′ , donc GO = 2GO′ , donc O′ est
le milieu de [OH]. On a donc de plus O′ A′ = O′ D : Si on projette O′ sur [BC], on peut appliquer
Thalès pour s’en convaincre. En appliquand le meme raisonnement aux trois cotés, on a donc bien
la cocylicité des six points.

Source : Cercle d’Euler

85
3 Récapitulatif des résultats
Voici une liste des résultats important présentés au cours de l’année ou en exercice, dans tous
les domaines. Ils sont pour la plupart des classiques, essentiels à la résolution de bon nombre
d’exercices en mathématiques olympiques.

3.1 Résultats généraux

Théorème 1. Raisonnement par contraposée


Pour montrer une implication, il suffit de montrer que sa contraposée est vraie
Théorème 2. Raisonnement par récurrence
Le principe de recurrence est le suivant : Soit P (n) une proposition dépendant d’un entier
naturel n. Si on a a la fois :
1) P (0) vraie
2) Pour tout m ∈ N, P (m) =⇒ P (m + 1).
Alors pour tout n ∈ N, P (n) est vraie.

Théorème 3. Raisonnement par l’absurde


Pour montrer une implication, on peut supposer l’hypothèse vraie et la conclusion fausse et
aboutir à une contradication.
Théorème 4. Principe de récurrence forte
Soit P (n) une proposition dependant de n un entier naturel. Si on a :
1) P (0) est vraie
2) Si P (0), P (1), P (2), . . . , P (n) sont vraies, alors P (n + 1) est vraie
Alors pour tout n ∈ N, P (n) est vraie.
(Exercice 232)
Théorème 5. Factorisation de Bernoulli
1) Si n est un entier naturel, alors pour tous nombres réels a et b, on a

an − bn = (a − b)(an−1 + ban−2 + . . . + bn−1 )

2) Si n est un entier naturel impair, alors pour tous nombres réels a et b,

an + bn = (a + b)(an−1 − ban−2 + b2 an−3 − . . . + bn−1 )

Théorème 6. Binome de Newton


Pour tout entier naturel n et pour tous a, b ∈ R,
n  
X n k n−k
(a + b)n = a b
k
k=0

(Exercice 29)

86
3.2 Arithmétique

3.2.1 Divisibilité et nombres premiers

Lemme 1. Propriétés de la divisibilité


1) Si a | b et c | d, alors ac | bd (Exercice 5)
2) Si a | b et a | c, alors pour tous entiers u, v ∈ Z, a | bu + cv
Théorème 7. Théorème de Bézout
Deux entier a et b sont premiers entre eux SSI il existe u, v ∈ Z tels que au + bv = 1

Lemme 2. Lemme de Gauss


Si a | bc et a est premier avec c, alors a | b
Théorème 8. Décomposition en produit de facteurs premiers
Tout entier naturel superieur ou égal à 2 possède une unique décomposition en produit de
facteurs premiers.
Lemme 3. Nombre de diviseurs d’un entier
Si on a n = pα 1 ×. . .×pk la décompsition en produit de facteurs premiers de n, alors n possède
1 αk

(α1 + 1) × . . . × (αk + 1) diviseurs.


Lemme 4. Nombre de nombres premiers
Il existe une infinité de nombres premiers.
(Exercice 174)
Lemme 5. Fonction indicatrice d’euler
k
Si n = pα
1 × . . . × pk , alors
1 α

   
1 1
φ(n) = n 1 − ... 1 −
p1 pk

(Exercice 241)

3.2.2 Congruences

Lemme 6. Propriétés des congruences


Soit n ∈ N, si ab (mod n) et c ≡ d (mod n), alors a + c ≡ b + d (mod n) et ac ≡ bd (mod n)
Théorème 9. Théorème de Wilson
Soit p un entier. p est premier SSI (p − 1)! ≡ −1 (mod p)
(Exercice 114)
Lemme 7. Inverse modulo n
Soit n et a deux entiers. a possède un inverse modulo n SSI a est premier avec n. Si c’est le
cas, cet inverse est unique.
(Exercices 111 et 112)
Théorème 10. Petit théorème de Fermat
Soit p un nombre premier et a un nombre entier premier avec p. Alors ap−1 ≡ 1 (mod p)
(Exercice 185)
Théorème 11. Théorème d’Euler-Fermat
Soit n un nombre entier naturel et a un entier premier avec n. Alors aφ(n) ≡ 1 (mod n)
(Exercice 179)

87
3.3 Algèbre

3.3.1 Suites

Théorème 12. Terme général d’une suite arithmétique


Soit (un )n∈N une suite de réels définie par :

u0 = a
un+1 = un + q pour n > 0
Alors ∀n ∈ N, un = a + nq
(Exercice 32)
Théorème 13. Terme général d’une suite géométrique
Soit (un )n∈N une suite de réels définie par :

u0 = a
un+1 = un × q pour n > 0
Alors ∀n ∈ N, un = a × q n
(Exercice 33)

3.3.2 Polynomes

Théorème 14. Polynome de degré 2


Soit P (X) = aX 2 + bX + c, a ̸= 0 un polynome de degré 2. Alors P possède deux racines réelles
SSI b2 − 4ac ≥ 0. Si c’est le cas, alors ses racines sont

−b ± b2 − 4ac
2a
(Exercice 40)
Lemme 8. Degré de polynomes
Soient P, Q ∈ R[X]. Alors deg(P + Q) ≤ max(deg P, deg Q) et que deg(P Q) = deg P + deg Q
(Exercice 42)
Lemme 9. Racines d’un polynome
Soit P un polynome. Soit α une racine de P (ie un nombre tel que P (α) = 0), alors il existe un
polynome Q tel que P = (X − α)Q. De plus, si on note n le degré de P , alors P possède au plus
n racines distinctes.
(Exercice 129)
Lemme 10. Egalité de polynomes
Soient P et Q deux polynomes de degré au plus n. si P et Q coincident en n + 1 points, alors
P et Q sont de meme degré et de plus, tous leurs coefficients sont égaux.
Lemme 11. Polynome d’interpolation de Lagrange
Soient a0 , . . . , an , b0 , . . . bn des réels. Il existe un unique polynome de degré inferieur ou égal à
n tel que ∀k ∈ [[0, n]], P (ak ) = bk
(Exercice 200)

3.3.3 Inégalités

Théorème 15. Inégalité arithmético-géométrique


Soit n un enntier naturel et a1 , . . . an des entiers positifs. Alors
√ a1 + . . . + an
n
a1 . . . an ≤
n
(Exercice 261)

88
Lemme 12. Lemme du tourniquet
Soient a, b, c ∈ R. Alors
a2 + b2 + c2 ≥ ab + ac + bc
(Exercice 192)
Théorème 16. Inégalité de Cauchy-Schwarz
Soient a1 , . . . , an , b1 . . . , bn ∈ R. On a alors
n
!2 n
! n
!
X X X
ak bk ≤ a2k b2k
k=1 k=1 k=1

(Exercice 243)

Lemme 13. Inégalité des mauvais élèves (inégalité de Titu)


Soient x1 , ..., xn , y1 , ..., yn ∈ R+ . On a

x21 x2 (x1 + . . . + xn )2
+ ... + n ≥
y1 yn y1 + . . . + yn

(Exercice 244)
Lemme 14. Inégalité de Nesbitt
Soient a, b, c ∈ R∗+ . Montrer que

a b c 3
+ + ≥
b+c c+a a+b 2
(Exercice 245)

Théorème 17. Inégalité de Jensen


Soit
Pn f : R → R une fonction convexe. Alors ∀(λ1 , λ2 , . . . , λn ) ∈]0, 1[ , ∀(x1 , x2 , . . . xn ) ∈ R tels
n n

que k=1 λk = 1, !
Xn X n
f λk xk ≤ λk f (xk )
k=1 k=1

(Exercice 252)

89
3.4 Combinatoire

Théorème 18. Principe des tiroirs


Soit n un entier naturel non-nul. Si on a n + 1 chausettes et n tiroirs, alors il existe un tiroirs
contenant au moins 2 chaussettes.
(Exercice 212)
Lemme 15. Définition des coefficients binomiaux
Le nombre de facons de choisir k objets si on a n objets devant soi est
 
n n!
=
k k!(n − k)!

Lemme 16. Symétrie des coefficients binomiaux


Pour tout entier naturel n et pour tout k entre 0 et n,
   
n n
=
k n−k

(Exercice 61)
Théorème 19. Formule d’Euler
Pour tout entier n et pour tout k entre 1 et n − 1,
     
n n−1 n−1
= +
k k−1 k

(Exercice 28)

Lemme 17. Somme des coefficients binomiaux

n  
X n
= 2n
k
k=0

(Exercice 146)
Théorème 20. Théorème d’Erdos-Szekeres
Soit une suite de taille RS + 1. Il existe soit une sous-suite croissante de taille R + 1, soit une
sous-suite décroissante de taille S + 1.
(Exercice 363)
Lemme 18. Identité de Vandermonde
  X mn
m+n
=
k i j
i+j=k

(Exercice 273)

Théorème 21. Théorème de Helly en dimension 2


Soient n parties convexes du plan telles que l’intersection de n − 1 d’entre elles soit toujours
non-vide. Alors l’intersection des n parties est non-vide.
(Exercice 274)

90
3.5 Géométrie

Théorème 22. Somme des angles dans un triangle


La somme des angles deans un triangle vaut 180.
Lemme 19. Somme des angles dans un n-gone
La somme des angles d’un n-gone vaut (n − 2) × 180
Théorème 23. Théorème de l’angle inscrit
Soient ABCD un quadrilatère non-croisé inscrit dans un cercle. Alors CAD \ = CBD. \ La
réciproque est vraie également : si on a cette égalité d’angles, alors le quadrilatère admet un
cercle circonscrit.
(Exercice 84)
Théorème 24. Théorème de l’angle au centre
Soit ABC un triangle, O le centre de son cercle circonscrit. Alors COB
\ = 2CAB.
\
(Exercice 83)
Lemme 20. Caractérisation des quadrilatères cycliques
Si ABCD est un quarilatère non-croisé. Alors ABCD est cyclique SSI CAB
\ = 180 − CDB
\
(Exercice 85)
Théorème 25. Théorème de Thalès
Soient d et d′ deux droites parallèles Soit A un point du plan ni sur d ni sur d′ . Soient B et C
deux points distincts de d. Soient enfin B ′ et C ′ les points d’intersection de (AB) et (AC) avec d′
respectivement.
Alors
AB AC BC
= = ′ ′
AB ′ AC ′ BC
(Exercice 82)

Lemme 21. Longueur des tangentes à un cercle


Soit Ω un cercle et P un point exterieur à Ω. Soient A et B les points de tangence a Ω issus de
P . Alors AP = BP . (Exercice 81)
Théorème 26. Propriétés des symétries
Les symétries axiales et centrales conservent les longueurs et les angles.
Lemme 22. Droites remarquables dans un triangle
Dans un triangle, les hauteurs sont concourrantes, les bissectrices sont concourrantes, les mé-
diatrices sont concourrantes et les médianes sont concourrantes.
(Exercices )
Théorème 27. Théorème du Pole Sud
Soit ABC un triangle. La bissectrice issue de A, la médiatrice de [BC] et le cercle circonscrit
à ABC s’intersectent tous en un point appelé pole Sud.
(Exercice 226)
Lemme 23. Loi des Sinus
Soit ABC un triangle. On a
a b c
= =
sin A sin B sin C
(Exercice 227)
Lemme 24. Loi des Cosinus
Soit ABC un triangle de cotés a, b, c. On a

c2 = a2 + b2 − 2ab cos(C)

(Exercice 228)

91

View publication stats

Vous aimerez peut-être aussi